You are on page 1of 152

2

Published by
Vedantu Innovations Private Limited
D. No. 1081, 3rd Floor, Vistar Arcade,
14th Main Rd, Sector 3, HSR Layout
Bangalore, Karnataka, India 560 102
www.vedantu.com
Vedantu Innovations Private Ltd.
All rights reserved. No part of this book may be reproduced or utilized in any form or
by any means, electronic or mechanical including photocopying, recording or by
any information storage and retrieval system, without permission in writing from the
publishers.
Notice : Vedantu is committed to serving students with the best of the resources and
knowledge. Bearing that in mind, we have obtained all the information in this book
from sources regarded as reliable, and taken utmost care in editing and printing this
book. However, we as authors and publishers are not to be held responsible for unin-
tentional mistakes that might have crept in. Having stated that, errors (if any) brought
to our notice shall be gratefully acknowledged and rectified in upcoming editions.

Printed by
Colours Imprint
475C, Adugodi Main Road,
8th Block, Koramangala, Bengaluru 560 095
www.coloursimprint.com
3
4

Founder’s Message
Dear Student,

It gives me immense pleasure to present to you a Ready Reckoner and Practice booklet by
Vedantu - ‘TATVA’. Tatva in Sanskrit, means a collection of “Core” content/truth and isn’t that
what Vedantu brings to your table - the Core? Vedantu, at all times, ensures that you have
easy accessibility to a collection of essential concepts, theory, derivations, definitions, solved
examples, concept videos, and practice questions, important questions from competitive
examinations and I am pleased to say that all questions in ‘Tatva’ come with detailed solutions.

Tatva is the result of the constant endeavour and research done by our highly experienced
team of teachers and subject experts who compiled relevant content for you so that you come
out with flying colours in IIT JEE/NEET and Olympiads. We strongly believe and vouch for the
effectiveness and relevance of this booklet to grab the desired rank in IIT JEE/ NEET.

Key Points on ‘How to obtain optimal benefits from ‘Tatva’ ?


* We suggest starting with the Practice of ‘Tatva questions’ at the end of a chapter after
revision and Practice of basic questions.
* For daily practice of questions, we strongly recommend attempting session assignments
along with Vedantu live sessions
* Tatva equips you with Theory, Concept Videos, and Solved examples to help you revise
concepts, mark your notes, walk you through the entire summary and eventually clear
all your conceptual doubts all by yourself.
* Attempt Tatva questions post revision of 11th and 12th Grade topics. Take one topic at a
time to solve. Attempt any revision test series for NEET/JEE once you have solved Tatva
questions. Following this will not only help you during revision but also give a major
boost to your confidence.
* First solve basic level questions and gradually progress to the advanced level.
* Practice basic level questions regularly all through your journey of preparation.
* Your success is our dream-come-true

I wish you all the best.

Anand Prakash
Founder, Academic Head
Vedantu

Anand Prakash Sir has been a pioneer in producing


Top Ranks in JEE/NEET and Olympiads. He has
personally taught and mentored AIR 1,6,7 (JEE
ADVANCED) and AIR-1, 7, 9 (AIIMS), and thousands of
more students who have successfully cleared these
competitive exams in the last few years.
5

Credits
“Happiness lies in the joy of achievement
and the thrill of creative effort.”
—Theodore Roosevelt

Tatva is the brainchild of the creative Vedans who The managers who understood every aspect of what
strived tirelessly to weave success stories for you. Our the leadership were trying to accomplish and brought
heartfelt thanks to the Super Vedans who give wings so much of their own to the table and managed the
to the vision of Vedantu. execution of ‘Tatva’ immaculately.

Our leaders who have been our guiding light and Kiran Kumari Sushmitha M D
encouragement in every step: Harish Rao Akshatha J
Vamsi Sir, Anand Prakash Sir and Pulkit Sir Bhavya Bangera J Charubak Chakrabarti
Shraddha R Shetty
Our gratitude to the insightful leadership and
guidance of our leaders who dreamt about ‘Tatva’,
steered the project in the right direction and were
instrumental in making this dream into a reality:
Sahil Bhatia Sudhanshu Jain
Shubam Gupta Ajay Mittal
Arshad Shahid Jaideep Sontakke

Mathematics Team
Our true appreciation for all the Master Teachers of Assistant Teachers
Vedantu whose relentless efforts helped us in Mohammed Siddiqui Priyadharshini R
accomplishing this vision into reality. Our heartfelt Jyoti Garg Pravin S Rathore
gratitude to our creative content developers, and the Anjulika Gupta Sukamal Chatterjee
DTP team who have put in their hard work, insights, Pankaj Gupta
eagerness to execute and nurture Tatva into ‘your SME
ready handbook’ and to bring positive learning Nikhil Goyal(Team Lead) Aman Bhartiya
experience to you. Vipul Sachdeva
DTP
Raman Kumar
A loud shout out for our media team - a bunch of creative minds with contagious energy. We cannot thank
them enough.

Suraj Bhan Singh


Gadde Ranjith
Ranjeeth Ramesh
Sreyoshi Biswas
Devika Ramachandran

Special thanks and appreciation for the enthusiastic support provided by Arunima Kar, Savin Khandelwal,
Dipshi Shetty and Mohamed Anzar.

The journey of the imagination to the reality of ‘Tatva’ would not have been possible without our enthusiastic
Operations Team, our amazing Academic Team, our dedicated team of Teachers and our talented Tech
Team.
Table of Contents

SETS, RELATIONS & FUNCTIONS

 Theory ............................................................................................................................................... 9
 Solved Examples .............................................................................................................................. 23
 Exercise - 1 : Basic Objective Questions ......................................................................................... 30
 Exercise - 2 : Previous Year JEE Mains Questions .......................................................................... 35

 Exercise - 3 : Advanced Objective Questions ................................................................................. 38


 Exercise - 4 : Previous Year JEE Advanced Questions .................................................................... 44
 Answer Key ....................................................................................................................................... 45

LIMITS

 Theory ............................................................................................................................................... 47
 Solved Examples ............................................................................................................................... 51

 Exercise - 1 : Basic Objective Questions ..........................................................................................58


 Exercise - 2 : Previous Year JEE Mains Questions ........................................................................... 62
 Exercise - 3 : Advanced Objective Questions ................................................................................. 65
 Exercise - 4 : Previous Year JEE Advanced Questions .................................................................... 70
 Answer Key ....................................................................................................................................... 73
TRIGONOMETRY

 Theory ............................................................................................................................................... 75
 Solved Examples ............................................................................................................................... 81
 Exercise - 1 : Basic Objective Questions ..........................................................................................91
 Exercise - 2 : Previous Year JEE Mains Questions ........................................................................... 97
 Exercise - 3 : Advanced Objective Questions ................................................................................. 102
 Exercise - 4 : Previous Year JEE Advanced Questions .................................................................... 109
 Answer Key ....................................................................................................................................... 115

STATISTICS

 Theory ............................................................................................................................................... 117


 Solved Examples ............................................................................................................................... 123
 Exercise - 1 : Basic Objective Questions ..........................................................................................127
 Exercise - 2 : Previous Year JEE Mains Questions ........................................................................... 130

 Answer Key ....................................................................................................................................... 133

MATHEMATICAL REASONING

 Theory ............................................................................................................................................... 135


 Solved Examples ............................................................................................................................... 139
 Exercise - 1 : Basic Objective Questions ..........................................................................................142
 Exercise - 2 : Previous Year JEE Mains Questions ........................................................................... 145

 Answer Key ....................................................................................................................................... 148


01
SETS, RELATIONS & FUNCTIONS
SETS, RELATIONS & FUNCTIONS 9

SETS, RELATIONS & FUNCTIONS

SETS

1. SET 2.2 Set-Builder Form

A set is a collection of well-defined and well distinguished In this form, we write a variable (say x) representing any
objects of our perception or thought. member of the set followed by a property satisfied by each
member of the set.
1.1 Notations
For example, the set A of all prime numbers less than 10 in
The sets are usually denoted by capital letters A, B, C, etc. the set-builder form is written as
and the members or elements of the set are denoted by lower- A = {x | x is a prime number less that 10}
case letters a, b, c, etc. If x is a member of the set A, we write The symbol '|' stands for the words 'such that'. Sometimes,
x  A (read as 'x belongs to A') and if x is not a member of the we use the symbol ':' in place of the symbol '|'.
set A, we write x  A (read as 'x does not belong to A,). If x
and y both belong to A, we write x, y  A. 3. TYPES OF SETS
2. REPRESENTATION OF A SET
3.1 Empty Set or Null Set
Usually, sets are represented in the following two ways :
A set which has no element is called the null set or empty
(i) Roster form or Tabular form
set. It is denoted by the symbol  .
(ii) Set Builder form or Rule Method

2.1 Roster Form For example, each of the following is a null set :
(a) The set of all real numbers whose square is –1.
In this form, we list all the member of the set within braces
(curly brackets) and separate these by commas. For example, (b) The set of all rational numbers whose square is 2.
the set A of all odd natural numbers less that 10 in the Roster (c) The set of all those integers that are both even and odd.
form is written as :
A set consisting of atleast one element is called a
A = {1, 3, 5, 7, 9}
non-empty set.

3.2 Singleton Set

A set having only one element is called singleton set.


For example, {0} is a singleton set, whose only member is 0.
(i) In roster form, every element of the set is listed
only once. 3.3 Finite and Infinite Set
(ii) The order in which the elements are listed is
A set which has finite number of elements is called a finite
immaterial.
set. Otherwise, it is called an infinite set.
For example, each of the following sets denotes
For example, the set of all days in a week is a finite set
the same set {1, 2, 3}, {3, 2, 1}, {1, 3, 2}
whereas the set of all integers, denoted by
{............ -2, -1, 0, 1, 2,...} or {x | x is an integer}, is an infinite set.
An empty set  which has no element in a finite set A is
called empty of void or null set.
10 SETS, RELATIONS & FUNCTIONS

3.4 Cardinal Number 4. OPERATIONS ON SETS


The number of elements in finite set is represented by n(A), 4.1 Union of Two Sets
known as Cardinal number.

3.5 Equal Sets The union of two sets A and B, written as A  B (read as 'A
union B'), is the set consisting of all the elements which are
Two sets A and B are said to be equals, written as A = B, if
every element of A is in B and every element of B is in A. either in A or in B or in both Thus,

3.6 Equivalent Sets A  B = {x : x  A or x  B}

Clearly, x  A  B  x  A or x  B, and
Two finite sets A and B are said to be equivalent, if n
(A) = n (B). Clearly, equal sets are equivalent but equivalent x  A  B  x  A and x  B.
sets need not be equal.
For example, the sets A = { 4, 5, 3, 2} and B = {1, 6, 8, 9} are
equivalent but are not equal.

3.7 Subset

Let A and B be two sets. If every elements of A is an element


of B, then A is called a subset of B and we write A  B or
B  A (read as 'A is contained in B' or B contains A'). B is
called superset of A.

For example, if A = {a, b, c, d} and B = {c, d, e, f}, then


A  B = {a, b, c, d, e, f}

4.2 Intersection of Two sets


(i) Every set is a subset and a superset itself.
The intersection of two sets A and B, written as A  B
(ii) If A is not a subset of B, we write A  B.
(read as ‘A’ intersection ‘B’) is the set consisting of all the
(iii) The empty set is the subset of every set. common elements of A and B. Thus,
(iv) If A is a set with n(A) = m, then the number of A  B = {x : x  A and x  B}
subsets of A are 2m and the number of proper
subsets of A are 2m -1. Clearly, x  A  B  x  A and x  B, and

For example, let A = {3, 4}, then the subsets of A x  A  B  x  A or x  B.


are  , {3}, {4}. {3, 4}. Here, n(A) = 2 and number
of subsets of A = 22 = 4. Also, {3}  {3,4}and {2,3}
 {3, 4} The shaded region
which is common
to both the shaded
3.8 Power Set regions represents
intersection of sets
The set of all subsets of a given set A is called the power set
of A and is denoted by P(A).
For example, if A = {1, 2, 3}, then

P(A) = {  , {1}, {2}, {3}, {1,2} {1, 3}, {2, 3}, {1, 2, 3}}
For example, if A = {a, b, c, d) and B = {c, d, e, f}, then
Clearly, if A has n elements, then its power set P (A) contains A  B = {c, d}.
exactly 2n elements.
SETS, RELATIONS & FUNCTIONS 11

4.3 Disjoint Sets 4.6 Complement of a Set

Two sets A and B are said to be disjoint, if A  B =  , i.e. A If U is a universal set and A is a subset of U, then the

and B have no element in common. complement of A is the set which contains those elements
of U, which are not contained in A and is denoted by
U A'or Ac. Thus,

Ac = {x : x  U and x  A}

For example, if U = {1,2,3,4 ...} and A {2,4,6,8,...}, then, Ac =


A B
{1,3,5,7, ...}

Important Results
For example, if A = {1, 3, 5} and B = {2, 4, 6},
then A  B =  , so A and B are disjoint sets.
a) Uc =  b)  c = U c) A  Ac = U
4.4 Difference of Two Sets
d) A  Ac = 
If A and B are two sets, then their difference A - B is defined
as : 5. ALGEBRA OF SETS
A - B = {x : x  A and x  B}.
1. For any set A , we have
Similarly, B - A = {x : x  B and x  A }.
a) A  A = A b) A  A = A

2. For any set A, we have

c) A   = A d) A   = 

e) A  U = U f) A  U = A

3. For any two sets A and B, we have


g) A  B = B  A h) A  B = B  A
For example, if A = {1, 2, 3, 4, 5} and B = {1, 3, 5, 7, 9} then A
4. For any three sets A, B and C, we have
- B = {2, 4} and B - A = {7, 9}.
i) A  (B  C) = (A  B)  C
Important Results
j) A  (B  C) = (A  B)  C
(a) A- B  B –A
5. For any three sets A, B and C, we have
(b) The sets A - B , B - A and A  B are disjoint sets
k) A  (B  C) = (A  B)  (A  C)
(c) A - B  A and B - A  B
l) A  (B  C) = (A  B)  (A  C)
(d) A -  = A and A - A = 
6. If A is any set, we have (Ac)c = A.
4.5 Symmetric Difference of Two Sets 7. Demorgan's Laws For any three sets A, B and C, we have
The symmetric difference of two sets A and B , denoted by m) (A  B)c = Ac  Bc
A  B, is defined as
n) (A  B)c = Ac  Bc
A  B = (A - B)  (B - A).
o) A - (B  C) = (A - B)  ( A - C)
For example, if A = {1,2,3,4,5} and B = {1, 3,5,7,9} then
p) A - (B  C) = (A - B)  (A - C)
A  B = (A - B)  (B - A) = {2,4}  {7,9} = {2,4,7,9}.
12 SETS, RELATIONS & FUNCTIONS

Important Results on Operations on Sets Example – 3

(i) A  A  B, B  A  B, A  B  A, A  B  B Write the set {x : x is a positive integer and x2 < 30} in the
roster form.
(ii) A - B = A  Bc (iii) (A - B)  B = A  B

(iv) (A - B)  B =  (v) A  B  Bc  Ac Sol. The squares of positive integers whose squares are less
than 30 are : 1, 2, 3, 4, 5.
(vi) A - B = Bc - Ac (vii) (A  B)  (A  Bc) = A Hence the given set, in roster form, is {1, 2, 3, 4, 5}.
(viii) A  B = (A - B)  (B - A)  (A  B)
Example – 4
(ix) A - (A - B) = A  B
Write the set {0, 1, 4, 9, 16, .......} in set builder form.
(x) A - B = B -A  A= B (xi)A  B =A  B  A= B

(xii) A  (B  C) = (A  B)  (A  C) Sol. The elements of the given set are squares of integers :

0,  1,  2,  3,  4, .......
Example – 1
Hence the given set, in set builder form, is {x2 : xZ}.
Write the set of all positive integers whose cube is odd.
Example – 5

Sol. The elements of the required set are not even.


State which of the following sets are finite and which are
[ Cube of an even integer is also an even integer] infinite
Moreover, the cube of a positive odd integer is a positive (i) A = {x : x  N and x2 – 3x + 2 = 0}
odd integer.
(ii) B = {x : x  N and x2 = 9}
 The elements of the required set are all positive odd integers.
(iii) C = {x : x  N and x is even}
Hence, the required set, in the set builder form, is :
(iv) D = {x : x  N and 2x – 3 = 0}.

2k  1 : k  0, k  Z .
Sol. (i) A = {1, 2}.

Example – 2 [ x2 – 3x + 2 = 0  (x – 1) (x – 2) = 0  x = 1, 2]

Hence A is finite.
1 2 3 4 5 6 7  (ii) B = {3}.
Write the set  , , , , , ,  in the set
2 3 4 5 6 7 8
[ x2 = 9  x = + 3. But 3  N]
builder form. Hence B is finite.

(iii) C = {2, 4, 6, ......}


Sol. In each element of the given set the denominator is one
Hence C is infinite.
more than the numerator.
Also the numerators are from 1 to 7.
 3 
Hence the set builder form of the given set is : (iv) D = .  2x  3  0  x   N 
 2 
x : x  n / n  1, n  N and 1  n  7 .
Hence D is finite.
SETS, RELATIONS & FUNCTIONS 13

Example – 6 Example – 8

Which of the following are empty (null) sets ? Are the following pairs of sets equal ? Give reasons.
(i) Set of odd natural numbers divisible by 2 (i) A = {1, 2}, B = {x : x is a solution of x2 + 3x + 2 = 0}
(ii) {x : 3 < x < 4, x  N}
(ii) A = {x : x is a letter in the word FOLLOW},
(iii) {x : x2 = 25 and x is an odd integer}
B = {y : y is a letter in the word WOLF}.
(iv) [x : x2 – 2 = 0 and x is rational]
(v) {x : x is common point of any two parallel lines}.
Sol. (i) A = {1, 2}, B = {–2, –1}

Sol. (i) Since there is no odd natural number, which is divisible [ x2 + 3x + 2 = 0 (x + 2) (x + 1) = 0 x = –2, —1]
by 2.
Clearly A  B.
 it is an empty set.
(ii) A = {F, O, L, L, O, W} = {F, O, L, W}
(ii) Since there is no natural number between 3 and 4.
 it is an empty set. B = {W, O, L, F} = {F, O, L, W}.

(iii) Now x2 = 25  x = + 5, both are odd. Clearly A = B.


 The set {– 5, 5} is non-emptry.
Example – 9
(iv) Since there is no rational number whose square is 2,
 the given set is an empty set. Let A = {1, 2, 3, 4, 5}, B = {3, 4, 5, 6, 7}, C = {6, 7, 8, 9} and
(v) Since any two parallel lines have no common point, D = {7, 8, 9, 10}. Find :
 the given set is an empty set. (a) (i) A  B (ii) B  D

Example – 7 (iii) A  B  C (iv) B  C  D

Find the pairs of equal sets from the following sets, if any, (b) (i) A  B (ii) B  D (iii) A  B  C.
giving reasons :
A = {0}, B = {x : x > 15 and x < 5},
C = {x : x – 5 = 0}, D = {x : x2 = 25}, Sol. (a) (i) A  B = {1, 2, 3, 4, 5}  {3, 4, 5, 6, 7}

E = {x : x is a positive integral root of the equation = {1, 2, 3, 4, 5, 6, 7}.


x2 – 2x – 15 = 0}.
(ii) B  D = {3, 4, 5, 6, 7}  {7, 8, 9, 10}
Sol. Here we have, = {3, 4, 5, 6, 7, 8, 9, 10}.
A = {0} (iii) A  B  C = {1, 2, 3, 4, 5}  {3, 4, 5, 6, 7}  {6, 7, 8, 9}.
B=
= {1, 2, 3, 4, 5, 6, 7}  {6, 7, 8, 9} = {1, 2, 3, 4, 5, 6, 7, 8, 9}.
[ There is no number, which is greater than 15 and less
than 5] (iv) B  C  D = {3, 4, 5, 6, 7}  {6, 7, 8, 9}  {7, 8, 9, 10}.
C = {5} [ x – 5 = 0  x = 5]
= {3, 4, 5, 6, 7, 8, 9}  {7, 8, 9, 10} = {3, 4, 5, 6, 7, 8, 9, 10}.
D = {– 5, 5} [ x2 = 25  x = + 5]
(b) (i) A  B = {1, 2, 3, 4, 5}  {3, 4, 5, 6, 7} = {3, 4, 5}.
and E = {5}.
[ x2 – 2x – 15 = 0  (x – 5) (x + 3) = 0  x = 5, – 3. Out of (ii) B  D = {3, 4, 5, 6, 7}  {7, 8, 9, 10} = {7}.
these two,
(iii) A  B  C = {1, 2, 3, 4, 5}  {3, 4, 5, 6, 7}  {6, 7, 8, 9} = {3,
5 is positive integral]
4, 5}  {6, 7, 8, 9} = .
Clearly C = E.
14 SETS, RELATIONS & FUNCTIONS

Example – 10 Example – 13

If A1 = {2, 3, 4, 5}, A2 = {3, 4, 5, 6}, A3 = {4, 5, 6, 7}, find Let A = {1, 2, 3, 4, 5, 6}, B = {3, 4, 5, 6, 7, 8}. Find
(A – B)  (B – A).
 Ai and  Ai, where i = {1, 2, 3}.
Sol. We have, A = {1, 2, 3, 4, 5, 6} and B = {3, 4, 5, 6, 7, 8}.
Sol. (i)  Ai = A1  A 2  A 3 = {2, 3, 4, 5}  {3, 4, 5, 6}   A – B = {1, 2} and B – A = {7, 8}
{4, 5, 6, 7} (A – B)  (B – A) = {1, 2}  {7, 8} = {1, 2, 7, 8}.
= {2, 3, 4, 5}  {3, 4, 5, 6, 7} = {2, 3, 4, 5, 6, 7}. Some Basis Results about Cardinal Number

(ii)  Ai = A1  A 2  A 3 = {2, 3, 4, 5}  {3, 4, 5, 6}  If A, B and C are finite sets and U be the finite universal set,
then
{4, 5, 6, 7}
(i) n (Ac) = n (U) - n (A)
= {2, 3, 4, 5}  {4, 5, 6} = {4, 5}.
(ii) n (A  B) = n (A) + n (B) - n (A  B)
Example – 11 (iii) n (A  B) = n(A) + n (B), where A and B are disjoint non -
empty sets.
Let U = {1, 2, 3, 4, 5, 6, 7, 8, 9}, A = {1, 2, 3, 4},
(iv) n (A  Bc) = n (A) - n (A  B)
B = {2, 4, 6, 8}. Find :
(v) n (Ac  Bc) = n (A  B)c = n (U) - n (A  B)
C
(i) AC (ii) BC (iii) (AC)C (iv)  A  B  (vi) n (Ac  Bc) = n (A  B)c = n (U) - n (A  B)
(vii) n (A - B) = n (A) - n (A  B)
C
Sol. (i) A = Set of those elements of U, which are not in (viii) n (A  B) = n (A  B) - n (A  Bc) - n (Ac  B)
A = {5, 6, 7, 8, 9}. (ix) n (A  B  C) = n (A) + n (B) + n (C) - n (A  B) – n(B  C) –
(ii) C
B = Set of those elements of U, which are not in n(C  A) + n (A  B  C)
B = {1, 3, 5, 7, 9}. (x) If A1, A2, A3, ... An are disjoint sets , then
(iii) (AC)C = Set of those elements of U, which are not in n (A1  A2  A3  ...  An) = n(A1) + n (A2) + n (A3)
A’ = {1, 2, 3, 4} = A. + ... + n(An)
(xi) n (A  B) = number of elements which belong to exactly
(iv) A  B = {1, 2, 3, 4}  {2, 4, 6, 8} = {1, 2, 3, 4, 6, 8}.
one of A or B.
C
  A  B  = Set of those elements of U, which are not in Example – 14

 A  B = {5, 7, 9}. If A = {1, 2, 3}, B = {4, 5, 6} and C = {7, 8, 9}, verify that
A  B  C  =  A  B    A  C  .
Example – 12

If U = {x : x is a letter in English alphabet}, Sol. We have, A = {1, 2, 3}, B = {4, 5, 6} and C = {7, 8, 9}.
A = {x : x is a vowel in English alphabet}.  A  B = {1, 2, 3}  {4, 5, 6} = {1, 2, 3, 4, 5, 6} ...(1)
Find AC and (AC)C. A  C = {1, 2, 3}  {7, 8, 9}
= {1, 2, 3, 7, 8, 9} ...(2)
Sol. (i) Since A = {x : x is a letter in English alphabet}, and B  C = {4, 5, 6}  {7, 8, 9} =  ...(3)
 AC is the set of those elements of U, which are not vowels
Now A  B  C   1, 2, 3   = {1, 2, 3} ...(4)
= {x : x is a consonant in English alphabet}.
(ii) (AC)C is the set of those elements of U, which are not and  A  B   A  C  = {1, 2, 3, 4, 5, 6}  {1, 2, 3, 7, 8, 9}
consonants = {x : x is a vowel = {1, 2, 3} ...(5)
in English alphabet} = A. From (4) and (5), A  B  C    A  B    A  C  , which
C C
Hence (A ) = A. verifies the result.
SETS, RELATIONS & FUNCTIONS 15

Example – 15 Example – 17

Let U = {1, 2, 3, 4, 5, 6, 7, 8, 9}, A = {2, 4, 6, 8} and Prove that :


B = {2, 3, 5, 7}. Verify that A (B – C) = (A B) – (A C)
C C
(i)  A  B   A C  BC (ii)  A  B   A C  BC . Sol. Let x be an arbitrary element of A (B – C).
Then x A (B – C)
Sol. We have, A = {2, 4, 6, 8} and B = {2, 3, 5, 7}.  x A and x (B – C)
 x A and (x B and x C)
(i) A  B = {2, 4, 6, 8}  {2, 3, 5, 7}
 (x A and x B) and (x A and x C)
 = {2, 3, 4, 5, 6, 7, 8}  x (A B) and x (A C)
C  x {(A B) – (A C)}
 A B = {1, 9} ...(1)
 A (B – C) (A B) – (A C) ... (1)
Also AC = {1, 3, 5, 7, 9} Let y be an arbitrary element of (A B) – (A C).
and BC = {1, 4, 6, 8, 9} Then y (A B) – (A C)
 y (A B) and y (A C)
 A C  BC = {1, 3, 5, 7, 9}  {1, 4, 6, 8, 9}
 (y A and y B) and (y A and y C)
= {1, 9} ...(2)  y A and (y B and y C)
C  y A and y (B – C)
From (1) and (2),  A  B   A C  BC , which verifies the
 y A (B – C)
result.
 (A B) – (A C) A (B – C) ... (2)
(ii) A  B = {2, 4, 6, 8}  {2, 3, 5, 7} = {2} Combining (1) and (2).
C A (B – C) = (A B) – (A C).
 A B = {1, 3, 4, 5, 6, 7, 8, 9} ...(3)
Example – 18
and A C  BC = {1, 3, 5, 7, 9}  {1, 4, 6, 8, 9}

= {1, 3, 4, 5, 6, 7, 8, 9} ...(4) Prove the following :

C A B Bc Ac


From (3) and (4),  A  B   A C  BC , which verifies the
result.
Sol. Let x Bc, where x is arbitrary.
Example – 16
Now x Bc
If A and B are any two sets, prove using Venn Diagrams  xB

(i) A – B = A  BC (ii) (A – B)  B = A  B.  x A[ A B]

 x Ac
Sol.
 Bc Ac ... (1)

Conversely : Let x A, where x is arbitrary.


(i)
Now x A

 x Ac

 x Bc [ Bc Ac]

(ii)  x B

 A B

Combining (1) and (2), A B Bc Ac.


16 SETS, RELATIONS & FUNCTIONS

Example – 19 n(I M) = 15, n(M A I) = 5

n(X) = 200
Prove the following :
n(M A I) = n(M) + n(A) + n (I) –
A – B = A – (A B)
n(M A) –n (A I) – n (M I) + n (M A  I)
where U is the universal set.
= 35 + 40 + 40 – 20 – 17 – 15 + 5 = 68
Sol. Let x (A – B), where x is arbitrary. (i) Number of students passed in all three examination
Now x (A – B) = 200 – 68 = 132
 x A and x B (ii) Number of students failed in IIT or AIEEE
 (x A and x A) and x B =n (I A) = n(I) + n(A) – n (I A)
[Note this step] = 40 + 40 – 17 = 63
 x A and (x A and x B)
Example – 21
[Associative Law]
In a hostel, 25 students take tea, 20 students take coffee,
 x A and x (A B)
15 students take milk, 10 students take both tea and coffee,
 x A – (A B)
8 students take both milk and coffee. None of the them
Hence A – B = A – (A B). take tea and milk both and everyone takes atleast one

Example – 20 beverage, find the number of students in the hostel.

In a class of 200 students who appeared in a certain


examination. 35 students failed in MHTCET, 40 in AIEEE,
40 in IIT, 20 failed in MHTCET and AIEEE, 17 in AIEEE Sol.
and IIT, 15 in MHTCET and IIT and 5 failed in all three
examinations. Find how many students

(i) Did not fail in any examination.

(ii) Failed in AIEEE or IIT. Let the sets, T and C and set M are the students who drink
tea, coffee and milk respectively. This problem can be solved
by Venn diagram.

n(T) = 25; n(C) = 20; n (M) = 15

Sol. n(T C) = 10; n(M C) = 8

Number of students in hostel

= n (T C M)

n(T C M) = 15 + 10 + 2 + 8 + 7 = 42


n(M) = 35, n(A) = 40, n(I) = 40

n(M A) = 20, n(A I) = 17,


SETS, RELATIONS & FUNCTIONS 17

2.2.3 Inverse of a relation : Let A, B be two sets and let R


RELATION & FUNCTION-I be a relation from a set A to set B. Then the inverse
–1
of R, denoted by R , is a relation from B to A and is
1. INTRODUCTION defined by
–1
In this chapter, we will learn how to create a relation between R = {(b, a) : (a, b)  R}
–1
two sets by linking pairs of objects from two sets. We will Clearly, (a, b)  R  (b, a)  R
–1 –1
learn how a relation qualifies for being a function. Finally, Also, Dom (R) = Range (R ) and Range (R) = Dom (R ).
we will see kinds of function, some standard functions etc.
3. FUNCTIONS
2. RELATIONS
3.1 Definition

2.1 Cartesian product of sets A relation ‘f’ from a set A to set B is said to be a function if
every element of set A has one and only one image in set B.
Definition : Given two non-empty sets P & Q. The cartesian
product P × Q is the set of all ordered pairs of elements from Notations
P & Q i.e.
P × Q = {(p, q); p  P; q  Q} x f y (= f (x))
(Domain) input output (Range)
2.2 Relations (independent variable) (dependent variable)

2.2.1 Definition : Let A & B be two non-empty sets. Then


Domain
any subset ‘R’ of A × B is a relation from A to B.
If (a, b)  R, then we write it as a R b which is read as f: A B co-domain.
a is related to b’ by the relation R’, ‘b’ is also called
(read as : f is a function
image of ‘a’ under R. from set A to set B)

2.2.2 Domain and range of a relation : If R is a relation


from A to B, then the set of first elements in R is f
called domain & the set of second elements in R is Range of ‘f ’

called range of R. symbolically. a b

Domain of R = { x : (x, y)  R} Domain


of ‘f’
Range of R = { y : (x, y)  R} (set A)
A B
The set B is called co-domain of relation R.
Co-domain
Note that range  co-domain. (set B)

3.2 Domain, Co-domain and Range of a function

Domain : When we define y = f (x) with a formula and the domain


is not stated explicitly, the domain is assumed to be the largest set
Total number of relations that can be defined from a set A of x–values for which the formula gives real y–values.
to a set B is the number of possible subsets of A × B. If
The domain of y = f (x) is the set of all real x for which f (x) is
n(A) = p and n(B) = q, then n(A × B) = pq and total defined (real).
pq
number of relations is 2 .
18 SETS, RELATIONS & FUNCTIONS

Algo Check : Rules for finding Domain :

(i) Expression under even root (i.e. square root, fourth root etc.)
should be non–negative.
Two functions f & g are said to be equal iff
(ii) Denominator  0.
1. Domain of f = Domain of g
(iii) logax is defined when x > 0, a > 0 and a  1.
2. Co-domain of f = Co-domain of g
(iv) If domain of y = f (x) and y = g(x) are D1 and D2 respectively,
then the domain of f (x) ± g(x) or f (x) . g(x) is D1  D2. While 3. f(x) = g(x)  x  Domain.

f x 3.3 Kinds of Functions


domain of is D1  D 2 – {x: g(x) = 0}.
g x

Range : The set of all f -images of elements of A is known as the


range of f & denoted by f (A).
Range = Co-domain
Range = f (A) = {f (x) : x  A}; f f

f (A)B {Range Co-domain}.

Algo Check : Rule for finding range :

First of all find the domain of y = f (x) A B A B


(one-to-one) & (onto) (one-to-one) & (into)
(i) If domain  finite number of points

range  set of corresponding f (x) values.


Range = Co-domain
(ii) If domain  R or R – {some finite points} f f

Put y = f(x)

Then express x in terms of y. From this find y for x to be


defined. (i.e., find the values of y for which x exists).
A B A B
(iii) If domain  a finite interval, find the least and greater value
for range using monotonocity. (many-to-one) & (onto) (many-to-one) & (into)

1. Question of format :

 Q L Q  Q  quadratic
 y  ; y  ; y   L  Linear
 Q Q L
(a) One-to-One functions are also called Injective
Range is found out by cross-multiplying & creating functions.
a quadratic in ‘x’ & making D  0 (as x  R) (b) Onto functions are also called Surjective
2. Questions to find range in which-the given (c) (one-to-one) & (onto) functions are also called
expression y = f(x) can be converted into x (or some Bijective Functions.
function of x) = expression in ‘y’.
Do this & apply method (ii).
SETS, RELATIONS & FUNCTIONS 19

Relations which can not be catagorized as a function 3.4.2 Constant Function : The function f : R  R defined
by y = f(x) = c,  x  R where c is a constant is
called constant function

y
A B
8
Not a function 6
4
As not all elements of set A are associated with some
elements of set B. (violation of– point (i)– definition 2.1) 2
x’ x
–8 –6 –4 –2 2 4 6 8
–2
–4
–6
–8
A B
Not a function one-to-many f (x) = 3

An element of set A is not associated with a unique


element of set B, (violation of point (ii) definition 2.1) 3.4.3 Modulus Function : The function f : R  R defined by

Methods to check one-one mapping  x; x  0


f (x)  
1. Theoretically : If f (x1) = f (x2)  x; x  0

 x1 = x2, then f (x) is one-one. is called modulus function. It is denoted by y


= f(x) = | x |.
2. Graphically : A function is one-one, iff no line parallel
to x-axis meets the graph of function at more than one
point. y
3. By Calculus : For checking whether f (x) is One-One, 8
find whether function is only increasing or only 6
4
decreasing in their domain. If yes, then function is
2
x’ x
one-one, i.e. if f '  x   0,  x  domain or i.e., –8 –6 –4 –2 2 4 6 8
–2
if f '  x   0,  x  domain, then function is one-one. –4
–6

3.4 Some standard real functions & their graphs –8

f (x) = | x |
3.4.1 Identity Function : The function f : R  R defined
by y = f(x) = x  x  R is called identity function. Its also known as “Absolute value function’.
Properties of Modulus Function :
y
8 The modulus function has the following properties :
6
4 1. For any real number x, we have x2  x
2
x’ x
–8 –6 –4 –2 2 4 6 8
–2 2. xy  x y
–4
–6
–8 3. x  y  x  y 
 triangle inequality
f (x) = x 4. x  y  x  y 
20 SETS, RELATIONS & FUNCTIONS

3.4.4 Signum Function : The function f : R  R defined by Properties of Greatest Integer Function :

If n is an integer and x is any real number between n and n + 1,


 1; x  0 then the greatest integer function has the following properties :

f (x)   0; x  0 (1) [–n] = – [n]
 1; x  0
 (2) [x + n] = [x] + n
(3) [–x] = – [x] –1
is called signum function. It is usually denoted by
y = f(x) = sgn(x). (4) 
[x] + [– x]  1, if x  I
0, if x  I

Y–axis

1
Fractional part of x, denoted by {x} is given by x – [x]. So,
X–axis
O
 x  1; 1  x  2
–1
x  x   x    x ; 0  x  1
 x  1;  1  x  0

3.4.6 Exponential Function :


f (x) = ax, a > 0, a  1
Domain : x  R
Range : f(x)  (0, )

x Y–axis
 ; x0
Sgn(x)   x
 0; x  0

(0, 1)
3.4.5 Greatest Integer Function : The function f : R  R X–axis
O
defined as the greatest integer less than or equal to x.
It is usually denoted as y = f(x) = [x]

f (x) = ax, when a > 1


Y–axis f(x) = [x]

2 Y–axis

1
–2 –1
X–axis (0, 1)
1 2 3
X–axis
–1 O

–2

–3
f (x) = ax, when 0 < a < 1
SETS, RELATIONS & FUNCTIONS 21

3.4.7 Logarithm Function : (b) Properties of Monotonocity of Logarithm

f (x) = logax, a > 0, a1 (i) If a > 1, loga x < logay  0 < x <y
Domain : x  (0, ) (ii) If 0 < a < 1, loga x < loga y  x > y >0
Range : y  R
(iii) If a > 1 then logax < p  0 < x < ap
(iv) If a > 1 then logax > p  x > ap
Y–axis
(v) If 0 < a < 1 then logax < p  x > ap
(vi) If 0 < a < 1 then logax > p  0 < x < ap

X–axis
O
(1, 0)

f (x)= l og a x, when a > 1 If the exponent and the base are on same side of the
unity, then the logarithm is positive.
If the exponent and the base are on different sides of
unity, then the logarithm is negative.
Y–axis

4. ALGEBRA OF REAL FUNCTION


(1, 0)
X–axis In this section, we shall learn how to add two real functins,
O
subtract a real function from another, multiply a real function
by a scalar (here by a scalar we mean a real number), multiply
f(x)= loga x, when 0 < a < 1
two real functions and divide one real function by another.

4.1 Addition of two real functions


(a) The Principal Properties of Logarithms
Let f : X  R and g : X  R by any two real functions, whre
Let M & N are arbitrary positive numbers, a > 0, a  1, X  R. Then, we define (f + g): X  R by
b > 0, b  1. (f + g) (x) = f (x) + g(x), for all x  X.
c
(i) logb a = c a = b
4.2 Subtraction of a real function from another
(ii) loga (M . N) = loga M + loga N
(iii) loga (M/N) = loga M – loga N Let f : X  R be any two any two real functions, whre X  R.
(iv) N
loga M = N loga M Then, we define (f – g): X  R by

l og c a (f – g) (x) = f (x) – g(x), for all x  X.


(v) log b a  , c > 0, c  1.
l og c b 4.3 Multiplication by a scalar

(vi) a l ogcb  bl ogca , a, b, c > 0, c  1. Let f : X  R be a real valued function and  be a scalar. Here
by scalar, we mean a real number. Then the product  f is a
function from X to R defined by ( f) (x) =  f(x), x  X.

4.4 Multiplication of two real functions

(a) loga a = 1 The product (or multiplication) of two real functions


(b) logb a . logc b . loga c = 1 f : X  R and g : X  R is a function fg : X  R defined by
(c) loga 1 = 0 (fg) (x) = f(x) g(x), for all x  X.
x This is also called pointwise multiplication.
(d) e x lna  e ln a  a x
22 SETS, RELATIONS & FUNCTIONS

4.5 Quotient of two real functions 5. PERIODIC FUNCTION

Let f and g be two real functions defined from X  R where Definition : A function f (x) is said to be periodic function, if there
X  R. The quotient of f by g denoted by f /g is a function exists a positive real number T, such that
defined by f  x  T   f  x  ,  x R.
Then, f (x) is a periodic function where least positive
f  f x value of T is called fundamental period.
 x 
g g  x  , provided g(x)  0, x  X.
Graphically : If the graph repeats at fixed interval, then function
is said to be periodic and its period is the width of
4.6 Even and Odd Functions that interval.
Some standard results on periodic functions :
Even Function : f (–x) = f (x),  x  Domain Functions Periods
n n n n
The graph of an even function y = f (x) is symmetric about the (i) sin x, cos x, sec x, cosec x ; if n is even.
y–axis. i.e., (x, y) lies on the graph  (–x, y) lies on the graph. 2; (if n is odd or fraction)
(ii) tann x, cotn x ; n is even or odd.
(iii) |sin x|, |cos x|, |tan x| 
y = x2 |cot x|, |sec x|, |cosec x|
Y–axis
(iv) x – [x], [.] represents 1
greatest integer function
(–x, y) (x, y) (v) Algebraic functions period does not exist
e.g., x , x2, x3 + 5, ....etc.
X–axis
O Properties of Periodic Function
(i) If f (x) is periodic with period T, then
Graph of an even function
(a) c . f (x) is periodic with period T.
(b) f (x ± c) is periodic with period T.
Odd Function : f (– x) = –f (x),  x  Domain (c) f (x) ± c is periodic with period T.
where c is any constant.
The graph of an odd function y = f (x) is symmetric about origin
i.e. if point (x, y) is on the graph of an odd function, then (ii) If f (x) is periodic with period T, then
(–x, –y) will also lie on the graph. k f (cx + d) has period T/|c|,
i.e. Period is only affected by coefficient of x
where k, c, d  constant.
3
Y–axis y=x (iii) If f1(x), f2(x) are periodic functions with periods T1, T2
(x, y)
respectively, then we have, h(x) = a f1(x) + b f2(x) has
period as, LCM of {T1, T2}

X–axis
O

 a c e  LCM of  a, c, e 
(–x, –y) Graph of an odd function (a) LCM of  , ,  
 b d f  HCF of  b, d, f 
(b) LCM of rational and rational always exists.
LCM of irrational and irrational sometime exists.
But LCM of rational and irrational never exists.
e.g., LCM of (2 , 1, 6 ) is not possible as
2 , 6  irrational and 1  rational.
SETS, RELATIONS & FUNCTIONS 23

SOLVED EXAMPLES

RELATION & FUNCTION-I Example – 5

Example – 1 Determine domain and range of :-

x   2x  
If   1, y  1   2, 1 find values of x and y.. R   x  4,  : 4  x  6, x  N 
3   2  x  

x   7 
Sol. 1  2 & y 1  1 Sol. R   8, 3 ,  9,  , 10, 2 
3
  3 
x = 3 and y = 2.
so, domain = {8, 9, 10}
Example – 2
 7 
range =  3,  ,  2 
If A = {1, 2}, find A × A × A  3 

Example – 6
Sol. A × A × A = {(x, y, z), x  A, y  A, z  A}
so, A × A × A = {(1, 1, 1), (1, 1, ), (1, 2, 1), (2, 1, 1), Let A = {1, 2}. List all the relation on A.

(2, 2, 2), (2, 2, 1), (2, 1, 2), (1, 2, 2)}


Sol. Given A = {1, 2}
Example – 3 A × A = {(1, 1), (1, 2), (2, 1), (2, 2)}

Following figure shows a relation between sets P and Q. Since relation R from set A to set A is a subset of A × A
Write this relation in (i) set builder form, (ii) roster form  All the relations on A are :
, {(1, 1)}, {(1, 2)}, {(2, 1)}, {(2, 2)}, {(1, 1), (1, 2)},
1
–1
{(1, 1), (2, 1)}, {(1, 1), (2, 2)}, {(1, 2), (2, 1)}, {(1, 2),
1
(2, 2)}, {(2, 1), (2, 2)}, {(1, 1), (1, 2), (2, 1)}, {(1, 1),
–2 4 (1, 2), (2, 2)}, {(1, 1), (2, 1), (2, 2)}, {(1, 2), (2, 1),
–4 16 (2, 2)}, {(1, 1), (1, 2), (2, 1), (2, 2)}.
2
Since n(A × A) = 4, the number of all relations in the set
P 4
A = 2 i.e., 16.

Example – 7
Sol. It is clear, that relation R is “y is the square of x”.
2
(i) In set builder form, R = {(x, y) : y = x , x  P, y  Q} Find the domain and range of the following functions

(ii) In roster form,


 x 2  1  
(i)  x,  : x  R, x  1
R = {(1, 1), (–1, 1), (2, 4), (–2, 4) (4, 16)}  x 1  
Example – 4

Let R be the relation on Z defined by R = {(a, b);  1  


a, b,  Z, a – b is an integer}. Find domain and (ii)  x, 2  : x  R, x   1
range of R.  1  x  

Sol. As for any two integers a & b, a – b an integer hence domain


 x 2  1  
and range is all integers. Sol. (i) Let f  x    x,  : x  R, x  1
 x 1  
24 SETS, RELATIONS & FUNCTIONS

Clearly, f is not defined when x = 1 Example – 9


 f is defined for all real values of x except x = 1
Let f, g : R  R be defined respectively by f(x) = x + 1, g(x)
 Domain = R – {1} = 2x – 3. Find f + g, f – g and f /g.

x2 1
Let y  x  1 as x  1 Sol. Let f(x) = x + 1, g(x) = 2x – 3
x 1
 f + g = f(x) + g(x) = (x + 1) + (2x – 3)
 x=y–1
= 3x – 2
Clearly, x is not defined when y = 2 as x  1
f – g = f (x) – g(x) = (x + 1) – (2x – 3)
 Range = R – {2}.
= x + 1 – 2x + 3 = –x + 4
 1  
(ii) Let f  x    x,  : x  R, x   1 f x
 1  x2   f x 1 3
  ;x
g g  x  2x  3 2
1 2
Clearly, f  x   2
is not defined when 1 – x = 0
1 x Example – 10

i.e., when x = ± 1 A function f is defined on the set {1, 2, 3, 4, 5} as follows :


 Domain = R – {1, –1}
 1  x if 1  x  2
1 
Further, y  f  x    2x  1 if 2  x  4
1 x2 3x  10 if 4  x  6

 1  x   1y
2

 1
x  1   
y 1 (i) Find the domain of the function.
 y y
(ii) Find the range of the function.
 x is defined when y (–, 0) [1, ). (iii) Find the values of f(2), f(3), f(4), f(6).
Range = (–, 0) [1, ).
Sol. (i) Domain : {1, 2, 3, 4, 5}
Example – 8
(ii) Range :

 x2   f(1) = 1 + 1 = 2 f(4) = 3(4) – 10 = 2


Let f   x, 1  x 2  : x  R  be a function from R to R.
   f(2) = 2(2) – 1 = 3 f(5) = 3(5) – 10 = 5
Determine the range of f. f(3) = 2(3) – 1 = 5
So, range is {2, 3, 5}
Sol. Clearly f : R  R is a function such that
Example – 11
x2
f x  Find the domain of definition of the following
1  x2
 x –1  x  2 
x2 function : f  x  
Let y 
2 2 2
. So x = y (1 + x ). Therefore, x (1 – y) = y which  x – 3  x – 4 
1 x2

y y
implies x   . Since x  R, 1  y  0 .  x –1  x  2   0
1 y Sol. For f (x) to be defined and x  3, 4
 x – 3  x – 4 
i.e. y  [0, 1). Thus range is [0, 1)
By wavy – curve method the domain of definition of f (x) is
the set
x   –  , – 2  1, 3   4,   .
SETS, RELATIONS & FUNCTIONS 25

Example – 12  x   –  , – 2    –1,   using number line rule as shown


Find the domain of definition of the following in figure.
+ – +
function : f  x   log 1  2x – 3
2 –2 –1
But x  – 3 {from (ii)}
Sol. For f(x) to be defined log1/2 (2x – 3)  0  x   –3, – 2    –1,   ...(a)
 2x – 3  1
Case II : When x + 3 0 ...(iii)
 x2 ....(1)
Also 2x – 3 > 0 –  x  3 – 2
 0
3 x2
 x . ....(2)
2
Combining (1) and (2) we get the required values of x. –  x  5
 0
3
 x  2
Hence the domain of definition of f(x) is the set  , 2
2 
x  5  0

x  2
Example – 13
 x   –5, – 2  using number line rule as shown in figure.
If y =3[x] +1 =2[x – 3] +5, then find the value of
[x + y], where [.] represents greatest integer function. + +

–5 –2
Sol. We are given that 3[x] + 1 = 2([x] – 3) + 5 But x<–3 {from (iii)}
 [x] = –2
 x   –5, – 3 ...(b)
 y = 3(–2) + 1 = –5
Hence [x + y] = [x] + y = – 2 – 5 = – 7 Thus from (a) and (b), we have;
x   –3, – 2    –1,     –5, – 3
Example –14
 x   –5, – 2    –1,  
x 3  x
Solve 1
x2
Example – 15

The value of x if | x + 3 | > | 2 x – 1 | is


x 3  x
Sol. –1 0
x2  2   2 
(a)   , 4  (b)   ,  
 3   3 
x 3  x – x – 2
 0 (c) (0, 1) (d) None of these
x2

x 3 – 2 Sol. Squaring both sides, we get


 0 ...(i) 2 2
x2 |x + 3| > |2x – 1|
Now two cases arises : or {(x + 3) – (2x – 1)} {(x + 3) + (2x – 1)} > 0
Case I : When x + 3 0 ...(ii)  {(–x + 4) (3x + 2)} > 0
x3–2 – + –
 0 –2/3 4
x2
 2 
x 1  x  , 4
 0  3 
x2
Hence, (a) is the correct answer.
26 SETS, RELATIONS & FUNCTIONS

Example – 16 Example – 18

The value of x, loge (x – 3) < 1 is Solve (x + 1)2 + (x2 + 3x + 2)2 = 0


(a) (0, 3) (b) (0, e)
(c) (0, e + 3) (d) (3, 3 + e) Sol. Here, (x + 1)2 + (x2 + 3x + 2)2 = 0 if and only if each term is zero
simultaneously,
(x + 1) = 0 and (x2 + 3x + 2) = 0
Sol. From definition of logarithms x – 3 > 0
i.e., x = –1 and x = –1, –2
or x>3 ...(i)
The common solution is x = –1
Also, e > 1, given inequality may written as Hence, solution of above equation is x = – 1
1
 x – 3 < (e)
Example – 19
or x<3+e ...(ii)
Find the domain of the function;
Using (i) & (ii)
1
f x    x  2
 x  (3, 3 + e) log10 1 – x 
Hence, (d) is the correct answer.

Example – 17 1
Sol. f  x    x2
log10 1 – x 
The value of x if log1/2 x  log1/3 x is
{as we know; loga x is defined when x, a > 0 and a  1 also
(a) (0, 1] (b) (0, 1) loga 1 = 0}
(c) [0, 1) (d) None of these Thus, log10 (1 – x) exists when, 1 – x > 0 ...(i)

1
Sol. Case I. When x  1 and x > 0. also exists when, 1 – x > 0
log10 1 – x 
log1/2 x  log1/3 x and 1 – x  1 ...(ii)
 x < 1 and x  0 ...(iii)
log x x log x x
  also we have x  2 exists when x + 2  0
1 1
log x   log x   or x –2 ....(iv)
2
  3
1
Thus, f  x    x  2 exists when (iii) and (iv)
log10 1 – x 
1 1
  both holds true.
 log x 2  log x 3
 –2 x < 1 and x 0

1 1  x   –2, 0    0 , 1
 
log x 2 log x 3
Example – 20
 logx 2  logx 3 where x  1
Find value of ‘x’ so that log|x| |x – 1|  0.
which is only possible, if 0 < x < 1
Case II. When x = 1.
Sol. It is clear that |x| > 0 and |x|  1
 log1/2 x = log1/3 x, equality sign holds true.
 x  0, –1, 1
Combining the above cases, Also, |x – 1| > 0 x  1,
0 < x  1 or x  (0, 1]. Case I : For 0 < |x| < 1
Hence, (a) is correct answer.  x   –1, 0    0, 1 ...(1)
SETS, RELATIONS & FUNCTIONS 27

and log|x| |x – 1|  0
1
 log|x| |x – 1|  log|x| 1 so that Case II : For x  , |2x – 1| = 2x – 1
2
0 < |x – 1|  1
 –1  x – 1  1 and x  1  2x – 1 = 3[x] + 2{x}.
 0  x  2 and x  1  2x – 1 = 3(x – {x}) + 2{x}.
 x   0 , 1  1, 2 . ...(2) {x} = x + 1
Now 0 {x} < 1
From (1) and (2) we have x  (0, 1)
Case II : For |x| > 1  0 x + 1 < 1. –1 x < 0.
 x < –1 or x > 1 1
which is not possible since x  .
i.e. x   – , – 1  1,   ...(3) 2
and log|x| |x – 1|  0
1
 |x – 1|  1 Hence x = is the only solution.
4
 x – 1  1 or x – 1  –1
 x  2 or x  0
Example – 23
i.e. x   –  , 0   2 ,  ...(4)
Find the domain of definition of the following
From (3) and (4), we find that x   –  , – 1   2, 
function : f  x   log 1  2x – 3
Hence x   0, 1   – , – 1   2,  2

Example – 21
Sol. For f(x) to be defined log1/2 (2x – 3)  0
If y =3[x] +1 =2[x – 3] +5, then find the value of  2x – 3  1
[x + y], where [.] represents greatest integer function.  x2 ....(1)
Also 2x – 3 > 0
Sol. We are given that 3[x] + 1 = 2([x] – 3) + 5
 [x] = –2 3
 x . ....(2)
 y = 3(–2) + 1 = –5 2
Hence [x + y] = [x] + y = – 2 – 5 = – 7 Combining (1) and (2) we get the required values of x.

3
Example – 22 Hence the domain of definition of f(x) is the set  , 2
Solve the equation |2x – 1| = 3[x] + 2{x} for x. 2 
where [.] represents greatest integer function and {}
represents fraction part function.
Example – 24

1 Find the range of the following function :


Sol. Case I : For x < , |2x – 1| = 1 – 2x
2
 1 – 2x = 3[x] + 2{x}. f  x   ln x 2  4x  5
 1 – 2x = 3(x – {x}) + 2{x} .
 {x} = 5x –1.
Now 0 {x} < 1 2
 0 5x – 1 < 1. Sol. Here f (x)  1n x 2  4x  5  1n  x  2 1

1 2 i.e. x2 + 4x + 5 takes all values in [1, )


 x  [x] = 0
5 5
 f (x) will take all values in [0, ).
 x = {x}  x = 5x – 1
Hence range of f (x) is [0, ).
1
 x = , which is a solution.
4
28 SETS, RELATIONS & FUNCTIONS

Example – 25  one-to-one / injective function.


Let A = {x : –1  x  1} = B for a mapping f : A  B. For each Also Range for domain x  [–1, 1] = [–1, 1] = co-domain
of the following functions from A to B, find whether it is  Surjective Function
surjective or bijective. Bijective Function
(a) f (x) = |x| (b) f (x) = x |x|  Similar approach for other questions.
3
(c) f (x) = x (d) f (x) = [x]
Example – 26
x
(e) f (x) = sin Check whether the function :
2
f (x) = 2x3 + 3x2 + 6x + 5 is
Sol. (a) f (x) = |x| one-to-one or many-to-one
Graphically we can see that for x  [–1, 1]
y = |x|  [0, 1] Sol. f (x) = 2x3 + 3x2 + 6x + 5
Since, Range ([0, 1])  co-domain (B = [–1, 1]) f ’ (x) = 6(x2 + x + 1) > 0  x  R
many-to-one as (a > 0 & D < 0) for x2 + x + 1
y  f (x) is increasing function on its entire domain
 one-to-one function.
f1 1

Example – 27
x
x1 x2
–1 1 1
Find the range of the function y 
2  sin 3x

Sol. Clearly, as Denominator (2 – sin 3x)  0


 into function
(sin 3x  2)
 f : [–1, 1]  [–1, 1], f (x) = |x|
is many-to-one & into  Domain : x  R

2
 x , x  0 1
(b) f (x) = x |x|   2 We have, y
 x , x  0 2  sin 3x

Note : (sin 3x) can be seperated & written as a function of y


y 2
x
1
 2 – sin 3x = y
1

–1 x
x1 1 2y  1
 sin 3x 
y
–x2 –1
for x to be real

2y1
 1 1 (since, –1  sin 3x  1)
By calculus y

 2x, x  0 Ask : Can we cross multiply ‘y’ across inequality?


f ' x   
2x, x  0
Since, f’(x)  0 always over the entire domain.
SETS, RELATIONS & FUNCTIONS 29

Example – 28
1
Yes!! as y > 0 (why??)  y
2  sin 3x
Find domain for f (x) = cos  sin x  .

2y  1 Alternate 2y  1
1  1  1  1
y y
Sol. f (x) = cos  sin x  is defined, if
–y  2y – 1  y cos (sin x)  0
2y  1 2y  1 As, we know
1  0  1  0
y y –1  sin x  1 for all x
cos  0
3y  1 y 1 (Here,  = sin x lies in the 1st and 4th quadrants)
2y – 1  – y & 2y – 1  y 0 0
y y i.e. cos (sin x)  0, for all x
i.e. x  R.
1 Thus, domain f (x)  R
 y  y 1
3
Example – 29

Solve for x
1 
 Range : y   , 1  |x| + |x + 4| = 4
3 

Sol. |x| + |x + 4| = 4
As we know, |x| + |y| = |x – y|, iff xy  0
x(x + y)  0
O y3 1 Using number line rule,
+ – +
Alternate Method : –4 0
 x  [–4, 0]
1
y
2  sin 3x Example – 30

we know, –1  sin 3x  1 x x2
Solve x  1  x 
 1  –sin 3x  1 x 1

 1  2 – sin 3x  1

1 1  1 x
Sol. Let f  x   and g(x) = x
   x 1
1  2  sin 3x  3
x x2
 f (x) + g(x)  x 
1  x 1 x 1
 Range y   , 1
3  Using, | f (x)| + |g(x)| = |f (x) + g(x)|
Inequality changes upon reciprocating as all expressions i.e. f (x) . g(x)  0
across inequality are (positive). x x2
 .x  0  0
x 1 x 1
+
–4 1
 x  {0}  (1, )
30 SETS, RELATIONS & FUNCTIONS

EXERCISE - 1 : BASIC OBJECTIVE QUESTIONS

SETS 8. If Y  {1, 2} = {1, 2, 3, 5, 9}, then

Definition and Types of Sets (a) The smallest set of Y is {3, 5, 9}

1. Which of the following is the empty set? (b) The smallest set of Y is {2, 3, 5, 9}

(a) {x : x is a real number and x2 - 1 = 0} (c) The largest set of Y is {1, 2, 3, 5, 9}

b) {x : x is a real number and x2 + 1 = 0} (d) The largest set of Y is {2, 3, 5, 9}

(c) {x : x is a real number and x2 - 9 = 0} 9. Given the sets A = {1, 2, 3}, B = {3, 4}, C = {4, 5, 6}, then

(d) {x : x is a real number and x2 = x +2} A  BC b g is


2. Let A = {2, 3, 4} and X = {0, 1, 2, 3, 4}, then which of the (a) {1, 2, 3, 4, 5, 6} (b) {1, 2, 4, 5}
following statements is correct
(c) {1, 2, 3, 4} (d) {3}
(a) {0}  Ac w.r.t. X (b)   Ac w.r.t. X

(c) {0}  Ac w.r.t.X (d) 0  Ac w.r.t. X.


b
10. If A and B are any two sets, then A  A  B is equal tog
(a) Bc (b) Ac
3. In a city 20% of the population travels by car, 50% travels
by bus and 10% travels by both car and bus. Then, persons (c) B (d) A
travelling by car or bus is 11. If A = {x : x = 4n + 1, 2  n  5}, then number of subsets of A
(a) 80 % (b) 40% is
(c) 60% (d) 70% (a) 16 (b) 15
4. If A = {1, 2, 3}, B = {a, b}, then A × B is given by
(c) 4 (d) None of these
(a) {(1, a), (2, b), (3, b)}
12. If A = {1, 2, 3, 4}, B = {2, 3, 5, 6} and C = {3, 4, 6, 7}, then
(b) {(1, b), (2, a)}
(a) A - (B  C) = {1, 3, 4} (b) A - (B  C) = {1, 2, 4}
(c) {(1, a), (1, b), (2, a), (2, b), (3, a), (3, b)}
(c) A - (B  C) = {2, 3} (d) A - (B  C)= {1}
(d) {(1, a), (2, a), (2, b), (3, b)}
5. The set of intelligent students in a class is Properties of cardinality and its practical applications

(a) a null set 13. X and Y are two sets such that n (X) = 17, n(Y) = 23,
n( X  Y) = 38 then n ( X  Y) is
(b) a singleton set
(a) 4 (b) 2
(c) a finite set
(c) 6 (d) None of these
(d) not a well defined collection
14. If X and Y are two sets such that X  Y has 18 elements, X
Operations on sets (Union and Intersection)
has 8 elements and Y has 15 elements; then number of
6. If A = {1, 3, 5 , 7, 9, 11, 1 3, 1 5, 1 7}, elements in X  Y are
B = {2, 4, .......,18}and N is the universal set, then
(a) 5 (b) 8
A c  ((A  B)  B c ) is
(c) 6 (d) None of these
(a) A (b) N
15. If S and T are two sets such that S has 21 elements, T has 32
(c) B (d) None of these
elements, and S  T has 11 elements, then number of
7. Let A = {x : x is a multiple of 3} and elements S  T has
B = {x : x is a multiple of 5}. Then A  B is given by
(a) 42 (b) 50
(a) {3, 6, 9...} (b) {5, 10, 15, 20, ...}
(c) 48 (d) None of these
(c) {15, 30, 45, ...} (d) None of these
SETS, RELATIONS & FUNCTIONS 31

16. In a committee 50 people speak French, 20 speak Spanish 25. The value of x for which 12 x – 6 < 0 and 12 – 3 x < 0
and 10 speak both Spanish and French. The number of (a)  (b) R
persons speaking at least one of these two languages is (c) R – {0} (d) none of these
(a) 60 (b) 40
x 3 x 1 x  2
(c) 38 (d) None of these 26. The value of x for which x  and
4 2 3
17. In a group of 1000 people, there are 750 people who can 2 – x > 2x – 8
speak Hindi and 400 who can speak English .Then number (a) [–1, 10/3] (b) (–1, 10/3)
of persons who can speak Hindi only is (c) R (d) none of these
(a) 300 (b) 400 27. Solve for x :
(c) 600 (d) None of these
4 4
18. In a statistical investigation of 1,003 families of Calcutta, it  2x  1  x  1  x  2  0
(i) 4
was found that 63families had neither a radio nor a T.V, 794  x  2  x  4
families had a radio and 187 had a T.V. The number of families
in that group having both a radio and a T.V is (a) [½, 2) (b) R
(c)  (d) (½, 2)
(a) 36 (b) 41
4 3 2
(ii) (x – 2) (x – 3) (x – 4) (1 – x) < 0
(c) 32 (d) None of these
19. If A has 3 elements and B has 6 elements, then the minimum (a) (1, 3) (b)   , 1   3,  
number of elements in the set A  B is
(c)   ,1  3,   (d) none
(a) 6 (b) 3
1
(c)  (d) None of these (iii) x 2
x
20. The set A = {x : x  R, x2 = 16 and 2x = 6} equals
(a)  0,   (b) R
(a)  (b) {14, 3, 4}
(c)  (d) 0,  
(c) {3} (d) {4}
x2
INEQUALITIES (iv)
x 1
0

21. The set of values of x satisfying the inequalities


(x – 1) (x – 2) < 0 and (3x – 7) (2x – 3) > 0 is (a) 1,   (b) 1,  
(a) (1, 2) (b) (2, 7/3)
(c) 0  1,   (d) none
(c) (1, 7/3) (d) (1, 3/2)
22. If x2 + 6x – 27 > 0 and x2 – 3x – 4 < 0, then 28. If c < d, x2 + (c + d) x + cd < 0, then x belongs to.
(a) x > 3 (b) x < 3 (a) (–d, –c] (b) (–d, –c)
(c) 3 < x < 4 (d) x = 7/2 (c) R (d) 

23. Find the set of values of 'x' for which the given condition x 7
29. Solution of  2 is
is true (x – 1) (x – 3) (x + 5) > 0 x 3
(a) (–5, 1)  (3, ) (b) (–1, 5)
(a)  3,   (b)  ,  13
(c) [–5, 1]  [3, ) (d) none of these
(c) (–13, –3) (d) none of these
 x  1  2 30. The set of values of x which satisfy the inequations
24. The value for which
x x2
5x + 2 < 3x + 8 and  4 is
x 1
(a) (0, 1) (b)   ,  1
(a)  , 1 (b) (2, 3)
(c)   , 0  (d)  1, 0 
(c)  , 3 (d)  , 1   2, 3
32 SETS, RELATIONS & FUNCTIONS

31. If x2 – 1 < 0 and x2 – x – 2 > 0, then x lies in the interval set


39. Find the domain y  1  x  x 5
(a) (1, –1) (b) (–1, 1)
(c) (1, 2) (d) {–1} (a) x  (b) y (–, 1]
(c) x (–, 1] [5, ) (d) none of these
x 2  3x  4
32. The solution set of  1, x  R is 40. The domain of the function
x 1

(a)  3,   (b)  1, 1   3,  


f(x)  x  3  2 x  4  x  3  2 x  4 is

(a) [4, ) (b) (-, 4]


(c)  1, 1  3,   (d) none
(c) (4, ) (d) (–, 4)
1 1
33. If  , then : 3 x
a b 41. The domain of the function f  x   log10
x
(a) |a| > |b| (b) a < b
(c) a > b (d) none of these
 3
34. If –2 < x < 3, then : (a)  0,  (b) (0, 3)
 2
(a) 4 < x2 < 9 (b) 0  | x |  5
 3  3
(c) 0  x 2  9 (d) None of these (c)  ,  (d) )  0, 
 2  2

35. x  2  x2 42. The domain of the function

(a) x (1, ) (b) x (– , –1)(1, ) 1


f x   2
x   x   6
(c) x  1, 2  (d) x   2,  
where [ ] denotes greatest integer function
Functions : Definition, Domain, Range and Types of Functions
(a) R – [–2, 4) (b) R – {–3, 2}
36. Let A = [–1, 1] and f : A  A be defined as f (x) = x |x| for all
x  A, then f (x) is (c) R (d) R – {2, 3}

(a) many-one into function 43. The domain of the function f (x) = loge (x – [x]), where [.]
denotes the greatest integer function, is
(b) one-one into function
(a) R (b) R  Z
(c) many-one onto function
(c) (0, + ) (d) None of these
(d) one-one onto function
37. The function f : R  R defined by 2x  1
44. Find the Range y 
f (x) = (x – 1) (x – 2) (x – 3) is x5

(a) one-one but not onto (a) y 2 (b) x  5


(b) onto but not one-one (c) y 5 (d) none of these
(c) both one-one and onto
x
(d) neither one-one nor onto 45. Range of the function f  x   is
1 x2

1 2 1 (a) (–, ) (b) [–1, 1]


38. Find the domain of f (x)= x 
x 5 x7
 1 1
(a) x [–7, 5] (b) x (5, ) (c)   ,  (d)   2, 2 
 2 2
(c) x (–, 7) (d) none of these
SETS, RELATIONS & FUNCTIONS 33

54. Solve for x :


1 2
46. The range of the function f  x   x  2 (i) |x + 4| > 5
x 1
(a)  , 1 (b)  , 9
(a) [1, ) (b) [2, ) (c) R – [–9, 1] (d) none
(ii) |x + 2| < 4
3 
(c)  ,   (d) None of these (a) (–6, 2) (b) (–6, 0)
2  (c) (–6, 2] (d) (0, 2)

Standard Functions Modulus, Greatest Integer & Logrithm x2  6


(iii) 1
Function 5x
47. |3x + 7| < 5, then x belongs to
(a)  ,  3
(a) (–4, –3) (b) (–4, –2/3)
(c) (–5, 5) (d) (–5/3, 5/3) (b)  ,  3   3,  
48. The least integer satisfying, (c) R

(d)  ,  3   2, 0    0, 2   3,  


 27  x   27  9x 
49.4 – < 47.4 –   is
10  10 
1
55. Solution of x  < 4 is
(a) 2 (b) 3 x

(c) 4 (d) none of these


  
(a) 2  3, 2  3  2  3,  2  3 
49. Solution of |3x – 2| > 1 is
(a) [1/3, 1] (b) (1/3, 1) 
(b) R  2  3, 2  3 
(c) {1/3, 1} 
(d) , 1 3   1,   
(c) R  2  3,  2  3 
(d) none of these
50. If 3 < | x | < 6, then x belongs to :
(a) (–6, –3)  (3, 6) (b) (–6, 6) 1
56. If f (x)  , then domain of f (x) is
(c) (–3, +3)  (3, 6) (d) None of these | x | x

51. If –5 < x < 4, then : (a) (–, 0) (b) (–, 2)


(c) (–, ) (d) None of the above
(a) 0  | x |  4 (b) 4 < |x| < 5 57. If | x | > 5, then
(c) 0  | x |  5 (d) none of these (a) 0 < x < 5 (b) x < –5 or x > 5
(c) –5 < x < 5 (d) x > 5
52. If |x| < x, then : 58. If [x]2 = [x + 2], where [x] = the greatest integer less than or equal
(a) x is a positive real number to x, then x must be such that
(b) x is a non-negative real number (a) x = 2, –1 (b) x  2, 3 
(c) there is no x satisfying this inequality
(d) x is a negative real number (c) x   1, 0  (d) none of these

53. |2x – 3| < |x + 5|, then x belongs to 59. The value of x, loge (x – 3) < 1 is

(a) (–3, 5) (b) (5, 9) (a) (0, 3) (b) (0, e)


(c) (–2/3, 8) (d) (–8, 2/3) (c) (0, e + 3) (d) (3, 3 + e)
34 SETS, RELATIONS & FUNCTIONS

60. Solve for x : Miscellanious


(i) log0.2 (x + 5) > 0 65. The number of real solutions of
(a) (–5, –4) (b) (–5, 4)
x 2  4x  3  x 2  9  4x 2  14x  6 is
(c) (0, 4) (d) (0, 5)
(ii) logx 0.5 > 2 (a) one (b) two
(c) three (d) none of these
(a)  
½ ,1 (b)   , 1 66. The largest interval among the following for which
(c) (–1, 0) (d) (–1, 1) x12 – x9 + x4 – x + 1 > 0 is
(iii) logx (x + 7) < 0 (a) – 4 < x < 0 (b) 0 < x < 1

(a) (0, 1) b
(b) , 1 g 67.
(c) – 100 < x < 100
2
(d) – < x < 
If f (x) = x – 3x + 1 and f (2) = 2f (), then  is equal to
(c) (–1, 0) (d) (–1, 1)
1 1
61. Let f (x)  l og x 2 25 and g(x) = logx 5 then f (x) = g(x) (a) (b) 
2 2
holds for x belonging to
(a) R (b) (0, 1)  (1, +) 1 1
(c) or  (d) none of these
(c)  (d) None of these 2 2

 3  68. The graph of a real-valued function f (x) is the following.


1 – x 
62. Solve for x : 3
x 2
–2    1  2 
The function is
 
3
Y


(a) – 2, –1  
(b) – 2, 2  y=2x


(c) – 2, – 2  (d) None of these y=0 O X
(a) f (x) = x – |x| (b) f (x) = x + |x|
63. The domain of the function f (x) = log2 (log3 (log4 x)) is
(c) f (x) = 2x (d) None of these
(a) (–, 4) (b) (4, ) 69. The number of real solutions of the equation ex = x is
(c) (0, 4) (d) (1, ) (a) 1 (b) 2
64. The value of x, log½ x > log1/3 x is (c) 0 (d) none of these
(a) (0, 1] (b) (0, 1) 70. The number of real solutions of the equation log0.5 x = |x| is
(c) [0, 1) (d) none (a) 1 (b) 2
(c) 0 (d) none of these
SETS, RELATIONS & FUNCTIONS 35

EXERCISE - 2 : PREVIOUS YEAR JEE MAINS QUESTIONS


3
1. A function f from the set of natural numbers to integers 6. For real x, let f(x) = x + 5x + 1, then (2009)
defined by (a) f is one-one but not onto R
(b) f is onto R but not one-one
 n 1
 2 , when n is odd (c) f is one-one and onto R
f n   is
  n , when n is even (2003) (d) f is neither one-one nor onto R
 2
1
7. The domain of the function f (x)  is (2011)
(a) onto but no one-one | x | x
(b) one-one and onto both
(a) (0, ) (b) (–, 0)
(c) neither one-one nor onto
(c) (–, ) – {0} (d) (–, )
(d) one-one but not onto 2 2
8. Let for a  a1  0, f(x) = ax + bx + c, g(x) = a1x + b1x+c1 and
2. Domain of definition of the function is p(x) = f(x) – g(x). If p (x) = 0 only for x = –1 and
p(–2) =2, then the value of p (2) is (2011)
3
f x  
4  x2

 log10 x 3  x ,  (2003) (a) 18 (b) 3
(c) 9 (d) 6
(a)  1, 0   1, 2  9. If X = {4n – 3n – 1 : n N} and Y = {9 (n–1) : n N}, where
N is the set of natural numbers, then X Y is equal to :
(b) 1, 2    2,  
(2014)
(c)  1, 0   1, 2    2,   (a) Y (b) N
(c) Y – X (d) X
(d) (1, 2)
10. Let P be the relation defined on the set of all real numbers
3. If f : R  R satisfies f (x + y) = f (x) + f (y), for all x,
such that
n
y  R and f (1) = 7, then  f  r  is (2003) P = {(a, b): sec2 a - tan2 b = 1}. Then P is:
r 1
(2014/Online Set–1)
7  n  1 (a) reflexive and symmetric but not transitive.
(a) (b) 7n (n + 1)
2 (b) reflexive and transitive but not symmetric.
(c) symmetric and transtive but not reflexive.
7n  n  1 7n
(c) (d) (d) an equivalence relation.
2 2
Therefore, P is transitive
4. The graph of the function y = f (x) is symmetrical about
the line x = 2, then (2004) 11. Let f be a function defined on the set of real number such
(a) f (x) = f (– x) (b) f (2 + x) = f (2 – x) 11
taht for x > 0, f (x) = 3 sin x + 4 cos x. Then (x) at x  
(c) f (x + 2) = f (x – 2) (d) f (x) = – f (– x) 6
5. If A, B and C are three sets such that A B = A C and is equal to : (2014/Online Set–2)
A B = A C, then (2009)
3 3
(a) A = C (b) B = C (a) 2 3 (b)   2 3
2 2
(c) A B =  (d) A = B
3 3
(c) 2 3 (d)   2 3
2 2
36 SETS, RELATIONS & FUNCTIONS

12. A relation on the set A = {x : |x| < 3,  xZ}, where Z is the


17. Let P = {  : sin   cos   2 cos } and
set of integers is defined by R = {(x, y) : y = |x|, x  -1}.
Then the number of elements in the power se of R is: Q  {  : sin   cos   2 sin } be two sets. Then:
(2014/Online Set–3) (2016/Online Set–2)
(a) 32 (b) 16
(a) P  Q and Q  P  
(c) 8 (d) 64
(b) Q  P
x 1
13. Let f : R  R be defined by f (x)  x  1 then f is: (c) P  Q
(d) P = Q
(2014/Online Set–4) 18. Let a, b, c  R. If f(x) = ax2 + bx + c is such that a + b + c =
(a) both one - one and onto 10

(b) one - one but not onto 3 and f(x + y) = f(x) + f(y)+ xy,  x, y R, then  f (n) is
n 1
(c) onto but not one - one
equal to: (2017)
(d) neither one - one nor onot.
(a) 330 (b) 165
14. Let A and B be two sets containing four and two elements
(c) 190 (d) 255
respectively. Then the number of subsets of the set
A × B, each having at least three elements is: (2015) 19. Let S  {x  R : x  0 and 2 x 3  x
(a) 275 (b) 510
(c) 219 (d) 256  
x  6  6  0} Then S : (2018)
15. In a certain town, 25% of the families own a phone and
15% own a car; 65% families own neither a phone nor a (a) Contain exactly four element
car and 2,000 families own both a car and a phone. (b) is an empty set.
Consider the following three statements : (c) contain exactly one element
(a) 5% families own both a car and a phone (d) contains exactly two elements.
(b) 35% families own either a car or a phone 20. Two sets A and B are as under :
(c) 40,000 families live in the town
A   a, b   R  R : a  5  1 and b  5  1
Then, (2015/Online Set–1)
(a) Only (b) and (c) are correct
(b) Only (a) and (c) are correct
 2 2

B   a, b   R  R : 4  a  6   9  b  5   36 .

(c) All (a), (b) and (c) are correct Then : (2018)
(d) Only (a) and (b) are correct (a) neither A  B nor B  A
16. If f : R  S, defined by (b) B  A

f (x)  sin x  3 cos x + 1, is onto, then the interval of S (c) A  B


is (2015/Online Set–2)
(d) A  B    an empty set 
(a) [0, 1] (b) [-1, 1]
(c) [0, 3] (d) [-1, 3]
SETS, RELATIONS & FUNCTIONS 37

21. Consider the following two binary relations on the set A 22. Let N denote the set of all natural numbers. Define two
={a, b, c} : binary relations on N as
R1={(c, a), (b, b), (a, c), (c, c), (b, c), (a, a)} and R1={( x, y)  N × N : 2 x + y =10} and
R2={(a, b), (b, a), (c, c), (c, a), (a, a), (b, b), (a, c)}. Then : R2={( x, y) N N : x + 2 y =10}. Then :
(2018/Online Set–1) (2018/Online Set–3)
(a) both R1 and R2 are not symmetric. (a) Range of R1 is {2, 4, 8}.
(b) R1 is not symmetric but it is transitive. (b) Range of R2 is {1, 2, 3, 4}.
(c) R2 is symmetric but it is not transitive. (c) Both R1 and R2 are symmetric relations.
(d) both R1 and R2 are transitive. (d) Both R1 and R2 are transitive relations.
23. Let A, B and C be three events, which are pair-wise
independent and E denotes the complement of an event

E. If P  A  B  C  = 0 and P  C  > 0, then P   A  B |C 


is equal to : (2018/Online Set–3)

(a) P  A  - P  B  (b) P  A  + P  B 

(c) P  A  - P  B  (d) P  A  + P  B 
38 SETS, RELATIONS & FUNCTIONS

EXERCISE - 3 : ADVANCED OBJECTIVE QUESTIONS


1. Find the set of values of 'x' for which the given conditions 7. The least integer satisfying,
are true :
 27  x   27  9x 
(a) – (x – 1) (x – 3) (x + 5) < 0 49.4 – < 47.4 –   is
10  10 
 x  1  x  2   0 (a) 2 (b) 3
(b)
 x  3 (c) 4 (d) none of these

2. If |x – 1| + |x| + |x + 1| > 6 ; then x lies in 8. Solution of |3x – 2| > 1 is


(a) [1/3, 1] (b) (1/3, 1)
(a)  , 2 (b)  ,  2   2,  
(c) R (d) 
(c) {1/3, 1} 
(d) , 1 3   1,  

9. Solution of |x – 1| > |x – 3| is
3
3. Solution set of, log3 (x2 – 2) < log3  | x |  1 is
2  (a) x < 2 (b) x > 2
(c) [1, 3] (d) none of these


(a)  2,  1  
(b) 2,  2  10. Solution of |1/x – 2| < 4 is


(c)  2, 2  (d) none of these 
(a) ,  1 2  
(b) 1 6 ,  

4. Solution set of the inequality :


1

1
is

(c)  1 2 , 1 6  
(d) ,  1 2  1 6 ,   
2  1 1  2 x 1
x

1
11. Solution of x  < 4 is
(a) 1,   
(b) 0, log 2  3 
4 x

    
(a) 2  3, 2  3  2  3,  2  3 
(c)  1,   
(d) 0, log 2 4 3  1,  


(b) R  2  3, 2  3 
5. Solution of the inequality x > 1  x  is given by


(c) R  2  3,  2  3 
 
(a) , 1  5 / 2   (d) none of these
12. The solution set of x2 + 2 < 3x < 2x2 – 5 is
(b)  
5  1 / 2,   (a)  (b) [1, 2]
(c) (– , – 1)  [5/2, ) (d) none
 
(c) , 1  5 / 2     
5  1 / 2,  
x 2  3x  4
13. The solution set of  1, x  R is
x 1
(d)  
5  1 / 2, 1

(a)  3,   (b)  1, 1   3,  
6. If x2 – 1 < 0 and x2 – x – 2 > 0, then x lies in the interval set
(a) (1, –1) (b) (–1, 1) (c)  1, 1  3,   (d) none
(c) (1, 2) (d) {–1}
SETS, RELATIONS & FUNCTIONS 39

x2 1 1 | x | x  1
14. The number of integral solutions of  is 21. Let f (x)  
x2 1 2  [x] x  1

(a) 4 (b) 5 where [.] denotes the greatest integer function.


(c) 3 (d) none of these Then f ( f (–2.3)) is equal to
(a) 4 (b) 2
2
1 x  2x  4 9.3  6.3  4 2x x (c) –3 (d) 3
15. If for x  R,   3, then
3 x 2  2x  4 9.32x  6.3x  4 22. The domain of the function y = log10 log10 log10 ... log10x is

lies b/w  ....... n times ....... 


(a) 1 and 2 (b) 1/3 and 3 (a) [10n, +) (b) (10n–1, +)
(c) 0 and 4 (d) none of these (c) [10n–2, +) (d) None of these

16. The solution set of log2 |4 – 5x| > 2 is 23. The largest set of real values of x for which

1

(a) 8 5 ,   
(b) 4 , 8
5 5  f (x)  ( x  2 )( 5  x ) 
2
x 4
is a real function is

(a) [1, 2)  (2, 5] (b) (2, 5]


(c)  , 0   8 , 
5   (d) none
(c) [3, 4] (d) None of these

17. Solution of 2x + 2|x| > 2 2 is 24. The domain of the function f (x)  x  1  x 2 is


(a) , log 2  2 1  
(a)  1, 
1   1 
 , 1 (b) [–1, 1]
 2   2 
(b)  0,  
 1  1 
(c)   ,     , 
 2   2 
1 
(c)  , log 2
2
 
2 1 

 1 
(d)  , 1
 2 

(d) , log 2  
2  1   ½,  
25. The domain of the function f (x) = log10 log10 (1 + x3) is
18. The largest interval among the following for which (a) (–1, ) (b) (0, )
x12 – x9 + x4 – x + 1 > 0 is (c) [0, ) (d) (–1, 0)
(a) – 4 < x < 0 (b) 0 < x < 1
26. The domain of the function f (x)  x 2 –[x]2 , where
(c) – 100 < x < 100 (d) – < x <  [x] = the greatest integer less than or equal to x is
19. Solution of |x – 1| + |x – 2| + |x – 3| > 6 is (a) R (b) [0, +)
(c) (–, 0] (d) None of these
(a) [0, 4] (b)  ,  2    4,  
27. The domain of f (x)  l og x 2 1 (x) is
(c)  , 0   4,   (d) none
(a) ( 2,  ) (b) (0, )
20. If f (x) = cos []x + cos [x], where [y] is the greatest integer
function of y then f (/2) is equal to (c) (1, +) (d) None of these
28. The domain of the real-valued function f (x) = loge |loge x | is
(a) cos 3 (b) 0
(a) (1, +) (b) (0, +)
(c) cos 4 (d) none of these
(c) (e, +) (d) None of these
40 SETS, RELATIONS & FUNCTIONS

29. If [.] denotes the greatest integer function then the domain
 x2 , 0 < x < 2

of the real-valued function log x 1 2 x 2  x  2 is 35. Let f (x) =  2x – 3, 2  x < 3 . Then
 x  2 , x  3

3  3 
(a)  ,    (b)  , 2   (2,  )   3  3
2  2  (a) f  f  f      f  
   2   2
1 
(c)  , 2   (2,  ) (d) None of these  
2  (b) 1 f  f   5   5 
f   f  
   2   2 
30. Let f (x)  l og x 2 25 and g(x) = logx 5 then f (x) = g(x)
(c) f ( f (1)) = f (1) = 1
holds for x belonging to
(d) None of these
(a) R (b) (0, 1)  (1, +) 36.
2 2 2
If a + b + c = 1, then ab + bc + ca lies in the interval
(c)  (d) None of these
 1   1
31. Let f : {x, y, z}  {a, b, c} be a one-one function and only (a)   , 1 (b)  0, 
 2   2
one of the conditions (i) f (x)  b, (ii) f (y), = b
(iii) f (z)  a is true then the function f is given by the set (c) [0, 1] (d) [1, 2]
2
(a) {(x, a), (y, b), (z, c)} (b) {(x, a), (y, c), (z, b)} 37. log2 (x – 3x + 18) < 4, then x belongs to
(a) (1, 2) (b) (2, 16)
(c) {(x, b), (y, a), (z, c)} (d) {(x, c), (y, b), (z, a)}
(c) (1, 16) (d) none of these
32. The function f : R  R defined by f (x) = 6x + 6|x| is
38. If x = loga (bc), y = logb (ca) and z = logc (ab) then which of
(a) one-one and onto (b) many-one and onto the following is equal to 1?
(c) one-one and into (d) many-one and into (a) x + y + z (b) (1+ x)–1 + (1+ y)–1 + (1+ z)–1
33. A function whose graph is symmetrical about the y-axis is (c) xyz (d) none of these
given by
 x   x   x  31
39. If 0 < x < 1000 and          x , where [x] is

(a) f (x)  l og e x  x 2  1   2   3   5  30
the greatest integer less than or equal to x, the number of
(b) f (x + y) = f(x) + f (y) for all x, y  R possible values of x is

(c) f (x) = cos x + sin x (a) 34 (b) 32


(c) 33 (d) none of these
(d) None of these
40. For a real number x, [x] denotes the integral part of x. The
34. The graph of a real-valued function f (x) is the following.
value of
The function is
1 1 1  1 2   1 99 
 2    2  100    2  100   ....   2  100  is
Y
(a) 49 (b) 50
y=2x (c) 48 (d) 51

2 f (n) 1
y=0 O X 41. If f (n + 1)  , n = 1, 2, ... and f (1) = 2, then
2
f (101) equals
(a) f (x) = x – |x| (b) f (x) = x + |x| (a) 52 (b) 49
(c) f (x) = 2x (d) None of these (c) 48 (d) 51
SETS, RELATIONS & FUNCTIONS 41

1 1
42. The domain of function f (x)  , where {x} 47. If < log0.1 x < 2 then
2
x 2  { x }2

denotes fraction part of x. 1


(a) the maximum value of x is
10
1  1
(a) R – [0, 1) (b) R   ,1 1
2  (b) x lies between and
100 10

1 1 1
(c) x does not lie between and
(c) (, ]  (1, ) (d) none of these 100 10
2
1
43. If graph of y = f (x) is (d) the minimum value of x is
100
48. If f is an even function defined on the interval (–5, 5) then
y=x  x 1 
a value of x satisfying the equation f (x) = f   is
 x2 
y = f (x)
–1  5 –3  5
(a) (b)
2 2

–1 – 5 –3 – 5
(c) (d)
2 2
Then f (x) can be 49. Let f (x) = [x] = the greatest integer less than or equal to x
(a) y = 2 e x
(b) y = 4 e x and g(x) = x – [x]. Then for any two real numbers x and y.
(a) f (x + y) = f (x) + f (y)
1
x 1 x (b) g(x + y) = g(x) + g(y)
(c) y  e 2
(d) y  e
4 (c) f (x + y) = f (x) + f (y + g(x))
44. The equation | |x – 1| + a | = 4 can have real solutions for x (d) none of these
if ‘a’ belongs to the interval 50. Let x  N and let x be a perfect square. Let f (x) = the quotient
(a) (–, 4] (b) (–, –4] when x is divided by 5 and g(x) = the remainder when x is
(c) (4, + ) (d) [–4, 4] divided by 5. Then x = f (x) + g(x) holds for x equal to
45. If logk x. log5 k = logx 5, k  1, k > 0, then x is equal to (a) 0 (b) 16
(c) 25 (d) None of these
1 2
(a) k (b) 51. Let f (x) = [x] + [x + 1] – 3, where [x] = the greatest integer
5 < x. Then
(c) 5 (d) None of these (a) f (x) is a many- one and into function
(b) f (x) = 0 for inifinite number of values of x
46. If x 4 f (x)  1  sin 2x  | f (x) | 2f (x), then f(-2) equals:
(c) f (x) = 0 for only two real values

1 1 (d) none of these


(a) (b)
17 11 52. Which of the following functions is not injective ?
(a) f (x) = |x + 1|, x  [–1, 0] (b) f (x) = x + 1/x, x  (0, )
1 (c) f (x) = x2 + 4x – 5
(c) (d) 0
19
(d) f (x) = e–x, x  [0, )
42 SETS, RELATIONS & FUNCTIONS

56. Assertion : The range of the function


3 – 2x
53. Let f : R  R be defined by f (x) = [x] and g(x) = , [.] 2
4 f (x) = sin x + p sin x + q, where |p| > 2,

represents greatest integer function, then


p2
will be real numbers between q –
(a) f is neither one-one nor onto 4

(b) g is one-one but f is not one-one


and q + p + 1.
(c) f is one-one and g is onto
Reason : The function g(t) = t2 + pt + 1, where
(d) neither f nor g is onto t  [–1, 1] and |p| > 2, will attain
Assertion Reason minimum and maximum values at
(A) If both ASSERTION and REASON are true and –1 and 1.
reason is the correct explanation of the assertion.
(a) A (b) B
(B) If both ASSERTION and REASON are true but reason
(c) C (d) D
is not the correct explanation of the assertion.
(e) E
(C) If ASSERTION is true but REASON is false.

(D) If both ASSERTION and REASON are false. 57. Assertion : Let A and B be two sets each with a
finite number of elements. Assume
(E) If ASSERTION is false but REASON is true.
that there is an injective mapping from
ax + b A to B and that there is an injective
54. Assertion : The function , (ad – bc0)
cx + d
mapping from B to A. Then there is a
bijective mapping from A to B.
a
cannot attain the value .
c Reason : An onto function is not necessarily
one-one.
Reason : The domain of the function
(a) A (b) B
b – dy
g(y) = is all the reals except a/c.
cy – a (c) C (d) D

(e) E
(a) A (b) B
58. Assertion : The domain of a function y = f (x) will
(c) C (d) D
be all reals if for every real x there
(e) E
exist y.
55. Assertion : The domain of the function
sin–1 x + cos–1 x + tan–1 x is [–1, 1] Reason : The range of a function y = f (x) will
be all reals if for every real y there exists
Reason : sin–1 x, cos–1 x are defined for |x|  1
and tan–1 x is defined for all x. a real x such that f (x) = y.

(a) A (b) B (a) A (b) B

(c) C (d) D (c) C (d) D


(e) E (e) E
SETS, RELATIONS & FUNCTIONS 43

Using the following passage, solve Q.59 to Q.61 63. The number of solutions of equation f (x) = y2 where y is
a positive integer and x < 5 is
Passage –1
(a) 5 (b) 2
A rational function is defined as quotient of two
(c) 3 (d) None of these
polynomials, p(x) and q(x). The domain of the rational
function must be all reals except the roots of the equation 64. The number of solutions of f (x) = y2 where x > 5 must be
q(x) = 0. The range of rational function can be found by (a) 1 (b) 2
finding minimum and maximum values of the function. In (c) 3 (d) None of these
case p(x) and q(x) have a common factor x – . Then after Match the column
cancelling the common factor, the rational function must
assume a value at x =  which should be deleted from the 65. Column–I Column–II
found range since  is not there in the domain of the (A) f (x + y) = f (x) + f (y) (P) log3 x
rational function. (B) f (xy) = f (x) + f (y) (Q) tan–1 x
3x + 1 (C) f (x + y) = f (x) . f (y) (R) 3x
59. The range of the rational function f (x) = must be
2x + 1
x+y
(D) f (x) + f (y) = f   (S) 3x
 1 – xy 
1 1
(a) R – 
–  (b) R –  – 
 2  3 66. Column–I Column–II
(A) odd function (P) x – [x]
3
(c) R –   (d) R
2
(B) even function 
(Q) log x  1 x


 2x + 1 1 x
60. The range of the rational function f (x) = (C) neither even nor odd (R) x log
2x 2 + 5x + 2
1– x
must be
(a) R – {0} (b) R – {–2} 2x / 2
(S)
1 2x / 2
 2
(c) R – {0, –2} (d) R – 0, 
 3 Subjective
67. When 0 < x < 2 and [x] denotes greatest integer < x, then
2x 2 + 5x + 2 [sin x] + [cos x] + [sin x + cos x] takes exactly k integer
61. The range of the rational function f (x) =
2x + 1 values. Then k must be
must be
68. The range of the function x – 6  12 – x is an interval
(a) R – {0} (b) R – {–2}
of length 2 3 – k then k must be
 2 3
(c) R – 0, – 2,  (d) R –  
 3 2
 1
69. If f  x   = x3 + x–3 then f (5) must be equal to
Using the following passage, solve Q.62 to Q.64  x
Passage –2 70. The least period of the function
Let f (x) be a function defined by

f(x) = 1! + 2! + 3! + 4! .... + x!, where x is a positive integer. cos (cos x) + sin (cos x ) + sin 4x is k

Answer the following questions :
62. The last digit of f (2007) will be then value of k must be

(a) 3 (b) 7
(c) 5 (d) 1
44 SETS, RELATIONS & FUNCTIONS

EXERCISE - 4 : PREVIOUS YEAR JEE ADVANCED QUESTIONS


Single Answer Type Questions Multiple Type Question
2 2
1. Indicate the correct alternative : The number log27 is 8. If f (x) = cos [ ] x + cos [– ] x, where [x] stands for the
(1990) greatest integer function, then (1990)

(a) an integer (b) a rational number 


(a) f    1 (b) f () = 1
(c) an irrational number (d) a prime number 2
2. The domain of definition of the function y(x) is given by
x y
the equation 2 + 2 = 2 is (2000) 
(c) f (–) = 0 (d) f    1
(a) 0  x  1 (b) 0  x  1 4
Match the Columns
(c)    x  0 (d)   x  1
Match the conditions/expressions in Column I with
3. Number of solutions of log4 (x – 1) = log2 (x – 3) is
statement in Column II.
(2001)
(a) 3 (b) 1 x 2  6x  5
9. Let f (x) 
x 2  5x  6
(c) 2 (d) 0
Column I Column II
x
4. Let f  x   , x   1 . Then, for what value of  is (A) If –1 < x < 1, then f (x) (p) 0 < f (x) < 1
x 1
f ( f (x)) = x ? (2002) satisfies
(B) If 1 < x < 2, then f (x) (q) f (x) < 0
(a) 2 (b)  2
satisfies
(c) 1 (d) –1
(C) If 3 < x < 5, then f(x) (r) f (x) > 0
x satisfies
5. If f : [0, ) [0, ) and f (x)  , then f is (2003)
1 x (D) If x > 5, then f(x) (s) f (x) < 1
(a) one-one and onto satisfies (2007)
(b) one-one but not onto Integer Answer Type Questions
(c) onto but not one-one 10. Find the values of x satisfying the equation (1990)
A 7 2
(d) neither one-one nor onto |x – 1| = (x – 1) where A = log3x – 2 logx 9.
2 11. Find all real numbers x which satisfy the equation,
6. Range of the function f (x)  x  x  2 ; x R is (2003) (1999)
x2  x  1
(a) (1, ) (b) (1, 11/7) 
2 log2 log2 x + log1/2 log2 2 2 x = 1. 
(c) (1, 7/3] (d) (1, 7/5) 2 2 –1
12. log3/4 log8 (x + 7) + log1/2 log1/4 (x + 7) = –2.
7. The function f : [0, 3]  [1, 29], defined by
(2000)
f (x) = 2x3 – 15x2 + 36x + 1, is (2012)
(a) one-one and onto
(b) onto but not one-one
(c) one-one but not onto
(d) neither one-one nor onto
SETS, RELATIONS & FUNCTIONS 45

ANSWER KEY
EXERCISE - 1 : BASIC OBJECTIVE QUESTIONS
1. (b) 2. (c) 3. (c) 4. (c) 5. (d) 6. (b) 7. (c) 8. (a,c) 9. (c) 10. (d)

11. (a) 12. (b,d) 13. (b) 14. (a) 15. (a) 16. (a) 17. (c) 18. (b) 19. (a) 20. (a)

21. (d) 22. (c) 23. (a) 24. (d) 25. (a) 26. (b) 27. (i) (a); (ii) (d); (iii) (a); (iv) (c) 28. (b)

29. (c) 30. (d) 31. (d) 32. (b) 33. (d) 34. (c) 35. (c) 36. (d) 37. (b) 38. (b)

39. (a) 40. (a) 41. (d) 42. (a) 43. (b) 44. (a) 45. (c) 46. (a) 47. (b) 48. (b)

49. (d) 50. (a) 51. (c) 52. (c) 53. (c) 54. (i) (c) (ii) (a) (iii) (d) 55. (a) 56. (a)

57. (b) 58. (d) 59. (d) 60. (i) (a) (ii) (a) (iii) (a) 61. (b) 62. (d) 63. (b) 64. (a)

65. (a) 66. (d) 67. (c) 68. (b) 69. (c) 70. (a)

EXERCISE - 2 : PREVIOUS YEAR JEE MAINS QUESTIONS


1. (b) 2. (c) 3. (c) 4. (b) 5. (b) 6. (c) 7. (b) 8. (a) 9. (a) 10. (d)
11. (c) 12. (b) 13. (d) 14. (c) 15. (c) 16. (d) 17. (d) 18. (a) 19. (d) 20. (c)
21. (c) 22. (b) 23. (a)

EXERCISE - 3 : ADVANCED OBJECTIVE QUESTIONS

1. (a) (–5, 1)  (3, ) ; (b) (– , 1]  [2, 3) 2. (b) 3. (d) 4. (d) 5. (d) 6. (d) 7. (b)

8. (d) 9. (b) 10. (d) 11. (a) 12. (a) 13. (b) 14. (c) 15. (b) 16. (c) 17. (d)

18. (d) 19. (c) 20. (c) 21. (d) 22. (d) 23. (b) 24. (d) 25. (b) 26. (d) 27. (a)
28. (d) 29. (b) 30. (b) 31. (c) 32. (d) 33. (d) 34. (b) 35. (a,b,c) 36. (a) 37. (a)

38. (b) 39. (c) 40. (b) 41. (a) 42. (d) 43. (d) 44. (a) 45. (b,c) 46. (a) 47. (a,b,d)
48. (a,b,c,d) 49. (c) 50. (b,c) 51. (a,b) 52. (b,c) 53. (a,b) 54. (a) 55. (a) 56. (d) 57. (b)

58. (b) 59. (c) 60. (d) 61. (d) 62. (a) 63. (b) 64. (d)

65. (A)  (R), (B)  (P), (C)  (S), (D)  (Q) 66. (A)  (Q), (B)  (R, S), (C)  (P)

67. (0005) 68. (0006) 69. (0110) 70. (0004)

EXERCISE - 4 : PREVIOUS YEAR JEE ADVANCED QUESTIONS


1. (c) 2. (d) 3. (b) 4. (d) 5. (b) 6. (c) 7. (b) 8. (a,c)

9. (A–p; B–q; C–q; D–p) 10. x = 2 or 81 11. x = 8 12. x = 3 or –3

Dream on !!

02
LIMITS
LIMITS 47

LIMITS

LIMITS 3. EVALUATION OF ALGEBRAIC LIMITS

1. INTRODUCTION Let f (x) be an algebraic function and ‘a’ be a real number.


Then lim f (x) is known as an algebraic limit.
Calculus is the mathematics of motion and change, while x a
algebra, geometry, and trigonometry are more static in nature.
th
The development of calculus in the 17 century by Newton, x 3 1 1 x  1 x x 5  32
E.g. lim , lim , lim etc.
Leibnitz and others grew out of attempts by these and earlier x 1 x 1 x 0 x x 2 x  2

mathematicians to answer certain fundamental questions are algebraic limits.


about dynamic real-world situations. These investigations
led to two fundamental procedures- differentiation and 3.1 Direct substitution method
integration; which can be formulated in terms of a concept
called- limit. If by direct substitution of the point in the given expression
we get a finite number, then the number obtained is the limit
In a very real sense, the concept of limit is the threshold to
of the given expression.
modern mathematics. You are about to cross that threshold,
and beyond lies the fascinating world of calculus. If upon substituting the point in the given expression, we
get the following forms. :
2. LIMIT OF A FUNCTION (INFORMAL DEFINITION) 0 
, ,  – , 0 × , °, 0°, 1
The notation : 0 
(Indeterminate Forms)
lim f  x   L
x c Then we can’t find the value of limit by direct substitution.
is read “the limit of f (x) as x approaches c is L” and means Following methods are followed to find the limit of the
that the functional values f (x) can be made arbitrarily close function.
to a unique number L by choosing x sufficiently close to c
3.2 Factorisation method
(but not equal to c).

2.1 One-Sided Limits f x


Consider lim . If by putting x = a the rational function
x a g  x
2.1.1 Right-hand Limit : We write lim f  x   L if we

x c
f x 0 
can make the number f (x) as close to L as we please takes the form , etc, then (x–a) is a factor of
by choosing x sufficiently close to c on a small interval g x 0 
(c, b) immediately to the right of c. both f (x) & g(x). In such a case we factorise the numerator
2.1.2 Left-hand limit : We write lim f  x   L if we can and denominator, and then cancel out the common factor
x c (x–a). After cancelling out the common factor (x–a), we again
make the number f (x) as close to L as we please by put x = a in the given expression and see whether we get a
choosing x sufficiently close to c on a small interval meaningful number or not. This process is repeated till we
(a, c) immediately to the left of c. get a meaningful number.
2.1.3 Limit of a function f (x) is said to exist as x  a
(x approaches a) when ; 3.3 Rationalisation method
lim f  x   lim f  x    (finite quantity)
x a x a This is particularly used when either numerator or
denominator, or both involve expressions consisting of
 lim f  x   
xa square roots (radical signs)
48 LIMIT, CONTINUITY & DIFFERENTIABILITY

3.4 Method of evaluating algebraic limits when x  sin  x  a 


7. lim 1
To evaluate this type of limits we follow the following x a  x  a 

procedure.
Step–1 : Write down the given expression in the form of a tan  x  a 
8. lim 1
x a  x  a 
f x
rational function, i.e., , if it is not so.
g x 5. EXPONENTIAL & LOGARITHMIC LIMITS
Step–2 : If k is the highest power of x in numerator and To evaluate the exponential and logarithmic limits we use
denominator both, then divide each term in numerator the following results.
and denominator by xk.
a x 1
1 1. lim  log e a
Step–3 : Use the result lim n  0 , where n > 0. x 0 x
x  x

e x 1
3.4.1 Important Results : If m, n are positive integers and 2. lim 1
x 0 x
a 0 , b 0  0 are non-zero numbers, then

a 0 x m  a1x m 1  ....  a m1 x  a m log 1  x 


lim 3. lim 1
x  b0 x n  b1x n 1  ....  b n 1 x  b n
x 0 x

 xn  an  n –1
4. lim    n .a
 a0
x a
 x a 
b , if mn
 0
  0, if mn 6. EXPONENTIAL LIMITS OF THE FORM 
 , if m  n ,  a 0  b0   0
 To evaluate the exponential limits of the form 1 we use the
   , if m  n ,  a 0  b0   0 following results.

1. If lim f  x   lim g  x   0 , then


x a x a
4. TRIGONOMETRIC LIMITS
f x
To evaluate trigonometric limits the following results are lim
1 / g x  x a gx 
lim 1  f  x   e ,
very useful. x a

sin x 2. If lim f (x) = 1 and lim g(x) = , then


1. lim 1 x a x a
x 0 x
gx g x  lim  f  x 1 g  x 
tan x lim  f  x    lim 1 f  x  1  e xa
2. lim 1 x a x a
x0 x

(in 1 & 2, x is measured in radians) 1.1 Particular Cases

sin 1 x 1. lim 1 x 


1/ x
e
3. lim 1 x 0
x0 x
x
 1
tan 1 x 2. lim 1   e
4. lim 1 x   x
x 0 x
1/ x
3. lim 1 x   e
sin x 0  x 0
5. lim 
x 0 x 180
x
 
6. lim cos x  1 4. lim  1   e
x 0 x   x
LIMITS 49

7. SQUEEZE RULE/SANDWICH RULE In particular

If g (x) < f (x) < h (x) on an open interval containing ‘c’, and  
(a) lim log f  x   log  lim f  x    log  ; (provided l > 0)
if : x a  x a 

lim g  x   lim h  x   L , then lim f  x   L lim f  x 


x c x c x c (b) lim e f  x   e x  a  e .
x a

g(x)
1
f(x) 7. If lim f  x     or – , then lim  0.
L x a x a f x
h(x)
9. LIMITS BY DE’L’ HOSPITAL’S RULE
x=c
If f (x) and g(x) be two functions of x such that
In words : If a function can be squeezed/sandwiched between
two functions whose limits at a particular point c have the 1. lim f  x   lim g  x   0
x a x a
same value L, then that function must also have limit
L at x = c. 2. both are continuous at x = a,

8. THE ALGEBRA OF LIMITS 3. both are differentiable at x = a,

Let f and g be two real functions with domain D. We define 4. f (x) and g(x) are continuous at the point x = a, then
four new functions f ± g, f g, f /g on domain D by setting
f x f ' x
( f ± g) (x) = f (x) ± g (x), ( f g) (x) = f (x) × g (x) lim  lim , provided that g (a)  0.
x a g  x  x a g'  x 
( f /g)(x) = f (x)/g(x), if g(x)  0 for any x  D.
Following are some results concerning the limits of these
The above rule is also applicale,
functions.
if lim f  x    and lim g  x    .
Let lim f  x    and lim g  x   m , if  and m exist. x a x a
x a xa

1. lim  f  g  x   lim f  x   lim g  x   l  m f ' x


x a x a x a Generalisation : If lim assumes the indeterminate
x a g'  x 
2. lim  f g  x   lim f  x   lim g  x   m
x a x a x a
form (0/0) or ()and f (x), g(x) satisfy all the conditions

lim f  x   embodied in De’L’Hospitals rule, we can repeat the


 f 
3. lim    x   x a  , provided m  0.
x a  g  lim g  x  m f ' x
x a application of this rule on to get
g'  x 
4. lim k f  x   k . lim f  x  , where k is constant.
x a x a
f x f x  f   x 
g x lim  lim  lim  ... .
5. lim  f  x   m
  ; ( provided lim f  x   0 ) x a g  x  x a g   x  x a g   x 
x a x a

Sometimes it may be necessary to repeat this process a


 
6. lim fog  x   f  lim g  x    f  m  ; provided f (x)
x a  x a  numer of times till our goal of evaluating limit is achieved.

is continuous at g (x) = m.
50 LIMIT, CONTINUITY & DIFFERENTIABILITY

10. EXPANSIONS TO EVALUATE LIMITS


x 3 x5
5. sin x  x  +  ....
3! 5!
n  n –1 n  n –1 n – 2  3
1. 1+ x n =1+ nx + + x +....
2! 3! x2 x4
6. cos x = 1– + –....
2! 4!
(for rational or integral n, Rule is not applicable for
irrational n.)
x3 2 5
7. tan x  x   x  ....
x 2 x3 3 15
2. e x = 1+ x + + +...
2! 3!
1 x3 1 3 x 5 1 3 5 x 7
8. sin –1x = x+ . + . . + . . +...
x2 2 2 3 2 4 5 2 4 6 7
3. a x  1  x  loge a    loge a   ....
2!
1 1
9. tan –1x = x – x 3 + x 5 –...
3 5
x2 x3 x4
4. log 1  x   x     ...
2 3 4
51 LIMITS

SOLVED EXAMPLES

Example – 1 Example – 3

Evaluate the following limit :


x 3  6x 2  11x  6
Evaluate lim .
x 2 x 2  6x  8
12  2 2  32  .....  n 2
lim .
n  n3
Sol. When x = 2, the expression
(x type problem)
3 2
x  6x  11x  6 0
2
of the form .
x  6x  8 0
12  2 2  .....  n 2 n  n  1  2n  1
Sol. lim  lim
x 3  6x 2 11x  6  0 
n  n3 n  6n 3
Now, lim 2  form 
x 2 x  6x  8 0 
1  1  1 1 1
 lim 1   2    1 0  2  0   .
 x 1 x  2  x  3 0  6 n   n  n 6 3
 lim  form 
x 2  x  2  x  4  0 
Example – 4

 lim
 x 1 x  3   2 1 2  3  1 Evaluate the following limits :
x 2  x  4  2  4 2.
sin  cos x  cos x
Example – 2 lim
x
 sin x – cosec x .
2

a  2x  3x
Evaluate lim .
x a 3a  x  2 x  sin x 
 using lim  1
 x  0 x 
a  2x  3x
Sol. When x = a, assumes the indeterminate
3a  x  2 x sin  cos x  cos x
Sol. xlim

 sin x – cosec x
0 2
form .
0
sin  cos x  sin x cos x
a  2x  3x  lim
Now, lim  sin 2 x  1
x
x a 3a  x  2 x 2

 lim
 a  2x  3x  a  2x  3x  3a  x  2 x  sin  co s x  sin x
 lim
x a
 3a  x  2 x  3a  x  2 x  a  2x  3x  x

2
co s x

a  x  = –1.
 lim
3a  x  2 x 
x a
a  x  a  2x  3x 

 lim
 3a  x  2 x  4 a  2
.
x a
 a  2x  3x  2  3 3a  3 3
52 LIMIT, CONTINUITY & DIFFERENTIABILITY

Example – 5 Example – 7

Evaluate the following limits :


tan 3 x – 3tan x
Solve : lim .
x  / 3  
cos  x  
 6 3x + 33 – x – 12
lim .
x2 33 – x – 3x / 2

tan 3 x – 3 tan x 
Sol. Let L  lim and x –  t x 3– x
x  / 3  
cos  x  
 3 Sol. L  lim 3 + 3 – 12
3– x x/ 2
 6 x 2 3 –3

    27
tan 3  t   – 3 tan  t   3x + – 12
 3  3  L  lim 3x
 L  lim 27
t 0   x2
– 3x / 2
cos  t  
 2 3x

    32x – 12.3x  27
tan  3t    3tan 2  t   – 1  L  lim
 L  lim   3 
x2

– 3x / 2 – 33 
t 0 – sin t
(3x – 9) (3x – 3)
 L  – lim
 tan  3t      x2
3 x/2
–3  3 x
 9  3.3x / 2 
 L  lim .lim 3 tan 2  t   – 1
t 0 – sin t t  0   3 
(3x/2 + 3) (3x – 3)
tan  3t  t      L  lim –
 L  – 3 lim
t 0 3t
 lim
t  0 sin t
 lim 3 tan 2  t   – 1
t0
  3 
x 2

3x  3.3x / 2  9 
 L = –3 × 1 × 1 × 8 = – 24. – 6.6 – 36 – 4
 L   .
Example – 6 9  3.3  9 27 3

Evaluate the following limits : Example – 8

x a
e –e . x
lim a
tan
x a x–a  2a
Evaluate : lim  2 – 
x a  x
 a x –1  [1 type of indeterminate form]
 using lim  l og a 
 x 0 x 

 a x 
Sol. Since, lim  2 –   1 and lim    .
x a  x x  a  2a 

Sol. lim
e x
–e a
 lim
e a
e x– a
–1 
x a x –a x a x –a tan
x
 a 2a
 lim  2 –  of the form (1)
x– a x a  x
a e –1 x– a
e lim lim
x a x – a x a x – a x
tan
 a 2a
Hence, lim  2 – 
a x – a e a
x a  x
e 1 lim 
x a
x – a  x  a  2 a
LIMITS 53

 a  x

 ex a
lim  2 – – 1  tan .
 x  2a
L  – 6 lim

cos3x 2 cos 2x 2 – x 
x 0 4x –1
 x –a   x 
lim    tan  .
 x   2a 
 ex a
x
lim
h   h 
x 0

sin 2x 2 – x 
lim  tan   
e h 0 a  h   2a  (putting, x – a = h)

1.1 1
 L  – 6. lim
– lim
h
h 0 a  h
 h 
 cot  
 2a 
   
 tan      – cot  

– 1 x  0  4x –1 cos 2x 2 – x 
e    

 h / 2a   lim  2a / h  1
– lim  L  – 6.  –6
h  0 tan  h / 2a  h  0  a  h  – 1.1
e

 e–2/. Example – 10


Example – 9
 1 1 
Evaluate : lim  –  using LH rule.
2 x 1 l n x x –1 
sin 3x 
Evaluate : lim using LH rule.
x 0

l n cos 2x 2 – x 
[– type of indeterminate form]

0
( type of indeterminate form)
0  1 1 
Sol. Let L = lim  –  [–  form]
x 1 l n x x –1 

sin 3x 2 0
Sol. Let lim [ form] Let us reduce it to an indeterminate form of the type 0/0.
x 0

l n cos 2x – x 2
 0
x –1– l n x
Apply LH rule to get : L  lim [0/0 form]
x 1  x –1 l n x

L  lim
–6 x cos 3x 2 cos 2x 2 – x   Apply LH rule to get,
x 0
 4x –1 sin  2x 2
–x 
1–1/x
L  lim .
x 1 l n x +1 – 1 / x
 – 6 lim
cos3x 2 cos 2x 2 – x  
x 0 4x – 1
x –1
 L  lim .
x 1 x l n x + x –1
x
lim
x 0

sin 2x 2 – x  Apply LH rule again,

The limit of the factor is computed directly, the limit of the x –1 1


 L  lim . 
second one, which represents an indeterminate form of the x 1 l n x+2 2
0/0 is found with the aid of the L’Hospital’s rule. Again
consider.
54 LIMIT, CONTINUITY & DIFFERENTIABILITY

Example – 11
 lim f (x)
x 0 
Evaluate the left hand and right hand limits of the function
defined by h | h |
 lim f (0  h)  lim
h 0 h 0 h
1  x 2 , if 0  x  1
f (x)   at x = 1.
 2  x, if x  1 hh 0
 lim  lim  lim 0  0
h 0 h h 0 h h 0

Also, show that lim f(x) does not exist.


x 1
Clearly, lim f (x)  lim f (x)
x 0 x 0

Sol. We have, lim f(x) does not exist.


So, x 0
(LHL of f(x) at x = 1)

 lim f (x)  lim f (1  h) Example – 13


x 1 h 0
Discuss the existence of each of the following limits :
2 2
 lim 1  (1  h)  lim 2  2h  h  2.
h 0 h 0 1 1
(i) lim (ii) xlim
and, x0 x 0 |x|

(RHL of f(x) at x = 1)
1
lim f (x)  lim f (1  h) Sol.(i) The graph of f(x) = is as shown in Fig. We observe that
x 1 h 0 x
as x approaches to 0 from the LHS i.e. x is negative and very
 lim 2  (1  h)  lim 1  h  1
h 0 h 0 1
close to zero, then the values of are negative and very
x
Clearly, lim f (x)  lim f (x).
x 1 x 1 large in magnitude.

So, lim f(x) does not exist. 1


x 1  lim 
x 0  x
Example – 12

 x | x |
 , x0
If f (x)   x show that xlim
0
f(x) does not
 2 , x0

exist.

Sol. We have,
(LHL of f(x) at x = 0)

= xlim
0
f (x)

h  |  h |
 lim f (0  h)  lim
h 0 h 0 ( h) Similarly, when x approaches to 0 from the right i.e. x is
1
h  h 2h positive and very close to 0, then the values of are very
 lim  lim  lim 2  2 x
h 0 h h 0  h h 0
large and positive.
(RHL of f(x) at x = 0)
LIMITS 55

Example – 14
1 1 1
 lim   Thus we have, lim  lim
x 0  x x 0 x x 0 x
 mx 2  n, x  0
1 
Hence, lim does not exist. If f (x)   nx  m,0  x  1
x0 x  3
 nx  m, x  1
1
(ii) The graph of f (x) = is shown in Fig.. We observe that
|x| For what values of integers m, n does the limits xlim
0
f(x)
as x approaches to 0 from LHS i.e. x is negative and close to
and lim f (x) exist.
0, then | x | is close to zero and is positive. Consequently, x 1

1
is large and positive. Sol. It is given that
|x|
lim f (x) and lim f (x) both exist
x 0 x 1
1
 lim 
x 0  | x | lim f (x)  lim f (x) and lim f (x)  lim f (x)

x 0 x 0 x 1 x 1
Also, if x approaches to 0 from RHS i.e. x is positive and
close to 0, then | x | is close to zero and is positive.  lim f (0  h)  lim f (0  h) and lim f (1  h)  lim f (1  h)
h 0 h 0 h 0 h 0
1
Consequently, is large and positive.  lim m(  h) 2  n  lim n (h) + m and
|x| h 0 h 0

lim n(1  h)  m  lim n(1  h)3  m


h 0 h 0

 n = m and n + m = n + m
 m=n

Hence, xlim
0
f (x) and lim f(x) both sides for n = m.
x 1

Example – 15

| x | 1 , x  0

If f (x)   0 , x  0 . For what value (s) of a does
| x | 1 , x  0

lim f(x) exist ?


x a

Sol. We have,
1
 lim 
x 0 | x |
 | x | 1 , x  0

f (x)   0 , x0
Thus, we have | x | 1 , x  0

1 1
lim  lim
x 0  | x | x 0 | x |

x  1 , x  0
 
1 f (x)   0 , x0
Hence, xlim exists and it tends to infinity..  x 1 , x  0
0 |x| 
56 LIMIT, CONTINUITY & DIFFERENTIABILITY

Example – 17
  x, x  0 
 | x |   
  x, x  0 Evaluate the left hand and right hand limits of the function

Clearly, xlim f (x) exists for all a  0. 


a
 x 2
– 6x  9 , x 3
f x    x – 3
So, let us see whether xlim
0
f (x) exist or not. 
 0 , x 3
We have,
at x = 3 and hence comment on the existence of limit at
lim f (x)  lim f (0  h)  lim  (  h)  1  1
x 0  x 0 h 0 x = 3.

lim f (x)  lim f (0  h)  lim h  1  1 Sol. The given function can be written as
x 0 x 0 h 0

lim f (x)  lim f (x)  x –3


 x 0 x 0   , x 3
f  x     x – 3
 0 , x 3
lim f(x) does not exist. Hence, lim f(x) exists for all a 0. 
So, x 0 x a

Example – 16  L.H.L. = lim f (x)


x  3–

= lim f (3 – h)
a  bx , x  1 h 0

Suppose f (x)   4 , x 1
 b  ax , x  1 3– h –3
 = lim
h 0  3 – h – 3
and if lim
x 1
f (x) = f (1). What are possible values of a and b ?
–h h
 lim  lim  –1
h 0  – 3 h  0 –h
Sol. We have,
& R.H.L. = lim f (x)
lim f (x)  f (1) x 3
x 1

= lim f (3 + h)
h 0
 lim f (x)  lim f (x)  f (1)
x 1 x 1
3 h – 3
= lim
 lim f (x)  f (1) and lim f (x)  f (1) h 0  3  h – 3
x 1 x 1

h h
 lim f (1  h)  4 and lim f (1  h)  4  lim  lim  –1
h 0 h 0 h 0 h h 0 h
Hence left hand limit and right hand limit of f (x) at x = 3 are
 lim {a  b(1  h)}  4 and lim {b  a(1  h)}  4 –1 and 1 respectively.
h 0 h 0
As left Hand Limit  Right Hand Limit, Limiting value
 a + b = 4 and b – a = 4 at x = 3.

 a = 0, b = 4 i.e. lim f (x) does not exist.


x 3
LIMITS 57

Example – 18
– 2h
 lim  lim 2  2
h 0 – h h 0

x – x
 , x 0
If f  x    x (R.H.L. of f (x) at x = 0)
 2 , x 0

 lim f (0 + h)  lim f (0 + h)

h 0 h 0

show that lim f (x) does not exist.


x 0
h– h h–h
 lim  lim
h 0 h h 0 h
Sol. We have,

(L.H.L. of f (x) at x = 0) 0
 lim  lim 0  0.
h 0 h h 0
 lim f (0 – h)
h 0

Since, lim f (x)  lim f (x).


x  0– x  0

–h– –h –h–h
 lim  lim
h 0 – h h  0 –h So, lim f (x) does not exist.
x 0
LIMITS 58

EXERCISE - 1 : BASIC OBJECTIVE QUESTIONS


Factorization
x2 1 1
7. lim
x3  8 x 0
x2  9  3
1. lim is
x 2 x2  4
(a) 3 (b) 4
3 (c) 1 (d) 2
(a) 3 (b)
2
x a  ba
(c) 1 (d) 0 8. lim
x b x 2  b2
x2 1  x 1 1
2. lim 1
x 1 x2 1 (a) (b)
4b a  b 4b b  a

1 1 1
(a) (b) 2 1
2 (c) (d)
4a a  b b ba
1
(c) 1 (d) 1 
2 1 2  x  3
9. lim
x2 x2
x2  3
3. lim
x 3 x 2  3 3x  12 1 1
(a) (b)
8 3 3
1 2
(a) (b) (c) 8 3 (d)
5 5 3
Exponential & Logarithmic
3 3
(c) (d)
5 5 etan x  e x
10. Limit 
x 0 tan x  x
1  x 2/3
4. lim (a) 1/2 (b) 0
x 1 1  x 1/3
(c) 1 (d) none
(a) 2 (b) 1
3x  1
12 11. lim is equal to
(c) (d) none of these x 0 x 1 1
3
(a) loge 9 (b) loge 3

5. lim
 
2  1  2x  3 
is equal to (c) 0 (d) 1
x 1 2x 2  x  3
41/ n  1
12. lim is equal to
1 1 n  31/ n  1
(a) (b)
10 10 (a) log4 3 (b) 1
(c) 1 (d) None of these (c) log3 4 (d) none of these
Rationalization
3x  2 x
13. lim is equal to
2 x 3 x  0 4 x  3x
6. The value of lim is
x 7 x 2  49 (a) 1 (b) –1
(a) 2/9 (b) -2/49 (c) 0 (d) none of these
(c) 1/56 (d) -1/56
LIMITS 59

 x 3  1 1 1 1
14. The value of l im  l og a  is 22. lim    ...  n equals
x 3  x  6  3 
n  2 2 2 23 2

(a) 2 (b) -1
(a) loga6 (b) loga3
(c) 1 (d) 3
(c) loga2 (d) None of these
5n 1  3n  22n
x 2  ax  b 23. Limit 
e 1 n  5n  2n  32n 3
15. If , are the roots of x2 – ax + b = 0, then lim
x  x  (a) 5 (b) 3
is (c) 1 (d) zero
(a) –  (b) – 
1 1 1
log 5 (    ...to n terms)
(c) 2 (d) 2 24. lim (0.2) 4 8 16
is equal to
n 

2x  1 (a) 2 (b) 4
lim 1
16. x 0 = (c) 8 (d) 0
1  x  2 1

(a) log2 (b) log4 25. lim 3x  9x 2  x


x 

(c) log 2 (d) None of these 1


1
(a) (b)
Limit tending to  3 6

 x  1 3x  4  1 1
17. The value of xlim is equal to (c)  (d) 
 x 2  x  8 6 3
Trigonometric Limit
(a) 2 (b) 3
(c) 1 (d) 0 2sin 2 3x
26. lim 
40 5
x 0 x2
(2  x) (4  x)
18. lim (a) 6 (b) 9
x  (2  x) 45
(c) 18 (d) 3
(a) –1 (b) 1
(c) 16 (d) 32  x 
27. lim (1  x) tan  
x 1  2 
Limit
 n + 2 !   n +1! ,
19. nN= 
n   n  3 ! (a) (b)  + 2
2
(a) 0 (b) 1
2
(c) 2 (d) –1 (c) (d) none of these

 1 2 n 
20. l im  2
 2
 ...   is equal to x  sin x
n 
 1 n 1 n 1 n 2  28. lim
x  x  cos 2 x
(a) 0 (b) – 1/2 (a) 0 (b) 
(c) 1/2 (d) None of these (c) 1 (d) none of these
1  2  3  ...  n
21. The value of lim is equal 1
n  n 2  100 x 2 sin  
lim x
29.
(a)  (b) 1/2 x 0 sin x
(c) 2 (d) 0
(a) 1 (b) 0

1
(c) (d) none of these
2
60 LIMIT, CONTINUITY & DIFFERENTIABILITY

30. lim (sec   tan )  1  cos x



 37. lim is equal to
2 x 0 x (2 x  1)
(a) 0 (b) 1
1 1
(c) –1 (d) 2 (a) log 2 e (b) log e 2
2 2
      (c) 1 (d) none of these
31. lim x.cos   sin  
x   4x   4x 

Limit

sin e x  2  1 
  38.
(a) (b) x 2 l n  x 1
4 3
(c)  (d) 0 (a) 0 (b) – 1
(c) 2 (d) 1
32. lim (1  cos x) cot 2 x
x 1
x x x  x 
39. The value of nlim cos   cos   cos   ...cos  n  is
1  2 4 8 2 
(a) –1 (b)
2
sinx
(c) 1 (d) none of these (a) 1 (b)
x
     x
2  3 sin   h   cos   h  
6  6  (c) (d) None of these
33. lim  sin x
h 0

3h 3 cosh  sinh 
(cos x  1) (cos x  e x )
40. lim  finite non-zero, then n is
(a) -2/3 (b) -3/4 x 0 xn
(c) -2 3 (d) 4/3 (a) 1 (b) 2
(c) 3 (d) 4
4   tan   sin  
34. lim is 1 Form
 0
1  cos 2 2
41. lim (2  tan x)log tan x is equal to
x  / 4
1
(a) (b) 1/2 (a) 0 (b) 1
2
(c) e (d) e–1
(c) 1 (d) 2
1/ x
  
cos ecx  cot x 42. lim  tan   x   is equal to
35. lim is equal to x 0
  4 
x 0 x
(a) 1 (b) e
1
(a) (b) 1 (c) e 2
(d) e–2
2
1/ x 2
1  1  5x 2 
(c) (d) 1 43. lim   is equal to
2 x  0 1  3x 2
 
cos  sin x   cosx (a) e (b) e1/2
36. The value of Limit 4
is equal to
x 0 x (c) e –2
(d) none of these
(a) 1/5 (b) 1/6 x2
 x 1 
(c) 1/4 (d) 1/2 44. lim   is equal to
x  x  1
 
(a) e (b) e–1
(c) e–2 (d) none of these
LIMITS 61

5/ x
45.
x 0 

Limit 1 tan 2 x    sin  x 
x  0

5 2 54. If f  x     x  , where [.] denotes the greatest
(a) e (b) e  0
(c) e (d) none  x  0
1/sin x
46. The limiting value of (cos x) as x  0 is
integer function, then xlim f  x  is equal to
(a) 1 (b) e 0

(c) 0 (d) none (a) 1 (b) 0


cot x
47. The xlim
0
 cos x  is (c) -1 (d) Does not exist
L. H. Rule
(a) - 1 (b) 0
(c) 1 (d) None of these 2
e x  log 1  x   1  x 
55. The value of lim is equal to
48. Limit x sin x
= x 0 x2
x 0

(a) 0 (b) 1 (a) 0 (b) -3


(c) –1 (d) none (c) -1 (d) infinity
Existence of Limit lim x log sinx is equal to
56. x 0
2
x  9x  20
49. Limit where [x] is the greatest integer not (a)  (b) Zero
x 5 x  x
greater than x (c) 1 (d) Cannot be determined

(a) is equal to 1 (b) 0 lim


57.  tanx logsinx =
(c) 4 (d) none x
4

50. l im  x  3  3  x   x  , where [.] denotes the greatest (a) 0 (b) 1


x 3
integer function, is equal to (c) -1 (d) None of these
(a) 4 (b) –4
(c) 0 (d) Does not exist  x3 
 sin x  x  
6 
51. Limit 1 x   x 1  1 x   = where [x] denotes greatest lim  5 
x 1 58. x 0
 x 
integer function  
(a) 0 (b) 1
(c) – 1 (d) does not exist (a) 1/120 (b) -1/120
(c) 1/20 (d) None of these
52. lim {[x] + |x|}, where [.] denotes the greatest integer
x 1

function, x cos x  log 1  x 


59. The value of lim is
(a) is 0 (b) is 1 x 0 x2
(c) does not exist (d) none of these
(a) 1/2 (b) 0
(c) 1 (d) None of these
1
ex 1 3 f x
53. lim  60. If f (9) = 9 and f ' (9) = 1, then lim is equal to
x 0 1
ex 1
x 9 3 x

(a) 0 (b) 1 (a) 0 (b) 1


(c) -1 (d) Does not exist (c) –1 (d) None of these
62 LIMITS

EXERCISE - 2 : PREVIOUS YEAR JEE MAINS QUESTIONS

2x
1  cos 2x  a b 
1. lim is (2002) 7. If xlim 1   2   e 2 , then the values of a and b are
x 0 2x 
 x x 
(2004)
(a)  (b) –1
(a) a R, b R (b) a = 1, b R
(c) zero (d) does not exist
(c) a R, b = 2 (d) a = 1, b = 2
x
 x 2  5x  3  8. Let and be the distinct roots of ax2 + bx + c = 0, then
2. lim  2  is equal to (2002)
x 
 x x2 
1  cos (ax 2  bx  c)
lim is equal to (2005)
(a) e4 (b) e2 x  (x  ) 2
(c) e3 (d) e
1 a2
x (a) (  ) 2 (b)  (  ) 2
 x 3  2 2
3. For x R, xlim 
 x  2
 is equal to (2002)
 
a2
(a) e (b) e–1 (c) 0 (d) (   ) 2
2
(c) e–5 (d) e5

xf (2)  2f (x) 9. Let f : R  R be a positive increasing function with


4. Let f (2) = 4 and f’ (2) = 4. Then, lim is given
x 2 x2
f (3x) f (2x)
by (2002) lim  1 . Then, xlim is equal to (2010)
x  f (x)  f (x)
(a) 2 (b) –2
(c) –4 (d) 3 2
(a) 1 (b)
3
  x 
1  tan  2   (1  sin x) 3
 
5. lim  is (2003) (c) (d) 3
x
 
 x  3 2
2 1  tan
    (  2x)
  2 
 1  {cos 2(x  2)} 
10. lim   (2011)
x 2  x2 
1  
(a) (b) 0
8
(a) equals 2 (b) equals  2
1
(c) (d) 
32 1
(c) equals (d) does not exist
2
log (3  x)  log (3  x)
6. If lim  k, the value of k is
x 0 x 11. Let f : R  [0, ) be such that lim f (x) exists and
x 5
(2003)
(a) 0 (b) –1/3 [f (x)]2  9
lim  0 . Then, lim f (x) equals to (2011)
x 5 x 5
(c) 2/3 (d) –2/3 | x 5|

(a) 3 (b) 0
(c) 1 (d) 2
LIMITS 63

12. If function f (x) is differentiable at x = a, then 1


2 2x
x  f (a)  a 2 f (x) 19. Let p  lim (1  tan x) then log p is equal to :
x 0 
lim (2011)
x a xa
(2016)
(a) 2a f (a) + a2 f’ (a) (b) –a2 f’ (a)
1
(c) af (a) – a2 f’ (a) (d) 2af (a) – a2 f’ (a) (a) 1 (b)
2

(1  cos 2x) (3  cos x) 1


13. lim is equal to (2013) (c) (d) 2
x 0 x tan 4x 4

2x
1 1  a 4 
(a)  (b) 20. If lim 1   2   e3 , then ‘a’ is equal to:
4 2 x   x x 
(c) 1 (d) 2 (2016/Online Set–1)

sin ( cos 2 x) 3
14. lim is equal to : (2014) (a) 2 (b)
x 0 x2 2

 2 1
(a)  (b) (c) (d)
2 3 4
(c) 1 (d) – 
(1  cos 2x)2
21. lim is : (2016/Online Set–2)
tan  x  2 x2  (k  2) x  2 k x 0 2x tan x  x tan 2x
15. If lim  5, then k is equal
x 2 x 2  4x  4
to: (2014/Online Set–2) 1
(a) – 2 (b) 
2
(a) 0 (b) 1
(c) 2 (d) 3 1
(c) (d) 2
2
1  cos 2x  3  cos x 
16. lim is equal to: (2015)
x 0 x tan 4x
cot x  cos x
22. lim 3 equals : (2017)

1
x
2
   2x 
(a) 2 (b)
2
1 1
(c) 4 (d) 3 (a) (b)
24 16
2
e x  cos x  1 1
17. lim is equal to : (2015/Online Set–1)
x 0 sin 2 x (c) (d)
8 4
(a) 2 (b) 3
3x  3
23. lim is equal to : (2017/Online Set–1)
5 3 x 3 2x  4  2
(c) (d)
4 2
1
x tan 2x  2x tan x (a) 3 (b)
18. lim is (2015/Online Set–2) 2
x 0
1  cos 2x 2
3 1
(a) 2 (b) -2 (c) (d)
2 2 2
(c) 1/2 (d) -1/2
64 LIMIT, CONTINUITY & DIFFERENTIABILITY

24. For each t  R, let [t] be the greatest integer less than or 1
equal to t. Then
lim
 27 + x  - 3
3

26. x0
2 equals : (2018/Online Set–3)
1  2 15   9 -  27 + x 
3
lim x       .....     (2018)
x  0   x x
     x 
1 1
(a) does not exist (in R) (b) is equal to 0. (a) (b) -
3 3
(c) is equal to 15. (d) is equal to 120.
1 1
x tan2x - 2x tanx (c) - (d)
25. lim 2 equals: (2018/Online Set–2) 6 6
x0
 1 - cos2x 
1
(a) (b) 1
4

1 1
(c) (d) -
2 2
LIMITS 65

EXERCISE - 3 : ADVANCED OBJECTIVE QUESTIONS

SINGLE ANSWER QUESTIONS 8. If ,  are the roots of the quadratic equation


2
x x
ax + bx + c = 0 then,
e e 2x
1. lim is equal to
x  sin x
x 0

Limit

1  cos ax 2  bx  c 
2
(a) 1 (b) –1 x 
x  
(c) 2 (d) 0
1
3
(a) 0 (b)   2
2
Limit
 4 1
x


2. x 0  x   x2  a2 a2
sin   l n 1  (c)    2 (d)     2
p  3  2 2
3
(a) 9 p (log 4) (b) 3 p (log 4) x 2  9x  20
3 2 9. Limit where [x] is the greatest integer not
(c) 12 p (log 4) (d) 27 p (log 4) x 5 x  x

cos  sin x   cosx greater than x


3. The value of Limit is equal to
x 0 x4 (a) is equal to 1 (b) 0

(a) 1/5 (b) 1/6 (c) 4 (d) none

(c) 1/4 (d) 1/2


Limit

sin e x  2  1 
10.
3 f x x 2 l n  x 1
4. If f (9) = 9 and f ' (9) = 1, then lim is equal to
x 9 3 x (a) 0 (b) – 1
(a) 0 (b) 1 (c) 2 (d) 1
(c) –1 (d) None of these
 x 3 
11. The value of l im  l og a  is
2x x 3  x  6  3 
 a b  
5. If l im 1   2   e2 , then the values of a and b, are
x   x x 
(a) loga6 (b) loga3
(a) a  R, b = 2 (b) a = 1 and b  R (c) loga2 (d) None of these
(c) a  R, b  R (d) a = 1 and b = 2
   ay     by   
sin  a+3h  – 3sin  a+2h  +3sin  a+h  – sina  exp  x l n 1+    exp  x l n 1+   
6. lim is   x     x  
h 0 h3 12. Limit  Limit 
y0  x y 
 
(a) sin a (b) –sin a  
(c) cos a (d) –cos a
(a) a + b (b) a – b

Limit
x 3

 27 l n  x  2 

(c) b – a (d) – (a + b)
7.
x 3 x 2
9 
(a) – 8 (b) 8
(c) 9 (d) – 9
66 LIMIT, CONTINUITY & DIFFERENTIABILITY

 ax      
13. The value of l im a 2  x 2 cot is 19. Limit n cos   sin   has the value equal to
x a 2 ax n   4n   4n 
(a) /3 (b) /4
2a 2a
(a) (b)  (c) /6 (d) none
 

4a  a  h 2 sin  a  h   a 2 sin a
(c) (d)  4a 20. The value of l im
h 0 h
is
 
14. If [x] denotes the greatest integer  x, then (a) 2a sin a + a2cos a (b) 2a sin a – a2 cos a
(c) 2a cos a + a2sin a (d) None of these
1
Limit
n 
   
13 x    23 x   ...   n 3 x  equals
n4     21. If f (a) = 2, f ' (a) = 1, g (a) = –1, g' (a) = 2, then

(a) x/2 (b) x/3 g  x  f a   g a  f  x 


lim is equal to
(c) x/6 (d) x/4 x a x a

(a) 3 (b) 5
 sin  x 
 if x  0 (c) –3 (d) 0
15. If f  x     x  where [x] denotes the
 0
 if x  0 sin 2 x + a sin x
22. If l im be finite, then the value of a and
x 0 x3
greatest integer less than or equal to x, then Limit f (x)
x 0 the limit are given by
equals (a) –2, 1 (b) –2, –1
(a) 1 (b) 0 (c) 2, 1 (d) 2, –1
(c) –1 (d) none 23. The value of

cos 2 x
 tan  x   2 2 2
 ,  x   0 l im 11 / cos x  21 / cos x  ... n1 / cos x  is
 
16. If f (x) =   x   where x 
2
 0,
  x   0 
(a) 0 (b) n
[x] denotes the greatest integer less than or equal to x,
n  n 1
then l im f  x  equals (c)  (d)
x 0 2

(a) 1 (b) –1 n 1
1 1 2 
(c) 0 (d) Does not exist 24. lim  1 e n  e n  ... e n
 is equal to
n  
n 

     (a) e (b) –e
2  3 sin   h   cos   h  
 6   6   is equal to
17. l im  (c) e – 1 (d) 1 – e
h 0

3 h 3 cos h  sin h  25. The values of a, b and c such that

(a) 4/3 (b) – 4/3 aex  b cos x  ce x


l im  2 are
(c) 2/3 (d) 3/4 x 0 x sin x

cos 2  cos 2 x (a) a = 1, b = –2, c = 1 (b) a = 1, b = 2, c = –1


18. Limit 
x  1 x2  x (c) a = 1, b = 2, c = 1 (d) a = –1, b = 2, c = 1
(a) 2 cos 2 (b) – 2 cos 2
(c) 2 sin 2 (d) – 2 sin 2
LIMITS 67

sin x
sin x –  sin x   100 x   99sin x  
26. lim equals 33. The value of lim      , where [.]
x
 1 – sin x + l n sin x x0
  sin x   x  
2
represents greatest integral function is
(a) 1 (b) 2
(a) 199 (b) 198
(c) 3 (d) 4
(c) 0 (d) None of these
1/sin x
 x2 / 2 34. The limiting value of (cos x) as x  0 is
e  cos x
27. Limit 
x 0 x 3 sin x (a) 1 (b) e

(a) 1/4 (b) 1/6 (c) 0 (d) none


(c) 1/12 (d) 1/8 MULTIPLE CHOICE QUESTIONS
1/x 2
Limit sin 1  sec x  35. If lim (cos x + a sin bx) = e , then the values of a and b
28. x 0
x 0
are
(a) is equal to /2 (b) is equal to 1
(a) a = 1, b = 2 (b) a = 2, b = 1/2
(c) is equal to zero (d) none of these
1
 cos 2x  (c) a  2 2 , b  (d) a = 4, b = 2
29. Lim x  x – 1 , where [.] denotes greatest integer 2
x 0

function, is equal to / x
 a x  bx  cx 
(a) 1 (b) 0 36. If lim   , (a, b, c, > 0) is equal to
x 0
 3 
(c) e (d) Does not exists
(a) 1, if  = 1 (b) abc, if  = 1
x3 (c) abc, if  = 3
2/3
(d) (abc) , if  = 2
30. If Lim  1, a > 0, then a + 2b is equal to
x  0 a  x  bx – sin x 
37. The value of a for which
(a) 36 (b) 37
4
(c) 38 (d) 40
lim
 e –1
x

 8, is
x0  x2   x2 
4 4
sin x – x cos x  x 4 20 sin  2  l og e 1 
31. l im is equal to a   2
x0
 4
x 4 e2x –1– 2x 4  (a) –2 (b) –1
(a) 0 (b) –1/6 (c) 1 (d) 2
(c) 1/6 (d) does not exist
aex – b cos x  ce – x
38. If lim  2 , then
1  9  x 0 x sin x
32. If f (n + 1) =  f n   , n  N and f (n) > 0 for all
2  f  n  
(a) a = 1 (b) b = 2
n  N then l im f (n) is equal to (c) c = –2 (d) c = 0
n 

x 1+ a cos x  – bsin x


(a) 3 (b) – 3 39. If lim  1, then
x 0 x3
(c) 1/2 (d) None of these
–3 5
(a) b  (b) a 
2 2

–1 –5
(c) b  (d) a 
2 2
68 LIMIT, CONTINUITY & DIFFERENTIABILITY

40. If x is a real number in [0, 1] then the value of Using the following passage, solve Q.45 to Q.47
2m
lim lim [1 + cos (n!x)] is given by, where [x] Passage –2
m n 

represents greatest integer < x. n


 x 
Consider two functions f (x) = l im  cos  and
(a) 2 if x is rational n   n
(b) 1 for all x
(c) 1 if x is irrational (d) 2 for all x g(x) = –x4b where b = xl 
im  
x 2  x 1 – x 2 1 Then.

41. The limit of sequence 2 , 2 2 , 2 2 2 ....... is 45. f (x) is


– x2
(a) a rational number (b) 2 –x2 2
(a) e (b) e
(c) is an irrational number (d) 2 2
x2
x2 2
Using the following passage, solve Q.42 to Q.44 (c) e (d) e

Passage –1 46. g(x) is


AP is a diameter of a unit circle with centre at O. Let AC be (a) –x2 (b) x2
an arc of this circle, which subtends angle  radian at centre (c) x4 (d) –x4
O. A tangent line is drawn to the circle at the point A and a 47. Number of solutions of f (x) + g (x) = 0 is
segement AB on this tangent is laid off whose length is
(a) 2 (b) 4
equal to that of the arc AC. A straight line BC is drawn to
intersect the extension of the diameter AP at Q. CD is the (c) 0 (d) 1
perpendicular let fall from the point C upon the diameter AP. ASSERTION REASON
42. The area of the trapezoid ABCD is (A) If ASSERTION is true, REASON is true, REASON is a
correct explanation for ASSERTION.
1 – cos  
(a) (b) ( + sin ) sin2 (B) If ASSERTION is true, REASON is true, REASON is
 – sin  2
not a correct explanation for ASSERTION.
 (C) If ASSERTION is true, REASON is false.
(c) 2 cos2 ( – sin ) (d) ( + sin )
2 (D) If ASSERTION is false, REASON is true.
43. The length AQ equal to 48. Assertion : If a and b are positive and [x] denotes
greatest integer < x, then
 1 – cos    1 – cos  
(a) (b) x b b
 – sin   + sin  l im 
x  0 a  x  a
 1 + cos    1 + cos  
(c) (d) x
 – sin   + sin  Reason : l im  0 where {x} denotes
x x
44. The value of the Lim AQ is fractional part of x.
  0
(a) A (b) B
(a) 0 (b) 1
(c) C (d) D
(c) 2 (d) 3
(e) E
LIMITS 69

Match the Following


f x
49. Assertion : For the existence of lim . It is 50. Column–I Column–II
x a g x

f  x  (A) Lt x cos  .sin   (P)
necessary that lim exists. x  8x 8x 8
x a g  x 

Reason : If f (a) = g(a) = 0 then tan  – 2  x 2 –  – 2  x 2


(B) Lt  (Q) 2
x 0 sin 2  x 
f x f  x 
lim  lim .
x a g  x  x a g  x 
2x – sin x + cos x 8
(C) Lt  (R)
(a) A (b) B x  x + cos 2 x  sin 2 x 

(c) C (d) D
1
(e) E  x n – 1  x –1 n –1
2
(D) Lt    (S) e
x 1  n  x – 1 
 

(T) 0
Subjective

sin x – tan x
51. The value of Lim is
x 0 tan –1 x – sin –1 x
LIMITS 70

EXERCISE - 4 : PREVIOUS YEAR JEE ADVANCED QUESTIONS

Single Answers Questions x tan 2x – 2 x tan x


6. l im (1999)
1. If f (a) = 2, f’ (a) = 1, g (a) = –1, g’ (a) = 2, then the value of x 0
1– cos 2x 2
g(x) f (a)  g(a) f (x)
lim is (1983) (a) 2 (b) –2
x a xa
1 1
1 (c) (d) 
(a) –5 (b) 2 2
5
x
(c) 5 (d) None of these  x 3 
7. For x R, xlim 
 x  2
 is equal to (2000)
 
 1 2 n 
2. l im  2
 2
 ...  is equal to : (1984) (a) e (b) e–1
n  1  n 1 n 1  n2 
(c) e–5 (d) e5
1
(a) 0 (b) –
2
8. l im

sin  cos 2 x  equals (2001)
x 0 x2
1
(c) (d) none of these (a) – (b) 
2
(c) /2 (d) 1
 sin [x]
 , [x]  0 (cos x  1) (cos x  e x )
3. If f (x)   [x] where [x] denotes the 9. The integer n for which lim is a
 0, [x]  0
x 0 xn

finite non-zero number, is (2002)
greatest integer less than or equal to x, then
(a) 1 (b) 2
lim f (x) equals (1985)
x 0 (c) 3 (d) 4
(a) 1 (b) 0 10. Let f : R  R be such that f (1) = 3 and f  (1) = 6, then
(c) –1 (d) None of these 1/ x
 f 1  x  
lim   equals : (2002)
x 0 
1  f 1 
(1  cos 2 x)
2
4. The value of lim (1991)
x 0 x 1
(a) 1 (b) e 2
(a) 1 (b) –1
(c) e2 (d) e3
(c) 0 (d) None of these
{(a  n) nx  tan x} sin nx
1  cos 2 (x  1) 11. If lim  0, where n is non zero
5. lim (1998)
x 0 x2
x 1 x 1
real number, then a is equal to (2003)
(a) exists and it equals 2 n 1
(a) 0 (b)
n
(b) exists and it equals – 2
(c) does not exist because x – 1 0 1
(c) n (d) n 
(d) does not exist because left hand limit is not equal to n
right hand limit
LIMITS 71

12. The value of l im ((sin x)1/x + (1/x)sin x), where x > 0 is Multiple Answers Questions
x 0
x2
(2006) a  a2  x2 
17. Let L  lim 4 , a  0 . If L is finite, them
(a) 0 (b) –1 x 0 x4
(c) 1 (d) 2
(2009)
1
(a) a = 2 (b) a = 1
13. If lim [1  x log (1  b 2 )]  2b sin 2 , b
x
> 0 and
x 0
1 1
(–, ], then the value of is (2011) (c) L  (d) L 
64 32
 
(a)  (b)  Let f (x) 
1  x(1 |1  x |)  1 
4 3 18. cos   for x  1. then
|1  x | 1 x 
  (2017)
(c)  (d) 
6 2 (a) lim x 1 f (x)  0

(b) lim x 1 f (x)  0


 x2  x  1 
14. If xlim   ax  b   4, then (2012)

 x 1  (c) lim x 1 f (x) does not exist

(a) a = 1, b = 4 (b) a = 1, b = –4 (d) lim x 1 f (x) does not exist


(c) a = 2, b = –3 (d) a = 2, b = 3 Fill in the Blanks
15. Let  (a) and (a) be the roots of the equation
x
19. lim (1  x) tan  ... (1978)
 3

1  a 1 x2   1 a 1 x    6

1  a  1  0, x 1 2

where a > –1. Then, lim (a) and alim


 0
(a) are (2012)
a 0
 4 1 2 
5 1   x sin    x  
x
(a)  and 1
2
(b)  and  1 20. lim      ... .
(1987)
2 x   (1 | x |3 ) 
 
7 9  
(c)  and 2 (d)  and 3
2 2
(1988)
(1  cos 2x) (3  cos x) 21. ABC is an isosceles triangle inscribed in a circle of radius
16. lim is equal to (2013) r. If AB = AC and h is the altitude from A to BC, then the
x 0 x tan 4x
triangle ABC has perimeter P  2 ( 2hr  h 2  2hr )
1 1
(a)  (b) A
4 2 and area A = ... . Also, lim  ... . (1989)
h 0 P3
(c) 1 (d) 2
x4
x6
22. lim    .... . (1991)
x  x  1
 

1/ x 2
 1  5x 2 
23. lim    ... . (1996)
x 0 1  3x 2
 
72 LIMIT, CONTINUITY & DIFFERENTIABILITY

24.
log (1  2h)  2 log (1  h)
(1997) (a  h) 2 sin (a  h)  a 2 sin a
lim  ... . 30. Evaluate lim (1980)
h 0 h2 h 0 h
25. For each positive integer n, let
a x 1
1 31. Use the formula lim  log e a, to find
1 x 0 x
yn 
n
 n  1 n  2  ...  n  n   .
n

For x   , let [x] be the greatest integer less than or 2x  1


lim (1982)
x  0 (1  x)1/ 2  1
equal to x. If nlim

y n  L, then the value of [L] is _____.
(2018)  1/ x
32. Find lim {tan (  x)} . (1993)
True/False x 0 4

26. If lim
x a
[f (x) g(x)] exists, then both lim
x a
f(x) and lim
x a
g(x)
33. The largest value of the non-negative integer a for which
exist. (1981)
Subjective Problems 1 x
 ax  sin  x  1  a 1 x 1
x 1 lim    is (2014)
  x 1  x  sin  x  1  1  4
27. Evaluate the following limit lim  2  (1978)  
x 1  2x  7x  5 

34. Let m and n be two positive integers greater than 1. If


 x   ecos n   e 
28. Evaluate lim (1  x) tan   (1978)
x 1  2  lim      e  m
 0  m  2
then the value of is (2015)
  n
x  sin x
29. Evaluate lim (1979) x 2 sin(x)
x 0 x  cos 2 x 35. Let ,   R be such that lim =1. Then 6( + )
x 0
x  sin x
equals (2016)
LIMITS 73

ANSWER KEY
EXERCISE - 1 : BASIC OBJECTIVE QUESTIONS
1. (a) 2. (d) 3. (b) 4. (a) 5. (b) 6. (d) 7. (a) 8. (b) 9. (a) 10. (c)

11. (a) 12. (c) 13. (d) 14. (a) 15. (b) 16. (b) 17. (d) 18. (a) 19. (a) 20. (b)

21. (b) 22. (c) 23. (d) 24. (b) 25. (b) 26. (c) 27. (c) 28. (c) 29. (b) 30. (a)

31. (a) 32. (b) 33. (d) 34. (b) 35. (c) 36. (b) 37. (a) 38. (d) 39. (b) 40. (c)

41. (b) 42. (d) 43. (d) 44. (c) 45. (a) 46. (a) 47. (c) 48. (b) 49. (d) 50. (b)

51. (c) 52. (c) 53. (d) 54. (d) 55. (b) 56. (b) 57. (a) 58. (a) 59. (a) 60. (b)

EXERCISE - 2 : PREVIOUS YEAR JEE MAINS QUESTIONS


1. (d) 2. (a) 3. (c) 4. (c) 5. (c) 6. (c) 7. (b) 8. (d) 9. (a) 10. (d)

11. (a) 12. (d) 13. (d) 14. (a) 15. (d) 16. (a) 17. (d) 18. (c) 19. (b) 20. (b)

21. (a) 22. (b) 23. (b) 24. (d) 25. (c) 26. (c)

EXERCISE - 3 : ADVANCED OBJECTIVE QUESTIONS


1. (c) 2. (b) 3. (b) 4. (b) 5. (b) 6. (d) 7. (c) 8. (c) 9. (d) 10. (d)
11. (a) 12. (b) 13. (c) 14. (d) 15. (d) 16. (d) 17. (a) 18. (c) 19. (b) 20. (a)
21. (b) 22. (b) 23. (b) 24. (c) 25. (c) 26. (b) 27. (c) 28. (d) 29. (a) 30. (c)
31. (c) 32. (a) 33. (b) 34. (a) 35. (a,c) 36. (c,d) 37. (a,d) 38. (a,b) 39. (a,d) 40. (a,c)
41. (a,b) 42. (b) 43. (a) 44. (d) 45. (b) 46. (a) 47. (a) 48. (a) 49. (e)

50. (A  P, B  T, C  Q, D  S) 51. 0001

EXERCISE - 4 : PREVIOUS YEAR JEE ADVANCED QUESTIONS

1. (c) 2. (b) 3. (d) 4. (d) 5. (d) 6. (c) 7. (c) 8. (b) 9. (c) 10. (c)

2
11. (d) 12. (c) 13. (d) 14. (b) 15. (b) 16. (d) 17. (a,c) 18. (b,c) 19. 20. –1

1 1 2
21. h ( 2hr  h 2 ), 22. e5 23. e2 24. –1 25. (1) 26. False 27.  28. 29. 0
128r 3 

30. a2 cos a+ 2 a sin a 31. 2  n 2 32. e2 33. (2) 34. (0002) 35. (7)

Dream on !!

03
TRIGONOMETRY
75 TRIGONOMETRY

TRIGONOMETRY
Trigonometric Ratios of Standard Angles
TRIGONOMETRIC RATIOS & IDENTITIES
1. The meaning of Trigonometry T–Ratio Angle ()
 0° 30° 45° 60° 90°
Tri Gon Metron
  
3 sides Measure
1 1 3
Hence, this particular branch in Mathematics was sin 0 1
2 2 2
developed in ancient past to measure 3 sides, 3 angles
and 6 elements of a triangle. In today’s time–trigonometric
functions are used in entirely different shapes. The 2 basic 3 1 1
cos 1 0
functions are sine and cosine of an angle in a right–angled 2 2 2
triangle and there are 4 other derived functions.
1
tan 0 1 3 
3
H
P
1
cot  3 1 0
3
B
sin  cos  tan  cot  sec  cosec  2
sec 1 2 2 
3
P B P B H H
H H B P B P
2
cosec  2 2 1
3
2. Basic Trigonometric Identities

(a) sin2 + cos2 = 1 : –1 sin 1; –1 cos 1  R The sign of the trigonometric ratios in different quadrants
are as under :
(b) sec2 – tan2 = 1 : | sec|  1  R

(c) cosec2 – cot2 = 1 : | cosec|  1  R


TRIGONOMETRY
76 76
TRIGONOMETRY

3. Trigonometric Ratios of Allied Angles


 3   3 
sin       cos  cos       sin 
 2   2 
Using trigonometric ratio of allied angles, we could find
the trigonometric ratios of angles of any magnitude.
tan       tan  cot       cot 

sin (–) = – sin  cos (–) = cos 


 3   3 
tan      cot  cot      tan 
     2   2 
sin      cos  cos      sin 
 2   2 

sec        sec  cos ec        cos ec 


tan (–) = – tan  cot (–) = –cot 

 3   3 
    sec       cosec  cos ec       sec 
tan      cot  cot      tan   2   2 
2  2 

cosec (–) = – cosec  sec (–) = sec 


 3   3 
sin       cos  cos      sin 
 2   2 
   
sec      cos ec cos ec      sec 
 2   2 
sin  2      sin  cos  2     cos 

   
sin      cos  cos       sin   3   3 
2  2  tan       cot  cot       tan 
 2   2 

sin       sin  cos        cos 


tan  2      tan  cot  2      cot 

   
tan       cot  cot       tan   3   3 
2  2  sec      cos ec  cos ec       sec 
 2   2 

tan        tan  cot        cot 


sec  2     sec  cosec  2      cos ec 

    sin  2     sin  cos  2     cos 


sec       cosec cos ec      sec 
2  2 

tan  2     tan  cot  2     cot 


sec        sec  cosec       cosec 

sec  2     sec  cos ec  2     cosec 


sin        sin  cos        cos 
TRIGONOMETRY 77

4. Trigonometric Functions of Sum or 5. Multiple Angles and Half Angles


Difference of Two Angles
 
(a) sin 2A = 2 sin A cos A ; sin  = 2 sin cos
(a) sin (A + B) = sin A cos B + cos A sin B 2 2

(b) cos 2A = cos2A – sin2A = 2 cos2A – 1 = 1 – 2 sin2 A ;


(b) sin (A – B) = sin A cos B – cos A sin B
 
(c) cos (A + B) = cos A cos B – sin A sin B 2cos2 = 1 + cos , 2 sin2 = 1 – cos 
2 2

(d) cos (A – B) = cos A cos B + sin A sin B



2 tan A 2 tan
(c) tan 2A = ; tan  = 2
tan A  tan B 1  tan 2 A 2 
(e) tan (A  B)  1  tan
2
1  tan A tan B

2 tan A 1  tan 2 A
(d) sin 2A = 2 ; cos 2A =
tan A  tan B 1  tan A 1  tan 2 A
(f) tan (A  B) 
1  tan A tan B
(e) sin 3A = 3 sin A – 4 sin3 A
(f) cos 3 A = 4 cos3 A – 3 cos A

cot A cot B  1
(g) cot (A + B) = 3 tan A  tan  A
cot B  cot A (g) tan 3A =
1  3tan 2 A

6. Transformation of Products into Sum


cot A cot B  1
(f) cot (A - B) = or Difference of Sines & Cosines
cot B  cot A

(h) sin2 A – sin2 B = cos2B – cos2A = sin (A + B) . sin (A – B) (a) 2 sin A cos B = sin (A + B) + sin (A – B)

(i) cos2 A – sin2 B = cos2B – sin2A = cos (A + B) . cos (A – B)


(b) 2 cos A sin B = sin (A + B) – sin (A – B)

(j) tan (A + B + C) = tanA  tanB  tanC  tanAtanBtanC


(c) 2 cos A cos B = cos (A + B) + cos (A – B)
1 tanAtanB  tanBtanC  tanCtanA
(d) 2 sin A sin B = cos (A – B) – cos (A + B)
78 TRIGONOMETRY

7. Factorisation of the Sum or Difference of 9. Conditional Identities


Two Sines or Cosines
If A + B + C =  then :
(i) sin 2A + sin2 B + sin 2C = 4 sin A sin B sin C
CD CD A B C
(a) sin C + sin D = 2 sin cos (ii) sin A + sin B + sin C = 4 cos cos cos
2 2 2 2 2
(iii) cos 2A + cos 2B + cos 2C = –1 – 4cosA cosB cosC
CD CD
(b) sin C – sin D = 2 cos sin A B C
2 2 (iv) cos A + cos B + cos C = 1 + 4sin sin sin
2 2 2
(v) tan A + tan B + tan C = tanA tanB tanC
CD CD
(c) cos C + cos D = 2 cos cos A B B C C A
2 2 (vi) tan tan  tan tan  tan tan  1
2 2 2 2 2 2

CD CD A B C A B C
(d) cos C – cos D = – 2 sin sin (vii) cot  cot  cot  cot .cot .cot
2 2 2 2 2 2 2 2

(viii) cot A cot B + cot B cot C + cot C cot A = 1


8. Important Trigonometric Ratios
10. Range of Trigonometric Expression
(a) sin n  = 0 ; cos n  = (–1)n ; tan n = 0 where n Z
E = a sin  + b cos 

 3 1 5  b
(b) sin 15º or sin  = cos 75º or cos ; E  a   b 2 sin(  ),  where tan   
12 2 2 12  a

 a
 3 1 5 E  a 2  b 2 cos(  ),  where tan   
cos 15º or cos = = sin 75º or sin ;  b
12 2 2 12
Hence for any real value of ,  a 2  b 2  E  a 2  b 2

3 1 11. Sine and Cosine Series


tan 15º =  2  3 = cot 75º ;
3 1 (a) sin  + sin ( + ) + sin ( + ) + ..... + sin ( + n  1  )

n
3 1 sin
tan 75º =  2  3 = cot 15º = 2 sin (  n  1 )
3 1  2
sin
2

 5 1
(c) sin or sin 18º = & (b) cos  + cos (  + ) + cos ( + 2) + ...... + cos ( + n  1  )
10 4
n
sin
 5 1 = 2 cos (  n  1 )
cos 36º or cos   2
5 4 sin
2
TRIGONOMETRY 79

(d) y = cot x,
12. Graphs of Trigonometric Functions
x  R – {n; n  z}; y  R
(a) y = sin x,
x  R ; y  [–1, 1]
y

x
0

(e) y = cosec x,
x R – {n; n Z}; y  ––][1, )
(b) y = cos x,
x  R ; y  [–1, 1]

(f) y = sec x,

  
x  R   2n  1 ; n  Z  ; y  ––][1, )
(c) y = tan x,  2 

  
x  R   2n  1 ; n  Z  ; y  R
 2 
80 TRIGONOMETRY

4. sin  = sin    = n  + ( – 1) n , where


TRIGONOMETRIC EQUATIONS
  
13. Trigonometric Equations    , 
 2 2
The equations involving trigonometric functions of
5. cos = cos = 2n± , where [0, ].
unknown angles are known as Trigonometric equations.
e.g., cos = 0, cos2– 4 cos = 1.   
6. tan = tan = n + , where     , 
A solution of a trigonometric equation is the value of the  2 2

unknown angle that satisfies the equation.


7. sin2= sin2 = n ± .

1   3 9 11 8. cos2 = cos2 = n ± .


e.g., sin      or   , , , ,...
2 4 4 4 4 4 9. tan2 = tan2 = n ± .

Thus, the trigonometric equation may have infinite 


10. sin = 1 = (4n +1) .
number of solutions and can be classified as : 2
(i) Principal solution
11. cos = 1 = 2n .
(ii) General solution
12. cos = – 1 = (2n + 1) .
14. General Solution 13. sin = sin and cos = cos = 2n + .

Since, trigonometric functions are periodic, a solution


generalised by means of periodicity of the trigonometrical
functions. The solution consisting of all possible solutions
of a trigonometric equation is called its general solution.

14.1 Results 1. Every where in this chapter ‘n’ is taken as an integer,


if not stated otherwise.
1. sin = 0 = n  2. The general solution should be given unless the
solution is required in a specified interval or range.

2. cos = 0 (2n + 1) 3.  is taken as the principal value of the angle.
2
(i.e., Numerically least angle is called the principal
3. tan = 0 = n 
value).
81 TRIGONOMETRY

SOLVED EXAMPLES

Example – 1

Multiplying and dividing by 2 sin   , we get
Solve : If sec and cosec are the roots of  2 
x2 – px + q = 0, then show p2 = q (q + 2).
 
2 cos   .sin  
2
Sol. Since, sec and cosec are roots of x – px + q = 0
cot    2   2   sin   sin 
sec + cosec = p and sec . cosec = q       cos   cos 
2 sin   sin  
 2   2 
1
sin + cos = p sin . cos and sin . cos  
q sin   sin 
 cot   .
cos   cos 
p
 sin + cos = .
q Example – 3
Squaring both sides, we get Solve : Prove that
p2 tan A + 2 tan 2A + 4 tan 4A + 8 cot 8A = cot A.
sin2 + cos2 + 2 sin . cos  
q2
 1  tan 2 4A 
p 2 Sol. L.H.S. = tan A + 2 tan 2A + 4 tan 4A + 8  
1 + 2 sin . cos    2 tan 4A 
q2

 4 tan 2 4A  4  4 tan 2 4A 
2 p2 = tan A + 2 tan 2A +  
or 1  2  p2 = q (q + 2). tan 4A
q q  

= tan A + 2 tan 2A + 4 cot 4A


Example – 2

Solve : If , and are in A.P., show that  1  tan 2 2A 


= tan A + 2 tan 2A + 4  2 tan 2A 
 
sin   sin 
cot   .
cos   cos 
 2 tan 2 2A  2  2 tan 2 2A 
= tan A +  tan 2A

 
Sol. Since, , and are in A.P.
= tan A + 2 cot 2A
2= + 
 1  tan 2 A  tan 2 A  1  tan 2 A
 = tan A + 2  2 tan A  
cot = cot  
 2    tan A

= cot A = R.H.S.
 Note: Students are adviced to learn above result as formulae.
cos  
 cot    2  i.e., tan A + 2 cot 2A = cot A

sin  
 2 
TRIGONOMETRY
82 82
TRIGONOMETRY

Example – 4 Example – 6

Solve : Evaluate : Solve : Prove that :


cos 12° cos 24° cos 36°. cos 48°. cos 72°. cos 84°.
3
cos3 A + cos3 (120° + A) + cos3 (240° + A) = cos 3A
4
Sol. cos 12° cos 24° cos 36° cos 48°. cos 72°. cos 84°.
 cos 12° cos 24° cos 48° . cos (180°–96°). Sol. We know that
cos 36° . cos 72°
1
 – (cos 12° . cos 24° . cos 48°. cos 96°) . cos 3A = 4 cos3 A – 3 cos A cos3 A = (cos 3A + 3 cosA)
4
(cos 36° . cos 72°)
1 1
 LHS = {cos 3A + 3 cos A} + {cos (360° + 3A) +
4 2
sin (2 .12) sin (2 .36) 4 4
  .
24.sin (12) 2 2.sin (36) 1
3 cos (120° + A)} + {cos (720 + 3A) + 3 cos (240° + A)}
4
sin(2 n A)
using, cos A cos 2 A ...... cos 2n – 1 A  1 1
2n sin A  LHS {cos 3A + 3 cos A} +
4 4
sin (192) sin (144) 1
  . {cos 3A + 3 cos 120° + A)} + {cos 3A + 3 cos (240° + A)}
16.sin (12) 4.sin (36) 4
3 3
sin (180  12) . sin (180  36)  LHS = cos 3A + {cos A + cos (120° + A) +
  4 4
64 . sin12. sin 36 cos (240° + A)}

sin12.sin 36 3 3
1  LHS = cos 3A + {cos A + 2 cos (180°–A) cos 60°}
  . 4 4
64 sin 12.sin 36 64
3 3 1 3
Example – 5  LHS= cos3A  cos A  2cos A    cos3A=RHS
4 4 2 4
Solve : Prove that : ALITER
tan A + tan (60° + A) – tan (60° – A) = 3 tan 3A We have,
cos A + cos (120° + A) + cos (240° + A)
Sol. We have, = cos A + 2 cos (180° + A) cos 120°
LHS = tan A + tan (60° + A) – tan (60° – A)
 CD C  D
cos C  cos D  2cos 2 cos 2 
3  tan A 3  tan A
 LHS  tan A  
1  3 tan A 1  3 tan A = cos A – cos A = 0
 cos3 A + cos3 (120° + A) + cos3 (240° + A)
8tan A
 LHS  tan A  = 3 cos A cos (120° + A) cos (240° + A)
1  3tan 2 A
[ a + b + c = 0 a3 + b3 + c3 = 3abc]
9 tan A  3tan 3 A = 3 cos A cos (180° – 60° + A) cos (180° + 60° + A)
 LHS 
1  3tan 2 A = 3 cos A cos {180° – (60° – A)} cos {180° + (60° + A)}
= 3 cos A cos (60° – A) cos (60° + A)
 3tan A  tan 3 A 
 LHS  3   = 3 tan 3A = RHS 1 3
2  3  cos3A  cos3A
 1  3tan A  4 4
TRIGONOMETRY 83

Example – 7
(cos 60  sin18)(cos36  sin 30)
 LHS 
3 3
Solve : Prove that : sin 3A sin A + cos 3A cos A = cos 2A 3 (cos 60  sin18)(cos36  sin 30)

Sol. We have,
1 5  1  5  1 1 
    
cos3A  3cos A  2 4   4 2
cos3 A   LHS 
4 1 5  1  5  1 1 
    
2 4   4 2
 LHS = sin 3A sin3 A + cos 3A cos3 A

 3sin A  sin 3A   cos3A  3cos A 


 LHS = sin 3A    cos3A   (3  5) (3  5) 9  5
 4   4     1  RHS
( 5  1) ( 5  1) 5  1

1
 LHS = {3 (cos A cos 3A + sin A sin 3A) + (cos2 3A–sin2 3A)} Example – 9
4

1 Solve : Prove that : 4 sin 27° = (5  5)  (3  5)


 LHS = {3 cos (3A – A) + cos 2 (3A)}
4

Sol. We have,
1
 LHS = {3 cos 2A + cos 3 (2A)} 16 sin2 27° = 8 (1 – cos 54°)
4
 16 sin2 27° = 8 (1 – sin 36°)
1
 LHS = {3 cos 2A + (4 cos3 2A – 3 cos 2A)}
4  10  2 5 
 16sin 2 27  8 1  
= cos3 2A = RHS 4
 

Example – 8

Solve : Prove that : tan 6° tan 42° tan 66° tan 78° = 1  
16sin 2 27  2 4  10  2 5 
Sol. We have,
 16sin 2 27  8  2 10  2 5
sin 6 sin 42 sin 66 sin 78
LHS 
cos 6 cos 42 cos 66 cos 78  16 sin2 27° = (5  5)  (3  5)  2 (5  5) (3  5)

2 2

 LHS 
(2sin 66 sin 6)(2sin 78 sin 42)
(2cos66 cos6)(2cos78 cos 42)
 16 sin2 27° =  5  5   3  5  2 (5  5) (3  5)

 LHS 
(cos 60  cos 72)(cos36  cos120)  16 sin2 27° =  5 5  3 5 
(cos60  cos 72)(cos36  cos120)

sin 27 5  5  3  5
84 TRIGONOMETRY

Example – 10 Example – 11

Solve : If A + B + C = , then prove the following


tan 3x 1
Solve : Prove that never lies between and 3. (i) sin2A + sin 2B + sin 2C = 4 sin A . sin B . sin C
tan x 3
(ii) sin2 A + sin2 B + sin2 C = 2 + 2 cos A . cos B. cos C

A B C


(iii) cos2    cos2    cos 2  
tan 3x 2 2 2
Sol. Let y  . Then,
tan x
 A  B C
= 2 + 2 sin   sin   sin  
 2 2 2
3tan x  tan 3 x
y
tan x(1  3tan 2 x) A B C
(iv) cos    cos    cos  
2 2 2

tan 2 x A B  C


 y  4cos   cos   cos  
1  3 tan 2 x  4   4   4 

 (3y – 1) tan2 x = y – 3 cos A  cos B  cos C  1 A C


(v)  cot   cos  
cos A  cos B  cosC  1  2 2
y3
 tan2 x =
3y  1
Sol. L.H.S.
= sin 2A + sin 2B + sin 2C
Now,

tan2 x 0 for all x  2A  2B   2A  2B 


 2sin   cos   + 2 sinC cos C
 2   2 
y3 = 2 sin (A + B) . cos (A – B) + 2 sinC [–cos (A + B)]
 0
3y  1
= 2 sin C . cos (A – B) – 2sinC . cos (A + B)
= 2 sin C [cos (A – B) – cos (A + B)]
(y  3) (3y  1)
 0 = 2 sin C × 2 sin A sin B
(3y  1) 2
= 4 sin A sin B sin C.
= R.H.S.
[(3y  1)2  0]
(ii) L.H.S.
 (y – 3) (3y – 1) 0 = sin2 A + sin2 B + sin2 C

2 1  cos 2B 1  cos 2C
1 = sin A  
 y  or, y  3 2 2
3
1
 y does not lie between 1/3 and 3. = [2 + 2 sin2 A – (cos 2B + cos 2C)]
2
TRIGONOMETRY 85

(iv) L.H.S.
1  2B  2C 
= [2 + 2 sin2 A – 2 cos  
2  2  A  B C
 cos    cos    cos  
2 2 2
 2B  2C 
cos  ]
 2 
B C B C
= 1 + sin2 A – cos (B + C) . cos (B – C)  A 22 22
 sin    2cos   cos  
 2   2   2 
= 2 – cos2 A + cosA . cos (B – C)
   
= 2 + cos A [–cos A + cos (B – C)]
= 2 + cos A [cos (B + C) + cos (B – C)]
= 2 + cos A × 2 cos B . cos C  A  A  BC  BC
 2sin   cos    2cos   cos  
= 2 + 2 cos A . cos B . cos C  4   4   4   4 
(iii) L.H.S.
 A   A  A  BC
 2sin   cos    2cos   cos  
A  B C  4   4   4   4 
 cos 2    cos2    cos 2  
2
  2
  2

  A 
 A  1  cos B 1  cos C  sin  4  
 cos2        A    
2 2 2  2cos  
 4   B  C 
  cos  
  4 
A 1
 1  cos 2    (cos B  cos C)
2 2

  BC 
A 1  BC
 1  cos 2    2cos 
 BC  sin  4  
 .cos      A    
2 2  2   2   2cos  
 4   B  C 
  cos  
A A  BC   4 
 1  1  sin 2    sin   .cos  
2 2  2 

 A   A   B  C      B     C   
 2  sin   sin    cos    cos    
2
    2  2     A   4   4   
 2cos  
 4     C     B   
  cos    
 A   B C   B  C    4   4   
2  sin   cos    cos  
2
   2   2 

A  B  C A  B  C


 2  sin    2sin   .sin     2cos   .2cos   .cos  
2 2  2  4   4   4 

A  B C A  B  C


 2  2sin   .sin   .sin    4cos   .cos   .cos  
2 2 2  4   4   4 
= R.H.S.
= R.H.S.
86 TRIGONOMETRY

(v) L.H.S. Solution of Equations by Factoring


cos A  cos B  cos C  1
 Example – 12
cos A  cos B  cos C  1

(cos A  cos C)  (1  cos B) Solve : 2 cos x cos 2x = cos x.



(cos A  cos C)  (1  cos B)
Sol. The given equation is equivalent to the equation cos x
(2 cos 2x – 1) = 0.
AC  AC 2 B
2cos   cos    2sin  
  2   2  2 This equation is equivalent to the collection of equations.
AC  AC 2 B

2cos   cos    2sin  
 2   2  2

 
AC  AC 2AC
cos x  0,  x  2   n, n  Z,
2cos   .cos    2 cos     
 2   2   2  cos 2x  1 ,  2x     2k, i.e. x     k, k  Z.
  2
AC AC 2AC
 3 6
2cos   .cos    2cos  
 2   2   2 

 
Answer :  n,   k (n, k Z)
 A  C   A  C   A  C  2 6
2cos    cos    cos  
 2   2   2 

 A C   A C   A  C  Solution of Equations Reducible to
2cos   cos    cos  
 2   2   2 
Quadratic Equations

 AC AC Example – 13


cos    cos  
  2   2 
 AC AC Solve : 3 cos2 x – 10 cos x + 3 = 0.
cos    cos  
 2   2 
Sol. Assume cos x = y. The given equation assumes the form

A C 3y2 – 10y + 3 = 0.


2cos   .cos  
 2 2
1
A C Solving it, we find that y1 = , y = 3.
2sin   .sin   3 2
2 2

The value y2 = 3 does not satisfy the condition since |cos x |  1.


A C
 cot   .cot  
2 2 1 1
Consequently, cos x = , x = ± cos–1 + 2n, n  Z
3 3
= R.H.S.

1
Answer : ± cos–1   + 2n (n  Z).
3
TRIGONOMETRY 87

Solution of Homogeneous Equations Solving Equations by Introducing an


and Equations Reducible to them Auxiliary Argument

Example – 15
Equations of the form

a 0 sin n x + a 1 sin n–1 x cos x + a 2 sin n–2 x cos 2 x


3 1
Solve : cos x  sin x  1
+ .... + an–1 sin x cosn–1 x + an cosn x = 0, 2 2

where a0, a1, ..., an are real numbers, are said to be


   
homogeneous with respect to sin x and cos x. Sol. cos cos x + sin sin x = 1, cos  x –  = 1,
6 6  6
Example – 14  
x– = 2 n (n Z), x = + 2n (n Z).
6 6
Solve : 6 sin2 x – sin x cos x – cos2 x = 3.

Answer : + 2 n (n Z).
6
Sol. 6 sin2 x – sin x cos x – cos2 x –3 (sin2 x + cos2 x) = 0.
Solving Equation by Transforming a Sum
Removing the brackets and collecting like terms, we get
of Trigonometric Functions into a Product
3 sin2 x – sin x cos x – 4 cos2 x = 0.
Example – 16

 Solve : cos 3x + sin 2x – sin 4 x = 0


Since the values x = + n are not roots of the equation
2
Sol. cos 3x + (sin 2x – sin 4 x) = 0
and cos x  0, Transforming the expression in brackets by formula
 –  
we divide both sides of the equation by cos2 x sin  – sin  = 2 sin cos
2 2
we obtain
3 tan2 x – tan x –4 = 0,
cos 3 x + (–2 sin x cos 3 x) = 0,
cos 3x (1 – 2 sin x) = 0.
 The last equation is equivalent to the collection of
whence tan x = –1, x = – + n, n Z equation
4
1
cos 3x = 0, sin x = ;
2
4 4   
and tan x = , x = tan–1 + k, k Z consequently, x =  n, x = (–1)k + k (n, k  Z)
3 3 6 3 6

The set of solution x = (–1)k + k (k  Z) belongs
6
 4
Answer :  + n, tan–1 + k (n, k Z)  n
4 3 entirely to the set of solution x =  (n  Z).
6 3
Therefore, this set alone remains as a set of solutions.
 
Answer :  n (n Z).
6 3
88 TRIGONOMETRY

Solving Equations by Transforming a Product Solving Equation with the Use of


of Trigonometric Functions into a sum 1 + cos 2 = 2 cos21 – cos 2 = 2 sin2

Example – 17 Example – 19

Solve : sin 5 x cos 3x = sin 6 x cos 2x. x


Solve : cos x – 2 sin2 = 0.
2
1
Sol. We apply formula sin  cos  = (sin   sin Sol. cos x – (1– cos x) = 0  2 cos x –1 = 0
2
) to both sides of the equation : 1 
 cos x = x=± + 2n (nZ)
1 1 2 3
(sin 8 x + sin 2x) = (sin 8 x + sin 4x),
2 2
sin 2x – sin 4x = 0 
Answer :  + 2 n (n Z).
 –   3
Using formula sin  – sin  = 2 sin cos ,
2 2 Solving Equations with the Use of Formulas
we obtain –2 sin x cos 3x = 0. for Double & Triple Arguments

 x   n, n  Z, Example – 20
 sin x  0,  
 3x    k, x     k, k  Z.
cos3x  0,  2 6 3 x x
Solve : 2 sin cos2 x – 2 sin sin2 x = cos2 x – sin2 x.
2 2
 
Answer :  k (n, k Z).
6 3
Sol. On the left-hand side of the equation we put the factor
Solving Equ. with the Use of x
2 2
2 sin before the parentheses :
cos  = sin  = for Lowering a Degree 2

Example – 18 x
2 sin (cos2 x – sin2 x) = cos2 x – sin2 x.
2
Solve : sin2 x + sin2 2x = 1
Replacing the expression cos2 x – sin2 x by cos 2x
according to formula (2), we get
1– cos 2x 1– cos 4x
Sol.  1  cos 2x + cos 4 x = 0  2 cos
2 2 x
2 sin cos 2x = cos 2x,
3 x cos x = 0. 2
The last equation is equivalent to the collection of two
equations. x
or 2 sin cos 2x – cos 2x = 0
   2
(a) cos 3 x = 0, 3 x = + n, x =  n, n  Z
2 6 3 x
 
  cos 2x  2 sin – 1  = 0
(b) cos x = 0, x = + k, k  Z  2 
2
The set of solutions of equation (b) is a subset of the set   
of solutions of (a) and, therefore, in the answer we write  cos 2x  0,  x  4  2 n, n  Z,
only roots equation (a).   
sin x  1 ,  x   –1k   2k, k  Z.
 n  2 2 
Answer :  (n Z). 3
6 3
  
Answer :  n, (–1)k + 2k (n, k Z).
4 2 3
TRIGONOMETRY 89

Solving Equations by a Change of Variable Example – 22

(a) Equations of the form P (sin x ± cos x, sin x cos x) = 0, Solve : 3 cos x + 4 sin x = 5.
where P (y, z) is a polynomial, can be solved by the
change.
x x
cos x ± sin x = t 1 ± 2 sin x cos x = t2. 1– tan 2 2 tan
Sol. 3 2 4 2  5,
Let us consider an example. 2 x 2 x
1 tan 1 tan
Example – 21 2 2

Solve : sin x + cos x = 1 + sin x cos x. x x x


3 – 3 tan2 + 8 tan = 5 + 5 tan2 ,
2 2 2
Sol. We introduce the designation sin x + cos x = t. 2
x x  x 
Then (sin x + cos x)2 = t2, 1 + 2 sin x cos x = t2, 4 tan2 – 4 tan + 1 = 0,  2 tan –1  =0
2 2  2 
t 2 –1
sin x cos x = . x 1 1
2 tan  , x = 2 tan–1 + 2n, n Z
2 2 2
In the new designations the initial equation looks like
1
t 2 –1 Answer : 2 tan–1 + 2n, (n Z).
t=1+ or t2 – 2t + 1 = 0, (t –1)2 = 0, t = 1, 2
2
(c) Many equations can be solved by introducing a new
i.e.,
variable.
 1 1 
sin x + cos x = 1, 2 sin x  cos x   1, Solution of Trigonometric Eqn. of the Form
 2 2 

  1 f (x) =  (x)
cos cos x + sin sin x = ,
4 4 2
Example – 23
  2
cos  x –  
 4 2 Solve : 1 – cos x  sin x, x 

 
x –    2 n, n Z,
4 4 1– cos  0,
Sol. 
   sin x  0.
x    2 n, n Z.
4 4 Under the condition that both sides of the equation are
nonnegative, we square them:

Answer : + 2 n, 2 n (n Z). 1 – cos x = sin2 x, 1 – cos x = 1 – cos2 x,
2
cos2 x – cos x = 0, cos x (cos x –1) = 0.
(b) Equations of the form a sin x + b cos x + d = 0, where a,
b, and d are real numbers, and a, b 0, can be solved by 
(1) cos x = 0, x = + n, n Z,
the change. 2
(2) cos x = 1, x = 2k, k Z. But since sin x  0
x x
1– tan 2 2 tan 5
cos x  2 , sin x  2 , and x  we leave x = 2, .
2 x 2 x
2
1 tan 1 tan
2 2
Answer : 2, 5 .
x  + 2n (n Z) 2
90 TRIGONOMETRY

Solving Equations with the Use of the Trigonometric Systems


Boundedness of the Functions sin x & cos x
Example – 25
Example – 24

 x   x   1
Solve :  cos – 2sin x  sin x  1 sin – 2cos x   cos x  0. sin x cos y  ,
 4   4  Solve :  4
3tan x  tan y.

x x
Sol. cos sin x –2 sin2 x + cos x + sin cos x –2 cos2 x = 0.
4 4 Sol. We transform the second equation & get
3 sin x cos y – sin y cos x = 0.
 x 5x Substituting now the value of the product, sin x cos y
sin  x   + cos x – 2 (sin2 x + cos2 x) = 0, sin + cos x = 2.
 4 4 from the first equation into the equation obtained, we
get a system.
5x
Since the functions sin and cos x have the greatest  3
4
cos x sin y  4
5x  .......(1)
value equal to 1, their sum is equal to 2 if sin =1 and sin x cos y  1
4  4
cos x = 1 simultaneously, i.e.
Adding together the equations of system (1) and then
subtracting the first equation from the second, we get a
 5x  5x 
 sin  1,    2n, system which is equivalent to system :
 4  4 2
cos x 1,  x  2k  n, k  Z  ; sin  x  y  1,


 1 .......(2)
2 8 1 4n sin  x – y   ,
2k   n, k   2
5 5 5
whence we have
Since k  Z, it follows that n = 1 + 5m (m  Z), and then
x = 2  + 8m, m Z  
 x  y  2  2k,
Answer : 2  + 8m, m Z 
 x – y  –   2l .......(3)
 6

and

 
 x  y  2  2k,
 .......(4)
 x – y  – 5  2l
 6

From system (3) we find


 
x     k  l , y     k – l.
6 3
From system (4) we find
 2
x–    k  l , y    k – l.
6 3
91 TRIGONOMETRY

EXERCISE - 1 : BASIC OBJECTIVE QUESTIONS

Basic System of Measurement


2sin  1  sin   cos 
7. If   then is equal to
1. 10° 40’ 30’’ can be express in radians as follows 1  sin   cos  1  sin 

527  427 
(a) (b) 1
7200 7200 (a) (b) 

  (c) 1 –  (d) 1 + 
(c) (d)
7200 427
8. tan x is defined for all x in
2. The degree and radian measure of the angle between the
(a) R (b) R – {n: n I}
hour-hand and the minute-hand of a clock at twenty
minutes past seven is 
(c) R  {(2n  1) : n  I} (d) none of these
2
5 5
(a) (b)
4 8 9. cot x is defined for all x in
(a) R (b) R – {n: n I}
5 5 c
(c) (d)
7 9 
(c) R  {(2n  1) : n  I} (d) none of these
3. If the perimeter of a sector of a circle, of area 2
25 sq. cms. is 20 cms then area of a sector is
10. Which of the following is not correct ?
(a) 20 sq. cms (b) 24 sq. cms
(c) 50 sq. cms (d) 25 sq. cms 1
(a) sin    (b) cos  = 1
5
3
4. Number of sides of regular polygon of interior angle
4 1
(c) sec   (d) tan = 20
is 2

(a) 10 (b) 5 11. If sin  and cos  are the roots of the equation
(c) 8 (d) 9 ax2 – bx + c = 0, then a, b and c satisfy the relation :
5. The sum of two angles is 5 and their difference is 60°. (a) a2 + b2 + 2 ac = 0 (b) a2 – b2 – 2 ac = 0
Then the angles are (c) a2 + c2 – 2 ab = 0 (d) a2 – b2 + 2 ac = 0
(a) 480°, 420° (b) 470°, 450° 12. If x R and x 0, then which of the following is not
(c) 520°, 580° (d) 360°, 120° possible ?
Trigonometric Ratios
1 1
6. cos 24º + cos 5º + cos 175º + cos 204º + cos 300º = (a) 2sin   x  (b) 2cos   x 
x x
(a) 1/2 (b) –1/2
1 1
3 (c) 2sin   x  (d) sin   x 
(c) (d) 1 x x
2
TRIGONOMETRY
92 92
TRIGONOMETRY

13. In a trangle ABC, if cotA cotB cotC > 0, then the triangle is 20. If 3 sin – 5 cos = a, then 5 sin + 3 cos is equal to
(a) acute angled (b) right angled
(a) 1 – a (b) 1  a 2
(c) obtuse angled (d) does not exist
14. Which of the following is correct – (c) 34  a 2 (d) 34  a 2 or  34  a 2
(a) sin 1º > sin 1 (b) sin 1º < sin 1
 3
21. The value of tan tan is
 8 8
(c) sin 1º = sin 1 (d) sin1º  sin1
180
(a) 0 (b) 1
15. If sin (x –y) = cos (x + y) = 1/2, then the values of x and y
lying between 0º and 180º are given by 1
(c) (d) none of these
2
(a) x = 45º, y = 15º (b) x = 45º, y = 135º
(c) x = 165º, y = 15º (d) none of these 22. If tan  = – 4/3, then sin is

sec   tan  4 4 4 4
16. If 5 sin = 3, then is equal to (a) but not (b) or
sec   tan  5 5 5 5

4 4
1 (c) but not (d) none of these
(a) (b) 4 5 5
4
23. The value of cos 1º cos 2º cos 3º .... cos 179º is
(c) 2 (d) none of these
(a) 1/ 2 (b) 0
17. A value of satisfying cos + 3 sin = 2 is
(c) 1 (d) None of these
5 4
(a) (b) 24. If sinx + sin2x = 1, then the value of
3 3
cos 12x + 3cos10x + 3cos8x + cos6x – 1 is equal to

2  (a) 0 (b) 1
(c) (d)
3 3 (c) –1 (d) 2
18. Which of the following is correct ? Trigonometric Identities
(a) cos 1 > cos 2 (b) cos 1 < cos 2
1  tan 2 15
(c) cos 1 = cos 2 (d) none of these 25. The value of is
1  tan 2 15

1  sin A sin A 1
19. If   , for all permissible
1  sin A cos A cos A 3
(a) (b) 1
2
values of A, then A may belongs to
(a) First Quadrant (b) Second Quadrant 1
(c) (d) 3
(c) Third Quadrant (d) Fourth Quadrant 2
TRIGONOMETRY 93

17 5   3  
26. If sec A = and cosec B = ; if A, B lies in first quadrant 29. If f (x) = 3 sin 4   x   sin 4  3  x    2
8 4   2  

then sec (A + B) can have the value equal to


 6  6 
85 85 sin  2  x   sin  5  x   then, for all permissible
   
(a) (b) 
36 36
values of x, f (x) is

85 85 (a) – 1
(c)  (d)
84 84 (b) 0
(c) 1
27. The two legs of a right triangle are
(d) not a constant function
 3   3  30. The sines of two angles of a triangle are equal to
sin + sin     and cos – cos     . The
 2   2 
5 99
& . The cosine of the third angle can be :
length of its hypotenuse is 13 101

(a) 1 (b) 2 (Assume that sum of all angles in a triangle are


(c) 2 (d) some function of  supplementary)

28. Which of the following when simplified reduces to unity ? 245 255
(a) (b)
1313 1313

1  2 sin 2  735 765


(a) (c) (d)
    1313 1313
2 cot     cos 2    
4  4 
31. If tanx. tany = a and x + y = /6, then tanx and tany satisfy
the equation
sin     
(b)  cos      (a) x 2  3 1  a  x  a  0

sin   cos  tan
2
(b) 3x   1  a  x  a 3  0

2 (c) x 2  3 1  a  x  a  0
(c)
1

1  tan 2  
2 2 2
4 sin  cos  4 tan 
(d) 3x   1  a  x  a 3  0

1  sin 2  
(d) 2 32. If  = and  = , then tan  equals
 sin   cos   2

(a) tan  + 2 tan  (b) 2tan  + tan 


(c) tan  + tan  (d) none of these
94 TRIGONOMETRY

33. tan 5x tan 3x tan 2x = .....


 3 4
(a) tan 5x – tan 3x – tan 2x 39. If 0      , cos (  )  and cos (  ) 
4 5 5
sin 5x  sin 3x  sin 2x then sin 2is equal to
(b)
cos5x  cos3x  cos 2x
(a) 1 (b) 0
(c) 0
(c) 2 (d) none of these
(d) None of these
40. For all real values of , cot – 2 cot 2is equal to
m 1
34. If tan = , tan  = , then  is equal to (a) tan 2 (b) tan 
m 1 2m  1
(c) –cot 3 (d) none of these
(a) /4 (b) /3
41. If cos 20° – sin 20° = p then cos 40° is equal to
m 1 m 1
(c) tan (d) tan 1
2m  1 2m  1 (a)  p 2  p 2 (b) p 2  p 2

3
35. If  < 2 < , then 2  2  2cos 4 is equal to (c) p  2  p
2
(d) none of these
2

(a) –2cos (b) –2sin 42. If cos 2x + 2 cos x = 1 then sin2x (2–cos2x) is equal to
(c) 2cos (d) 2sin (a) 1 (b) –1

 (c)  5 (d)
36. If A – B = , then (1 + tan A) (1 – tan B) = 5
4
43. If A + C = B, then tan A tan B tan C is
(a) 1 (b) 2
(a) tan A tan B + tan C
(c) –1 (d) –2
(b) tan B – tan C – tan A
 
37. If tan and tan are the roots of the equation (c) tan A + tan C – tan B
2 2
8x2 – 26x + 15 = 0 then cos ( + ) is equal to (d) –(tan A tan B + tan C)

627 627 1 1
(a)  (b) 44. If x + = 2 cos , then x3 + 3 =
725 725 x x

(c) –1 (d) none of these (a) cos 3 (b) 2 cos 3

  1 1
38. If sin  + sin = a and cos – cos = b then tan is
2 (c) cos 3 (d) cos 3
2 3
equal to
45. If tan, tan are the roots of the equation
a b x2 + px + q = 0 (p  0), then
(a)  (b) 
b a
(a) sin () = –p (b) tan () = p/ (q – 1)
(c) a 2  b2 (d) none of these (c) cos () = 1 – q (d) none of these
TRIGONOMETRY 95

46. The value of Transformation of Product to Sum


51. sin x + sin 2x + sin 3x = 0 if
 3 5 7 9 11 13
sin .sin .sin .sin .sin .sin .sin (a) sin x = 1/2 (b) sin 2x = 0
14 14 14 14 14 14 14
is equal to (c) sin 3x = 3/2 (d) cos x = – 1/2

52. If (2n + 1)  = , then 2 n cos  cos 2  cos 2 2  ...


1
(a) 1 (b) cos 2n–1 =
16
(a) –1 (b) 1
1 (c) 1/2 (d) none of these
(c) (d) none of these
64 53. If n = 1, 2, 3, ..., then cos cos 2 cos 4 ... cos 2n–1 is
equal to
 5 7
47. The numerical value of sin .sin .sin is equal to
18 18 18 sin 2n  sin 2n 
(a) (b)
2n sin  2n sin 2n 1 
1
(a) 1 (b)
8
sin 4 n 1  sin 2 n 
(c) (d)
4 n 1 sin  2 n sin 
1 1
(c) (d) 54. tan 3 A – tan 2A – tan A is equal to
4 2
48. The value of (a) – tan 3 A tan 2 A tan A

cos 12° . cos 24° . cos 36° . cos 48° . cos 72° . cos 84° is (b) tan A tan 2 A tan 3 A
(c) tan A tan 2A – tan 2 A tan 3 A – tan 3 A tan A
1 1
(a) (b) (d) none of these
64 32
 3 5 7
55. cos cos cos cos is equal to
1 1 8 8 8 8
(c) (d)
16 128

49. The value of sin 78° – sin 66° – sin 42° + sin 6° is 1 2
(a) 1/2 (b)
2 2
1 1
(a) (b) 
2 2 1 2
(c) 1/8 (d)
(c) –1 (d) none of these 2 2

Maximum and Minimum Values


50. Let 0     and x = X cos  + Y sin , y = X
2 56. Minimum value of 5 sin2 + 4 cos2  is
sin – Y cos such that x2 + 4xy + y2 = aX2 + bY2, where a, (a) 1 (b) 2
b are constants. Then (c) 3 (d) 4
 57. Maximum value of sin x + cos x is
(a) a = –1, b = 3 (b)  
4 (a) 1 (b) 2

 1
(c) a = 3, b = –1 (d)   (c) 2 (d)
3 2
96 TRIGONOMETRY

58. Minimum value of sec2 + cosec2 is



(a) 2 (b) 4 66. The general solution of tan   = 0 is
2
(c) 1 (d) 3
(a) 2n n I (b) n n I
59. Minimum value of sin + cos6 is
6

(a) 0 (b) 1 
(c) (2n + 1) ; n I (d) None of these
2
1 1
(c) (d) 67. In 2 cos2  + 3sin  = 0, then the general value of is–
2 4

60. If A = 2 sin2 – cos 2, then A lies in the interval  


(a) n1n  n I (b) 2n   n I
6 6
(a) [–1, 3] (b) [1, 2]
(c) [–2, 4] (d) none of these 
(c) n1n+1  n I (d) None of these
61. If sin1 + sin2 + sin 3 = , then cos + cos2 + cos 3 = 6
(a) 3 (b) 2
68. General solution of equation 3 cos  + sin = 2 is
(c) 1 (d) 0
62. The maximum value of 12 sin  – 9 sin2 is  
(a) n±  ; n I
4 6
(a) 3 (b) 4
(c) 5 (d) None of these  
(b) 2n±  ; n I
63. The number of solutions of cos + 3 sin = 5, 0 5, is 4 6

(a) 4 (b) 0  
(c) 2n± – ; n I
(c) 5 (d) None of these 4 6

Trigonometric Equations (d) None of these


2
64. A solution of the equation cos + sin + 1 = 0, lies in the
tan 3x  tan 2x
interval 69. The set of values of x for which  1 is
1  tan 3x tan 2x
(a) (–/4, /4) (b) (/4, 3/4)
(a) 
(c) (3/4, 5/4) (d) (5/4, 7/4)
(b) { / 4}
65. If 4 sin2 = 1, then the values of are
(c) {n +  / 4 | n = 1, 2, 3.....}
  (d) {2n +  / 4 | n = 1, 2, 3.....}
(a) 2n   ,n  Z (b) n  , n  Z
3 3
70. The number of solutions of the equation
tan x + sec x = 2 cos x lying in the interval [0, 2], is
 
(c) n  , n  Z (d) 2n  , n  Z (a) 0 (b) 1
6 6
(c) 2 (d) 3
97 TRIGONOMETRY

EXERCISE - 2 : PREVIOUS YEAR JEE MAINS QUESTIONS

1. The number of solutions of tan x + sec x = 2 cos x in 8. In a triangle ABC, a = 4, b = 3, A = 60°, then c is the root
[0, 2) is (2002) of the equation. (2002)
(a) 2 (b) 3 (a) c2 – 3c – 7 = 0 (b) c2 + 3c + 7 = 0
(c) 0 (d) 1 (c) c2 – 3c + 7 = 0 (d) c2 + 3c – 7 = 0

 ABC A 5 C 2
2. In a triangle ABC, 2ca sin   is equal to 9. In a ABC, tan  , tan  , then (2002)
 2  2 6 2 5
(2002) (a) a, c, b are in AP (b) a, b, c are in AP
(a) a2 + b2 – c2 (b) c2 + a2 – b2 (c) b, a c are in AP (d) a, b, c are in GP
(c) b2 – c2 – a2 (d) c2 – a2 – b2
10. The equation a sin x + b cos x = c where | c |  a 2  b 2 has
4xy (2002)
3. sin 2   is true, if and only if (2002)
(x  y)2 (a) a unique solution
(a) x – y  0 (b) x = – y (b) infinite number of solutions
(c) x + y  0 (d) x  0, y  0 (c) no solution
(d) None of the above
1  tan 2 15
4. The value of is (2002) 
1  tan 2 15 11. If is a root of 25 cos2+ 5 cos – 12 = 0,    , then
2
(a) 1 (b) 3 sin 2is equal to (2002)

3 24 24
(c) (d) 2 (a) (b) 
2 25 25

4 13 13
5. If tan    , then sin is (2002) (c) (d) 
3 18 18
12. The sum of the radii of inscribed and circumscribed circles
4 4 4 4 for an n sided regular polygon of side a, is (2003)
(a)  but not (b)  or
5 5 5 5
 a   
4 4 (a) a cot   (b) cot  
(c) but not  (d) None of these n 2  2n 
5 5
   a   
1 (c) a cot   (d) cot  
6. If sin ( + ) = 1, sin ( –  )  , then tan  2n  4  2n 
2
(+ 2) tan (2+ ) is equal to 13. If in a triangle ABC
(2002)
C  A  3b
(a) 1 (b) –1 a cos 2    c cos 2    ,
2 2 2
(c) zero (d) None of these
2 2
then the sides a, b and c (2003)
7. If y = sin + cosec , 0, then (2002)
(a) are in AP (b) are in GP
(a) y = 0 (b) y 2
(c) are in HP (d) satisfy a + b = c
(c) y – 2 (d) y 2
TRIGONOMETRY
98 98
TRIGONOMETRY

14. In a triangle ABC, medians AD and BE are drawn. If  P Q


  19. In a triangle PQR, if R  . If tan   and tan   are
AD = 4, DAB  and ABE  , then the area of the 2 2 2
6 3 the roots of ax + bx + c = 0, a 0 then
2
(2005)
ABC is (2003)
(a) b = a + c (b) b = c
8 16 (c) c = a + b (d) a = b + c
(a) sq unit (b) sq unit
3 3 20. The number of value of x in the interval [0, 3] satisfying
the equation 2 sin2 x + 5 sin x – 3 = 0 is (2006)
32 64
(c) sq unit (d) sq unit (a) 4 (b) 6
3 3 3
(c) 1 (d) 2
3 21. A triangular park is enclosed on two sides by a fence and
15. The upper   th portion of a vertical pole subtends an on the third side by a straight river bank. The two sides
4
having fence are of same length x. The maximum area
3 enclosed by the park is (2006)
angle tan 1   at a point in the horizontal plane through
5 1 2
(a) x3 (b) x
its foot and at a distance 40m from the foot. A possible 2
height of the vertical pole is (2003) 8
(a) 20 m (b) 40 m 3 2
(c) x2 (d) x
(c) 60 m (d) 80 m 2
16. Let ,  be such that  <  –  < 3.
1
21 27 22. If 0 < x < and cos x + sin x = , then tan x is (2006)
If sin + sin =  and cos  + cos    , then the 2
65 65

value of cos
 
is (2004) (a) 1  7  (b)
4  7 
2 4 3
6 3
(a)
65
(b)
130 (c) 
4  7  (d)
1  7 
3 4
3 6 23. A tower stands at the centre of a circular park. A and B are
(c)  (d)
130 65 two points on the boundary of the park such that AB (=a)
subtends an angle of 60° at the foot of the tower and the
17. The sides of triangle are sin , cos and 1  sin  cos  angle of elevation of the top of the tower from A or B is 30°.
 The height of the tower is (2007)
for some 0    . Then the greatest angle of the triangle
2 2a
is (2004) (a) (b) 2a 3
3
(a) 120° (b) 90°
(c) 60° (d) 150° a
(c) (d) 3
18. A person standing on the bank of river observes that the 3
angle of elevation of the top of a tree on the opposite bank
of the river is 60° and when he retires 40 meters away from
the tree the angle of elevation becomes 30°. The breadth of
the river is (2004)
(a) 40 m (b) 30 m
(c) 20 m (d) 60 m
TRIGONOMETRY 99

24. AB is a vertical pole with B at the ground level and A at the 28. If A = sin2 x + cos4 x, then for all real x (2011)
top. A man finds that the angle of elevation of the point A
from a certain point C on the ground is 60°. He moves away 13
(a)  A 1 (b) 1  A  2
from the pole along the line BC to a point D such that 16
CD = 7 m. From D the angle of elevation of the point A is
45°. Then the height of the pole is (2008) 3 13 3
(c) A (d)  A 1
4 16 4
7 3 1  7 3 1 
(a)  m (b)  m 29. The possible values of   (0, ) such that
2  3 1 2  3 1  sin () + sin (4) + sin (7) = 0 are (2011)

7 3 7 3 2  4  3 8  5  2 3 8
(c) ( 3  1) m (d) ( 3  1) m (a) , , , , , (b) , , , , ,
2 2 9 4 9 2 4 9 4 12 2 3 4 9

25. Let A and B denote the statements 2   2 3 35 2   2 3 8


(c) , , , , , (d) , , , , ,
A : cos + cos + cos = 0 9 4 2 3 4 36 9 4 2 3 4 9
B : sin + sin + sin = 0 30. In a  PQR, if 3 sin P + 4 cos Q = 6 and
4 sin Q + 3 cos P=1, then the angle R is equal to (2012)
3
If cos (– ) + cos (– ) + cos (– )   , then
2 5 
(2009) (a) (b)
6 6
(a) A is true and B is false (b) A is false and B is true
 3
(c) both A and B are true (d) both A and B are false (c) (d)
4 4
4 5 31. ABCD is a trapezium such that AB and CD are parallel and
26. Let cos (+ ) = and let sin (– ) = , where 0 ,
5 13 BC  CD. If ADB = , BC = p and CD = q, then AB is
equal to (2013)

 . Then tan 2is equal to (2010)
4 (p 2  q 2 ) sin  p 2  q 2 cos 
(a) (b)
p cos   q sin  p cos   q sin 
25 56
(a) (b)
16 33
p2  q 2 (p 2  q 2 ) sin 
(c) (d)
19 20 p 2 cos   q 2 sin  (p cos   q sin ) 2
(c) (d)
12 7
tan A cot A
27. For a regular polygon, let r and R be the radii of the inscribed 32. The expression  can be written as
1  cot A 1  tan A
and the circumscribed circles. A false statement among the
following is (2010) (2013)
r 1 (a) sin A cos A + 1 (b) sec A cosec A + 1
(a) there is a regular polygon with 
R 2 (c) tan A + cot A (d) sec A + cosec A

r

1 33. Let fk (x)  1 (sin kx + cos kx) where x  R and
(b) there is a regular polygon with k
R 2
k 1. Then f4(x) – f6(x) equals : (2014)
r 2
(c) there is a regular polygon with  1 1
R 3 (a) (b)
12 6
r 3 1 1
(d) there is a regular polygon with 
R 2 (c) (d)
3 4
100 TRIGONOMETRY

34. A bird is sitting on the top of a vertical pole 20 m high and 40. If 0  x  2 , then the number of real values of x,
its elevation from a point O on the ground is 45°. It flies off which satisfy the equation
horizontally straight away from the point O. After one
second, the elevation of the bird from O is reduced to 30°. cosx + cos2x + cos3x + cos4x = 0, is : (2016)
Then the speed (in m/s) of the bird is : (2014) (a) 5 (b) 7
(c) 9 (d) 3
(a) 20 ( 3  1) (b) 40 ( 2  1)
41. The number of x  [0, 2] for which
(c) 40 ( 3  2) (d) 20 2
2 sin 4 x  18 cos2 x  2 cos4 x  18 sin 2 x = 1 is
35. The number of values of  in [0, 2] for which : (2016/Online Set–1)
2 sin3  - 7 sin2  + 7 sin  = 2, is:(2014/Online Set–1) (a) 2 (b) 4
(a) 6 (b) 4 (c) 6 (d) 8
(c) 3 (d) 1

42. If A > 0, B > 0 and A + B = , then the minimum value of
  6
36. If 2 cos  + sin   1    then 7 cos  + sin  is
 2 tanA + tanB is : (2016/Online Set–2)
equal to (2014/Online Set–2)
(a) 3 2 (b) 2  3
1
(a) (b) 2 2
2 (c) 4  2 3 (d)
3
11 46
(c) (d) 43. If 5(tan2 x – cos2 x) = 2cos 2x + 9, then the value of cos 4x
2 5 is (2017)
37. If the angles of elevation of the top of a tower from three
collinear points A, B and C on a line leading to the foot of 3 1
(a)  (b)
the tower, are 300, 450 and 600 respectively, then the ratio, 5 3
AB : BC, is: (2015)
2 
(a) 1: 3 (b) 2 : 3 (c) (d) 
9 9
(c) 3 :1 (d) 3: 2 44. Let a vertical tower AB have its end A on the level ground.
Let C be the mid-point of AB and P be a point on the
a ground such that AP = 2AB. If BPC = , then tan  is
38. In a ABC,  2    3 and C  60º . Then the
b equal to: (2017)
ordered pair (A, B) is equal to : (2015/Online Set–1) 6 1
(a) (45º, 75º) (b) (75º, 45º) (a) (b)
7 4
(c) (105º, 15º) (d) (15º, 105º)
2 4
3 1 (c) (d)
39. If cos  + cos   and sin  + sin   and  is the 9 9
2 2
45. If sum of all the solutions of the equation
arithmetic mean of  and  , then sin 2  + cos 2  is
     1
equal to: (2015/Online Set–2) 8cos x.  cos   x  .cos   x     1 in  0,  is k,
 6  6  2
3 7 then k is equal to : (2018)
(a) (b)
5 5
20 2
4 8 (a) (b)
9 3
(c) (d)
5 5
13 8
(c) (d)
9 9
TRIGONOMETRY 101

46. If tanA and tanB are the roots of the quadratic equation, 49. A tower T1 of height 60m is located exactly opposite to a
3x 2 - 10x - 25 = 0, then the value of tower T2 of height 80m on a straight road. From the top of
3 sin 2 (A + B) –10sin(A + B).cos(A + B) –25 T1, if the angle of depression of the foot of T2 is twice the
cos2(A +B) is : (2018/Online Set–1) angle of elevation of the top of T2, then the width (in m) of
(a) -10 (b) 10 the road between the feet of the towers T1 and T2 is :
(c) -25 (d) 25 (2018/Online Set–2)
47. An aeroplane flying at a constant speed, parallel to the (a) 10 2 (b) 10 3
horizontal ground, 3 km above it, is observed at an
0
(c) 20 3 (d) 20 2
elevation of 60 from a point on the ground. If, after five
seconds, its elevation from the same point, is 300, then the 50. If an angle A of a  ABC satisfies
speed (in km/hr) of the aeroplane, is :
5 cos A + 3 = 0, then the roots of the quadratic equation,
(2018/Online Set–1)
9x2 + 27x + 20 = 0 are : (2018/Online Set–3)
(a) 1500 (b) 1440
(a) sec A, cot A (b) sin A, sec A
(c) 750 (d) 720
(c) sec A, tan A (d) tan A, cos A
48. The number of solutions of sin 3x = cos 2x, in the interval
51. A man on the top of a vertical tower observes a car moving
  at a uniform speed towards the tower on a horizontal road.
 ,   is : (2018/Online Set–2) If it takes 18 minute for the angle of depression of the car
2 
to change from 300 to 450; then after this, the time taken (in
(a) 1 (b) 2 minute) by the car to reach the foot of the tower, is :
(c) 3 (d) 4 (2018/Online Set–3)


(a) 9 1 + 3  
(b) 18 1 + 3 
9
(c) 18  3 -1  (d)
2
 3 -1 
102 TRIGONOMETRY

EXERCISE - 3 : ADVANCED OBJECTIVE QUESTIONS

7. cos2 A (3 – 4 cos2 A)2 + sin2 A (3 – 4 sin2 A)2 =


3 A 5A
1. If cos A = then the value of sin sin is
4 2 2 (a) cos 4A (b) sin 4A

(a) 1/32 (b) 11/8 (c) 1 (d) None of these

(c) 11/32 (d) 11/16 8. 2 sin2  + 4 cos () sin  sin  + cos 2 () =

2. sin (sin  + sin 3) is (a) sin 2 (b) cos 2 


(a)  0 for all  (b) 0 only when   0 (c) cos 2 (d) sin 2
(c)  0 for all  (d) 0 only when   0 9. If sin, sin  and cos are in G.P. then roots of the equation
3. Given A = sin2 + cos4, then for all real , x2 + 2x cot  + 1 = 0 are always
(a) equal (b) real
3
(a) 1  A  2 (b)  A  1 (c) imaginary (d) greater than 1
4
10. If in a triangle ABC, sin2A + sin2B + sin2C = 2, then the
13 3 13
(c)  A 1 (d) A . triangle is always
16 4 16
(a) isosceles triangle (b) right angled
2cos   1   (c) acute angled (d) obtuse angled
4. If cos = , (0 <  < ,0 <  < ), then tan cot
2  cos  2 2
11. If sin = 3sin ( + 2), then the value of tan ( + ) + 2 tan
is equal to  is

(a) 1 (b) 2 (a) 3 (b) 2


(c) 1 (d) 0.
(c) 3 (d) none of these
12. The least value of cos2  – (6 sin  . cos ) + 3 sin2 + 2 is
5. If sin () = 1, sin ( – ) = 1/2, where   [0, /2), then
tan ( + 2) tan(2) is equal to (a) 4  10 (b) 4  10

(a) 1 (b) 0 (c) 0 (d) 4


(c) –1 (d) none of these
6. Which of the following statements are possible with    
13. If sin  + sin  = a and cos  – cos  = b, then tan   is
 2 
a, b, m and n being non-zero real numbers :
(a) 4 sin2 = 5 equal to
(b) (a2 + b2) cos  = 2ab
a b
(c) (m2 + n2) cosec = m2 – n2 (a)  (b) 
b a
(d) none of these
(c) a 2  b2 (d) None of these
TRIGONOMETRY 103

1    2   4  1
14. sin   sin 3      sin 3      is equal to 20. If the equation sin  = – 1/2 and tan  = , then most
sin 3   3   3  3
common general values of  is
4 3
(a) (b)
3 4 7 7
(a) 2n ± (b) 2n –
6 6
3
(c)  (d) none of these
4 7
(c) 2n + (d) None of these
15. The number of distinct solution of 6

21. If r > 0, –     and r,  satisfy r sin   = 3 and


 
sin 5. cos 3= sin 9. cos 7in 0,  is r = 4 (1 + sin ), then the number of possible solutions of
 2
the pair (r, ) is
(a) 4 (b) 5
(a) 2 (b) 4
(c) 8 (d) 9
(c) 0 (d) inifinite
16. The value of satisfying

3 cos2  – 2 3 sin cos – 3 sin2 = 0 are  5  


22. If x   – ,  ,  the greatest positive solution of
 2 2
2    1 + sin4 x = cos2 3x is
(a) n – , n  (b) n – , n 
3 6 3 6
(a)  (b) 2
  (c) 5/2 (d) none of these
(c) 2n – , n (d) 2n , n
3 3 23. The number of real solutions of
17. If the equation tan  + tan 2 + tan  tan 2 =1, then is sin ex . cos ex = 2x–2 + 2–x–2 is
equal to
(a) zero (b) one
n  n  (c) two (d) infinite
(a) – (b) 
3 6 3 12
24. All solutions of the equation, 2 sin + tan = 0 are obtained
n  by taking all integral values of m and n in :
(c)  (d) None of these
3 2
2 2
18. The equation k cos x –3 sin x = k + 1 is solvable only if k (a) 2n  (b) n & 2m 
3 3
belongs to the interval
(a) [k, + ] (b) [–4, 4]  
(c) n & m  (d) n & 2m 
(c) (–, 4] (d) None of these 3 3
19. If 2 sin x + 1  0 and x  [0, 2], then the solution set for x is
 
25. If x  0,  , the number of solutions of the equation,
 7   7  11   2
(a) 0,  (b) 0,    , 2 
 6  6   6 
sin 7x + sin 4x + sin x = 0 is :
(a) 3 (b) 5
11 
(c)  , 2  (d) None of these
 6  (c) 6 (d) None
104 TRIGONOMETRY

26. The number of solution in [0, /2] of the equation 33. cos 15 x = sin 5x if
cos 3x tan 5x = sin 7x is
   
(a) 5 (b) 7 (a) x    n (b) x   n
20 5 40 10
(c) 6 (d) none of these
3  3 
27. sin x – cos2 x – 1 assumes the least value for the set of (c) x   n (d) x    n
20 5 40 10
values of x given by :
34. sin2x + 2 sin x cos x – 3cos2x = 0 if
(a) x = n + (–1)n + 1 (/6)
(a) tan x = 3 (b) tan x = –1
(b) x = n + (–1)n (/6)
(c) x = n +  / 4 (d) x = n + tan–1 (–3)
(c) x = n + (–1)n (/3)
35. 5 sin2 x + 2
3 sin x cos x + 6cos x = 5 if
(d) x = n – (–1)n (/6) where n  Z
28. If 2 cos2( + x) + 3 sin ( + x) vanishes then values of x lying (a) tan x = –1/ 3 (b) sin x = 0
in the interval from 0 to 2  are (c) x = n +  / 2 (d) x = n +  / 6
(a) x = /6 or 5 /6 (b) x = /3 or 5 /3 36. sin x + sin 2x + sin 3x = cos x + cos 2x + cos 3x if
(c) x = /4 or 5 /4 (d) x = /2 or 5 /2 (a) cos x = – 1/2 (b) sin 2x = cos 2x
29. 4 sin4x + cos4x = 1 if (c) x = n/ 2 + / 8 (d) x = 2n  ± 2 / 3 (n  I)

37. If sin x + 7 cos x = 5 then cos (x – ) = 1/ 2 if


1 2
(a) x = n (b) n  sin
5
(a)  cos –1 7 / 50 (b)  sin –1 1/ 50
(c) x = n/2 (d) none
(c)  cos –1 1/ 7 (d)  sin –1 5/ 7
30. The general solution of , sin x + sin 5x = sin 2x + sin 4x is :
38. If 6 cos 2 + 2 cos2 / 2 + 2 sin2  = 0, –  <  < , then
(a) 2n ; n  1 (b) n ; n  1
=
(c) n ; n  1 (d) none
(a) /3 (b)  /3, cos–1 (3/5)
31. General solution of the equation, cot 3 – cot  = 0 is
(c) cos–1 (3/5) (d)  /3,  – cos–1 (3/5)
  39. If sin4x + cos4y + 2 = 4 sin x cos y, and 0  x, y /2 then
(a)   (2n  1) (b)   (2n  1)
2 4 sin x + cos y is equal to :
(a) – (b) 0
 (c) 2 (d) none of these
(c)   (2n  1) (d) none
3
40. If sin + 7 cos = 5, then tan (/2) is a root of the equation
sin 3 1 (a) x2 – 6x + 1 = 0 (b) 6x2 – x – 1 = 0
32.  if
2cos 2   1 2 (c) 6x2 + x + 1 = 0 (d) x2 – x + 6 = 0

  41. The value of cos y cos (/2 –x) – cos (/2 –y)
(a)   n   (b)   2n   cos x + sin y cos (/2 –x) + cos x sin (/2 –y) is zero if
6 6
(a) x = 0 (b) y = 0

(c)   n  (1) n  (d)   n   (c) x = y + /4 (d) y = x – 3/4
6 6
TRIGONOMETRY 105

42. tan (p / 4) = cot(q/4) if


sin 3   cos3  cos 
47.  – 2 tan cot = – 1 if :
(a) p + q = 0 sin   cos  1  cot 2 
(b) p + q = 2n + 1
   
(c) p + q = 2n (a)   0,  (b)   ,  
 2 2 
(d) p + q = 2 (2n + 1) where n is any integer
3  3 
43. If cos x – sin x  1 and 0  x  2then the solution set for x (c)    ,  (d)    , 2 
 2   2 
is
48. The least positive values of x satisfy the equation
    7   3 7  2 x  |cos 3 x |  ...... 
= 43 will be (where |cos x| < 1)
(a) 0,    , 2 (b)  ,   {0} 81  |cos x| cos
 4   4  2 4
 2
(a) (b)
3 3
 3 
(c)  , 2   {0} (d) none of these
2  
(c) (d) None of these
4
44. If |tan x| 1 and x [–, ] then the solution set for x is
49. If m and n (> m) are positive integers, the number of
solutions of the equation n |sin x| = m | cos x| in [0, 2] is
(a)  ,  3      ,     3 ,  
 4   4 4   4  (a) m (b) n

(c) mn (d) none of these

  3
(b)   ,    ,     
 4 4  4  50. The number of values of  in the interval   , 
     2 2

sec 2 

(c)   ,
 satisfying the equation  3 = tan4 + 2 tan2 is
 4 4 

(a) 1 (b) 2
(d) none of these (c) 3 (d) none of these
51. If k1 = tan 27– tan and
45. If 4 sin2x – 8 sin x + 3  0, 0  x  2, then the solution set for
x is sin  sin 3 sin 9
k2    , then
cos3 cos9 cos 27
   5  (a) k1 = 2k2 (b) k1 = k2 + 4
(a)  0,  (b) 0, 
 6  6
(c) k1 = k2 (d) none of these

 5 7
 5    5  52. If a = sin sin sin , and x is the solution of the
(c)  , 2  (d)  ,  18 18 18
6  6 6 
equation y = 2 [x] + 2 and y = 3 [x – 2], where [x] denotes the
integral part of x, then a is equal to
46. cos 4x cos 8x – cos 5x cos 9x = 0 if
(a) cos 12x = cos 14x (b) sin 13x = 0 1
(a) [ x ] (b)
[x]
(c) sin x = 0 (d) cos x = 0
(c) 2 [ x ] (d) [ x ]2
106 TRIGONOMETRY

57. If in  ABC if A > B, tan A + tan B + tan C = 6 and


xy
53. If sin x + cos y = a and cos x + sin y = b, then tan is tan A tan B = 2, then sin2 A : sin2 B : sin2 C is
2
(a) 8 : 9 : 5 (b) 8 : 5 : 9
equal to
(c) 5 : 9 : 8 (d) 5 : 8 : 5
(a) a + b (b) a – b
2
ab ab  sin   tan 
(c) (d) 58. If     3, then
ab ab  sin   tan 

54. If cot (– ), 3 cot , cot (+ ) are in A.P., and is not an
(a) tan = 1/ 3 (b) tan = – 1/ 3

integral multiple of , then sin cosec is equal to :
2 (c) tan = 3 (d) tan = – 3

3 
59. Let f n     tan (1 + sec ) (1 + sec 2)
(a)  2 (b)  2
2
(1 + sec 4)............ (1 + sec 2n ), then
2
(c)  (d) none of these   
3 (a) f 2    1 (b) f3    1
 16   32 
55. Which of following functions have the maximum value
unity?     
(c) f 4    1 (d) f5   1
(a) sin2x – cos2x  64   128 

60. In a triangle ABC


6 1 1 
(b) sin x  cos x 
5  2 3  3 3
(a) sin A sin B sin C 
8
(c) cos6 + sin6 x
(d) cos2x + sin4x 9
(b) sin2 A + sin2 B + sin2 C 
56. If sin + sin = a and cos + cos = b, then 4

(c) sin A sin B sin C is always positive


 1
(a) cos   (a 2  b2 ) (d) sin2 A + sin2 B = 1 + cos C
 2  2

sin 3 x cos3 x
  1 61. The value(s) of the expression  are
(b) cos   (a 2  b 2 ) 1  cos x 1  sin x
 2  2

   
(a) 2 cos   x  (b) 2 cos   x 
  4  a 2  b2  4  4 
(c) tan      2 2 
 2   a b 

   
(c) 2 sin   x  (d) 2 sin   x 
a 2  b2  2  4   4 
(d) cos (  ) 
2
TRIGONOMETRY 107

62. The equation sin 4x = a tan x, a > 0 66. Assertion : The function f (x) = min {sin x, cos x} takes the
(a) has no solution other than nif a > 4 4 20 43
value twice when x varies from to .
(b) has solutions which are not of the type nif 0 < a < 4 5 3 6
Reason : The periods of sin x and cos x are equal to 2.
(c) has solutions if a = 2009
(a) A (b) B (c) C
(d) has no solutions if a = 2008
(d) D (e) E
1
63. The equation |cot x| = cot x + , (n  Z) 67. Assertion : The inequation tan x < 3 is equivalent to
sin x
1
cot x > .
2 3
(a) has a general solution (3n + 1)
3
1 1
Reason : If a < b then  .
2 a b
(b) has a general solution (3n – 1)
3 (a) A (b) B (c) C
(c) is not defined if x = n (d) D (e) E
68. Assertion : The equation cos x + cos 2009 x = – 2 has
(d) cannot have a solution if cot x is positive
infinite solution.
Assertion Reason Reason : 2009 is an odd integer.
(A) If ASSERTION is true, REASON is true, REASON is a (a) A (b) B (c) C
correct explanation for ASSERTION. (d) D (e) E
(B) If ASSERTION is true, REASON is true, REASON is Using the following passage, solve Q.69 to Q.71
not a correct explanation for ASSERTION.
Passage –1
(C) If ASSERTION is true, REASON is false Given cos 2 cos 2
m m+1 n
.............. cos 2 
(D) If ASSERTION is false, REASON is true
sin 2 n 1  m
(E) If both ASSERTION and REASON are false.  n  m 1
, where 2 k, n, m, k I
2 sin 2 m 
  Solve the following :
64. Assertion : If 2 sin  1  sin   1  sin  , then lies
2 2
69. sin 9 . sin
11 sin 13

 3 14 14 14
between 2n+ and 2n+
4 4
1 1
(a) (b) 
  3  64 64
Reason : If runs from to , then sin >0
2 4 4 2 1 1
(c) (d) 
(a) A (b) B (c) C 8 8

(d) D (e) E    
70. cos 23 cos 24 cos25 ........ cos 210 =
10 10 10 10
65. Assertion : The numbers sin 18° and – sin 54° are the roots
of same quadratic equation with integer co-efficients.
1 1
Reason : If x = 18°, then 5x = 90°, if y = – 54°, then (a) (b)
128 256
5y = –270°
(a) A (b) B (c) C 1  5 1 3
(c) sin (d) sin
512 10 512 10
(d) D (e) E
108 TRIGONOMETRY

76. Match the following for the trigonometric equation



71. cos cos 2 cos 3 .... cos 11 
11 11 11 11 1 1
x  x  = cos x, (n is an integer) :
4 4
1 1
(a)  (b)
32 512 Column - I Column - II

1 1  1 1   1 
(c) (d)  (A) Over  ,   (P)    2n  , n  0 
1024 2048  4 3  3 

Using the following passage, solve Q.72 to Q.74


 1 1  2  2 
Passage-2 (B) Over   ,  (Q)     2n  , n  0 
 4 4  3  3 
If Pn = sinn + cosn where n  W (whole number)
and R (real number)
1 
72. If P1 = m, then the value of 4 (1 – P6) is (C) Over  ,   (R) No solution
4 
(a) 3 (m – 1)2 (b) 3 (m2 – 1)2
Subjective
(c) 3 (m + 1)2 (d) 3 (m2 + 1)2
77. A solution of the equation sin 5x + sin x + 2 sin2 x = 1 lying
73. The value of 2P6 – 3P4 + 10 is
(a) 0 (b) 6  1 
in the interval  0,  is then  must be
(c) 9 (d) 15  5 
2 2
74. If Pn – 2 – Pn = sin cos P, then the value of is 78. The smallest positive angle (in degrees) satisfying
(a) n – 1 (b) n – 2 tan (x + 100°) = tan (x + 50°) tan x tan (x – 50°) must be
(c) n – 3 (d) n – 4 79. If k is a positive integer, such that
Match the column
7
75. Column - I Column - II (i) cos2 x sin x   , for all x
k

2 1 1
(A) cos 52  sin 2 22 (P) 1 7
2 2 (ii) cos2 x sin x   for some x, then k must be equal
k 1

3 4 3 3 to
(B) cos2  cos 2 (Q)
5 5 4 2 80. The value of (1 – cot 23°) (1 – cot 22°) must be equal to

3
(C) sin 24° + cos 6° (R)
4

2 2 15  3
(D) sin 50 + cos 130° (S)
4
TRIGONOMETRY 109

EXERCISE - 4 : PREVIOUS YEAR JEE ADVANCED QUESTIONS

Single Answer Question 5. The general solution of the trigonometric equation


sin x + cos x = 1 is given by :
4
1. If tan    , then sin is (1978) (1981)
3
(a) x = 2n; n = 0, ±1, ±2.......
4 4 4 4 (b) x = 2n; n = 0, ±1, ±2.......
(a)  but not (b)  or
5 5 5 5
 
(c) x = n + (–1)n – ; n =0, ±1, ±2.....
4 4 4 4
(c) but not  (d) None of these
5 5 (d) none of these
2. If + + = 2, then (1979)   3  5   7 
 
6. 1  cos   1  cos  1  cos   1  cos 
       8  8   8  8 
(a) tan  tan  tan  tan tan tan
2 2 2 2 2 2 is equal to (1984)
      1 
(b) tan tan  tan tan  tan tan  1 (a) (b) cos
2 2 2 2 2 2 2 8

      1 1 2
(c) tan  tan  tan   tan tan tan
2 2 2 2 2 2 (c) (d)
8 2 2
(d) None of the above
 3 
3. Given A = sin2  + cos4 , then for all real values of  : 7. The expression 3 sin 4      sin 4  3    
(1980)   2  
3
(a) 1  A  2 (b)  A 1    
4 2 sin 6      sin 6  5     is equal to (1986)
  2  
13 3 13 (a) 0 (b) 1
(c)  A 1 (d) A
16 4 16
(c) 3 (d) sin 4 + cos 6
1   8. The number of all possible triplets (a1, a2, a3) such that
4. The equation 2 cos2  x  sin2 x = x2 + x–2, x  has : a1 + a2 cos (2x) + a3 sin2 (x) = 0 for all x is : (1987)
2  9
(1980) (a) zero (b) one
(a) no real solution (c) three (d) inifinite
(b) one real solution 9. The smallest positive root of the equation, tan x – x = 0 lies
in : (1987)
(c) more than one real solution
(d) none of these    
(a)  0,  (b)  ,  
 2 2 

3  3
(c)  ,  (d)  ,2 
 2   2 
110 TRIGONOMETRY

The value of the expression 18. The number of values of x in the interval [0, 5] satisfying
10. 3 cosec 20º – sec 20º is
the equation 3 sin2 x –7 sin x +2 = 0 is : (1998)
equal to : (1988)
(a) 0 (b) 5
(a) 2 (b) 2 sin 20º/ sin 40º
(c) 6 (d) 10
(c) 4 (d) 4 sin 20º/ sin 40º
19. Which of the following numbers is rational ? (1998)
11. The general solution of sin x –3 sin 2x + sin 3x
= cos x –3 cos 2x + cos 3x is : (1989) (a) sin 15º (b) cos 15º
(c) sin 15º cos 15º (d) sin 15º cos 75º
 n 
(a) n + (b) 
8 2 8 
20. If     and      , then tan  equals : (2001)
2
n  3
(c) (–1)n  (d) 2n + cos–1 (a) 2 (tan  + tan ) (b) tan  + tan 
2 8 2
(c) tan  + 2 tan  (d) 2 tan  + tan 
12. The equation (cos p –1) x2 + (cos p) x + sin p = 0 in the
variable x, has real roots. Then p can take any value in the 21. The number of integral values of k for which the equation
interval : (1990) 7 cos x + 5 sin x = 2k + 1 has a solution, is : (2002)
(a) (0, 2) (b) (–, 0) (a) 4 (b) 8
(c) 10 (d) 12
  
(c)  – ,  (d) (0, ) 1 1
 2 2 22. Given both  and  are acute angles sin   , cos   ,
2 3
13. In a triangle ABC, angle A is greater than angle B. If the
then the value of  +  belongs to (2004)
measures of angles A and B satisfy the equation
3 sin x –4 sin3 x –k = 0, 0 < k < 1, then the measure of angle      2 
C is : (1990) (a)  ,  (b)  , 
 3 6 2 3 
(a) /3 (b) /2
(c) 2/3 (d) 5/6  2  5   5 
(c)  ,  (d)  ,  
14. The number of solutions of the equation  3 6  6 
sin (e)x = 5x + 5–x is : (1991) 23. The number of ordered pairs (), where  (–)
(a) 0 (b) 1 1
(c) 2 (d) infinitely many satisfying cos ( – =1 and cos       is :
e
15. Number of solution of the equation (2005)
tan x + sec x = 2 cos x lying in the interval [0, 2] is:
(a) 0 (b) 1
(1993)
(c) 2 (d) 4
(a) 0 (b) 1
24. The set of value of  satisfying the inequation
(c) 2 (d) 3
2 sin2 – 5 sin  +2 > 0, where 0 < < 2 is: (2006)
16. The general value of  satisfying the equation
2 sin2  – 3 sin –2 = 0, is : (1995)     5      5 
n n (a)  0,    ,2  (b) 0,    , 2 
(a) n+ (–1) /6 (b) n+ (–1) /2  6  6   6  6 
(c) n+ (–1)2 5/6 (d) n+ (–1)n 7/6
    2 
17. 3 (sin x – cos x)4 + 6 (sin x + cos x)2 + 4 (sin 6 x + cos6 x) equal (c) 0,    , 2  (d) none of these
to : (1995)  3   3 
(a) 11 (b) 12
(c) 13 (d) 14
TRIGONOMETRY 111

13
 1
25. Let   0,  and t 1 = (tan ) cot  , t 2 = (tan ) cot  , 31. The value of     k 1    k 
is equal to
 4 k 1
sin   sin   
 6    6 
t3 = (cot )tan and t4 = (cot )cot , then (2006)
(a) t1 > t2 > t3 > t4 (b) t4 > t3 > t1 > t2 (2016)

(c) t3 > t1 > t2 > t4 (d) t2 > t3 > t1 > t4 (a) 3  3 (b) 2  3  3 
26. The number of solutions of the pair of equations
2sin2 – cos 2 =0 & 2cos2 – 3sin =0 in the interval (c) 2  3 1 
(d) 2 2  3 
[0, 2] is (2007) Multiple Answer Question
(a) zero (b) one 32. The values of  lying between  = 0 and  = /2 and
(c) two (d) four satisfying the equation

27. Let P = {: sin – cos = 2 cos } and 1  sin 2  cos 2  4sin 4
2 2
Q = {: sin + cos = 2 sin } be two sets. Then sin  1  cos  4sin 4  0, is (1988)
(2011) sin 2  2
cos  1  4 sin 4
(a) P Q and Q – P  (b) Q P (a) 7/24 (b) 5/24
(c) P Q (d) P = Q (c) 11/24 (d) /24
28. In a triangle the sum of two sides is x and the product of the
 
same two sides is y. If x2 – c2 = y, where c is the third side of
33. For 0 < < /2, if x   cos 2n , y   sin 2n ,
the triangle, then the ratio of the in-radius to the circum- n 0 n 0
radius of the triangle is (2014)

3y 3y z   cos2n sin2n , then (1993)


(a) 2x(x + c) (b) 2c(x + c) n 0

(a) xyz = xz + y (b) xyz = xy + z


3y 3y (c) xyz = x + y + z (d) xyz = yz + x
(c) 4x(x + c) (d) 4c(x + c)
4xy
34. sec2   is true if and only if (1996)
29. For x  (0, ) , then eqation sin x + 2 sin 2x – sin 3x = 3 has (x  y) 2
(2014) (a) x = y  0 (b) x = y, x  0
(a) infinitely many solutions (b) three solutions (c) x = y (d) x  0, y  0
(c) one solution (d) no solution 35. For a positive integer n, let
   
30. Let S  x   :x  0,  . The sum of all distinct f n ()   tan  (1  sec ) (1  sec 2)
  2

solution of the equation 3 sec x + cosec x + 2 (tan x – cot
(1 + sec 22 )... (1 + sec 2n), then (1999)
x) = 0 in the set S is equal to (2016)
  
 2 (a) f 2    1 (b) f 3    1
(a)  (b)   16   32 
 

5    
(c)  (d) (c) f 4    1 (d) f5   1
  64   128 
112 TRIGONOMETRY

40. In a triangle XYZ, let x, y, z be the lengths of sides opposite


sin 4 x cos 4 x 1
36. If   , then (2009) to the angles X, Y, Z, respectively, and 2s = x + y + z. If
2 3 5
s x s y sz
  and area of incircle of the triangle XYZ
2 sin 8 x cos8 x 1 4 3 2
(a) tan 2 x  (b)  
3 8 27 125 
is  then (2016)
8 8

1 sin x cos x 2
(c) tan 2 x  (d)   (a) area of the triangle XYZ is 6 6
3 8 27 125

 the 35
37. For 0 , solution(s) of (b) the radius of circumcircle of the triangle XYZ is 6
2 6
6
(m  1)   m  X Y Z 4
  (c) sin sin sin 
 cos ec   
m 1 4
 cos ec   
 
  4 2 is/are
4 
2 2 2 35

2  X  Y 3
(2009) (d) sin  
2  5
  41. Let  and  be non zero real numbers such that 2(cos  –
(a) (b)
4 6 cos ) + cos  cos  = 1. Then which of the following is/
are true? (2017)
 5
(c) (d)
12 12  
(a) tan    3 tan    0
38. Let , [0, 2] be such that 2 cos (1 – sin ) = sin2 2 2

    
 tan  cot  cos   1, tan (2 – ) > 0 and (b) 3 tan    tan    0
 2 2 2
  2
3
1  sin    . Then, cannot satisfy (2012)  
2 (c) tan    3 tan    0
2 2
  4
(a) 0    (b)   
2 2 3 (d) 3 tan    tan    0
2 2
4 3 3 Match the Column
(c)  (d)    2
3 2 2 42. (sin 3)/(cos 2) is (1992)
1 Column I Column II
39. In a PQR, P is the largest angle and cos P  . Further in
3 (A) positive (p) (13/48, 14/48)
circle of the triangle touches the sides PQ, QR and RP at N, (B) negative (q) (14/48, 18/48)
L and M respectively, such that the lengths of PN, QL and (r) (18/48, 23/48)
RM are consecutive even integers. Then, possible length(s)
of the side(s) of the triangle is (are) (s) (0, /2)
(2013)
(a) 16 (b) 18
(c) 24 (d) 22
TRIGONOMETRY 113

Integer Answer Type Question 53. General value of  satisfying the equation
43. The number of all possible values of , where 0 < < , for tan2 + sec 2=1 is.... . (1996)
which the system of equations True/False
(y + z) cos 3= (xyz) sin 3 54. If tan A = ( 1 – cos B) / sin B, then tan 2 A = tan B.
(1983)
2cos 3 2sin 3
x sin 3   55. There exists a value of  between 0 and 2that satisfies the
y z
equation sin4– 2 sin2 – 1 = 0. (1984)
and (xyz) sin 3 = (y + 2z) cos 3y sin 3have a solution Subjective Question
(x0, y0, z0) with y0z0 0, is ..... (2010)
56. Solve 2 (cos x + cos 2x) + (1 + 2 cos x) sin 2x
44. The number of distinct solution of the equation
= 2 sin x, – x  (1978)
5
cos2 2x + cos4 x + sin4 x + cos6 x + sin6 x = 2 in the interval  
4 57. Prove that 5cos   3cos     + 3 lies between –4 and 10.
 3
[0, 2] is (2015)
Analytical and Descriptive Questions (1979)

45. Prove that sin x . sin y. sin (x–y) + sin y. sinz.sin (y–z) 58. Given  +  +  = , prove that sin  + sin  – sin2
2 2

 = 2 sin  sin  cos . (1980)


+ sin z. sin x . sin (z–x) + sin (x–y) . sin (y–z) . sin (z–x) = 0
59. For all  in [0, /2], show that cos (sin ) sin (cos ).
(1978)
(1981)
4 5 60. Find the coordinates of the points of intersection of the
46. If cos (  )  , sin (  )  and , lie between 0
5 13
 
and /4, find tan 2. (1979) curves y = cos x, y = sin 3x, if   x . (1982)
2 2
n
61. Show that the equation esin x – e–sin x – 4 = 0 has no real
47. Suppose sin3 x sin 3 x   Cm cos mx is an identity in x,
m 0
solution. (1982)
where C0, C1, ..., Cn are constants and Cn  0. Then the value 62. Without using tables, prove that
of n is ..... (1981)
1
Fill in the blanks (sin 12º) (sin 48º) (sin 54º) = . (1982)
8

48. The larger of cos (log ) and log (cos ) if e  / 2    , is  2   4   8   16 
2 63. Show that 16 cos   cos   cos   cos   1.
....... (1983)  15   15   15   15 

49. The solution set of the system of equations (1983)


2 2
3 64. Find all the solutions of 4 cos x sin x – 2 sin x = 3 sin x.
2
xy , cos x  cos y  , where x and y are real, is..... (1983)
3 2
(1986) 65. Find the values of x (–, ) which satisfy the equation
2 x|  ....
50. The set of all x in the interval [0, ] for which 21  |cos x |  |cos =4 (1984)
2sin2x–3 sin x + 1 0, is.... . (1987) 66. Consider the system of linear equations in x, y, z
 3  7 9 11 13 (sin 3 ) x – y + z = 0
51. The value of sin sin sin sin sin sin sin
14 14 14 14 14 14 14 (cos 2) x + 4y + 3z = 0
is equal to ....... (1991) 2x + 7y + 7z = 0
52. If k = sin (/18) sin (/18) sin (/18), then the numerical Find the values of for which this system has non-trivial
value of k is ....... (1993) solutions. (1986)
114 TRIGONOMETRY

67. Prove that tan  + 2 tan 2 + 4 tan 4 + 8 cot 8 = cot . 71. Find the smallest positive number p for which the equation
(1988) cos (p sin x) = sin (p cos x) has a solution x  [0, 2].
68. ABC is a triangle such that (1995)
72. Find number of values of  in the interval
1
sin (2A + B) = sin (C – A) = –sin (B + 2C)  .   
2   ,  satisfying the equation
 2 2
If A, B and C are in Arithmetic Progression, determine the 2
values of A, B and C. (1990) (1–tan ) (1 + tan ) sec2   2tan 0.
69. If exp {(sin2x + sin4 x + sin6 x +.......) loge 2}, satisfies the (1996)
equation x 2 – 9x + 8 = 0, find the value of sin x cos3x
cos x  73. Prove that the values of the function do not lie
,0x . (1991) sin 3x cos x
cos x  sin x 2
1
70. Determine the smallest positive value of x (in degrees) for between and 3 for any real x. (1997)
3
which tan (x + 100°) = tan (x + 50°) tan (x) tan (x – 50°)
74. In any triangle prove that
(1993)
A B C A B C
cot  cot  cot  cot cot cot (2000)
2 2 2 2 2 2
115 TRIGONOMETRY

ANSWER KEY
EXERCISE - 1 : BASIC OBJECTIVE QUESTIONS
1. (b) 2. (d) 3. (d) 4. (c) 5. (a) 6. (a) 7. (b) 8. (c)
9. (b) 10. (c) 11. (d) 12. (d) 13. (a) 14. (b) 15. (a) 16. (b)
17. (d) 18. (a) 19. (a,d) 20. (d) 21. (b) 22. (b) 23. (b) 24. (a)
25. (a) 26. (b) 27. (c) 28. (a,b,c,d) 29. (c) 30. (b,c) 31. (b) 32. (a)
33. (a) 34. (a) 35. (d) 36. (b) 37. (a) 38.(b) 39. (a) 40. (b)
41 (b) 42 (a) 43. (b) 44. (b) 45. (b) 46. (c) 47. (b) 48. (a)
49 (b) 50 (b,c) 51. (b,d) 52. (b) 53. (d) 54. (b) 55. (c) 56. (d)
57. (c) 58. (b) 59. (d) 60. (a) 61. (d) 62. (b) 63. (b) 64. (d)
65. (c) 66. (a) 67. (c) 68. (b) 69. (a) 70. (c)
EXERCISE - 2 : PREVIOUS YEAR JEE MAINS QUESTIONS
1. (a) 2. (b) 3. (c) 4. (c) 5. (b) 6. (a) 7. (d) 8. (a)
9. (b) 10. (c) 11. (b) 12. (b) 13. (a) 14. (c) 15. (b) 16. (c)
17. (a) 18. (c) 19. (c) 20. (a) 21. (b) 22. (c) 23. (c) 24. (c)
25. (c) 26. (b) 27. (c) 28. (d) 29. (a) 30. (b) 31. (a) 32. (b)
33. (a) 34. (a) 35. (c) 36. (b) 37. (c) 38. (c) 39. (b) 40. (b)
41. (d) 42. (c) 43. (d) 44. (c) 45. (c) 46. (c) 47. (b) 48. (a)
49. (c) 50. (c) 51. (a)
EXERCISE - 3 : ADVANCED OBJECTIVE QUESTIONS
1. (c) 2. (a) 3. (b) 4. (c) 5. (a) 6. (b) 7. (c) 8. (c)
9. (b) 10. (b) 11. (d) 12. (b) 13. (b) 14. (c) 15. (d) 16. (b)
17. (d) 18. (c) 19. (b) 20. (c) 21. (a) 22. (b) 23. (a) 24. (b)
25. (b) 26. (c) 27. (a,d) 28. (a) 29. (a,b) 30. (c) 31. (a) 32. (c)
33. (a,b,c,d) 34. (c,d) 35. (a,c) 36. (a,b,c,d) 37. (a,b) 38. (d) 39. (c) 40. (b)
41. (d) 42. (d) 43. (c) 44.(a) 45. (d) 46. (a,b,c) 47. (b) 48. (a,b)
49. (d) 50. (d) 51. (a) 52. (b) 53. (d) 54. (b) 55. (a,b,c,d) 56. (a,c,d)
57. (b) 58. (a,b,c,d) 59. (a,b,c,d) 60. (a,b,c) 61. (a,d) 62. (a,b,c) 63. (a,c,d) 64. (b)
65. (a) 66. (d) 67. (e) 68. (a) 69. (c) 70. (b) 71. (c) 72. (b)
73. (c) 74. (d) 75. A  Q; B  R; C  S; D  P 76. A Q; B  R; C  P
77. 0018 78. 0030 79. 0018 80. 0002
EXERCISE - 4 : PREVIOUS YEAR JEE ADVANCED QUESTIONS
1. (b) 2. (a) 3. (b) 4. (a) 5. (c) 6. (c) 7. (b) 8. (d)
9. (c) 10. (c) 11. (b) 12. (d) 13. (c) 14. (a) 15. (c) 16. (d)
17. (c) 18. (c) 19. (c) 20. (c) 21. (b) 22. (b) 23. (d) 24. (a)
25. (b) 26. (c) 27. (d) 28. (b) 29. (d) 30. (c) 31. (c) 32. (a,c)
33. (b,c) 34. (a,b) 35. (a,b,c,d) 36. (a,b) 37. (c,d) 38. (a,c,d) 39. (b,d) 40. (a,c,d)
56
41. (a,c) 42. A–r; B–p 43. (0003) 44. (0008) 46. 47. 6 48. cos (log )
33
       5  1 1
49. no solution 50. x   0,   , 51. 52. 53.   m, n  
 6   2   6  64 8 3
         3 3 
54. True 55. False 56. x  ,  ,  , ,  60.  ,cos  ,cos   ,cos 
2 3 3 8 8  4 4  8 8
    3     2  
66.   n or n  (1) n  
n n
64. {x : x  n}   x : x  n  (1) 10    x : x  n  (1)  10   65.  ,  
      3 3  6
3 1 
68. A = 45 º, B = 60º, C = 75º 69. 70. 30° 71. 72. 4
2 2 2

Dream on !!

04
STATISTICS
117 STATISTICS

STATISTICS

1. INTRODUCTION : n
 fi x i
f x  f x  ....  f n x n
An average or a central value of a statistical series in the x 1 1 2 2  i 1
n
f1  f 2  ...f n
value of the variable which describes the characteristics of  fi
i 1
the entire distribution.
The following are the five measures of central tendency.
f  x  A
(1) Arithmetic Mean (ii) Short cut method : Arithmetic mean x  A
f
(2) Geometric Mean
Where A = assumed mean, f = frequency and x - A = deviation
(3) Harmonic Mean of each item from the assumed mean.
(4) Median (3) Properties of arithmetic mean
(5) Mode (i) Algebraic sum of the deviations of a set of values from
their arthmetic mean is zero. If xi/fi, i = 1, 2, ..., n is the
2. ARITHMETIC MEAN :
frequency distribution, then
Arithmetic mean is the most important among the n
mathematical mean.  fi  x i  x   0, x being the mean of the distribution.
i 1
According to Horace Secrist,
(ii) The sum of the squares of the deviations of a set of
“The arithmetic mean is the amount secured by dividing the
values is minimum when taken about mean.
sum of values of the items in series by their number”.
(1) Simple arithmetic mean in individual series (Ungrouped (ii) Mean of the composite series : If x i (i = 1,2,...,k) are the
data) means of k-component series of sizes ni, (i = 1, 2, ..., k)
(i) Direct method : If the series in this case be x1,x2,x3,...., xn respectively, then the mean x of the composite series
then the arithmetic mean x is given by obtained on combining the component series is given by

Sum of the series x  x 2  x 3  ...  x n 1 n n


x ,i.e., x  1   xi ni xi
Number of terms n n i 1 n x1  n 2 x 2  ....  n k x k i
the formula x  1  1 n
n1  n 2  ...  n k n
(ii) Short cut method i
i 1

d

Arithmetic mean x  A 
n
, Where, A = assumed mean,
3. GEOMETRIC MEAN:
d = deviation from assumed mean = x - A, where x is the
individual item, d = sum of deviations and n= number of If x1, x2, x3, ...xn are n values of a variate x, none of them being
items. zero, then geometic mean (G.M.) is given by
G.M. = (x 1 , x 2 , x 3 , ....x n) 1/n  log (G.M.)
(2) Simple arithmetic mean in continous series (Grouped
data) 1
  log x1  log x 2  ...  log x n  .
(i) Direct method : If the terms of the given series be x1, x2, 2
....xn and the corresponding frequencies be f1, f2, f3...fn, then In case of frequency distribution G.M. of n values x1, x2, ..., xn
the arithmetic mean x is given by,, of a variate x occuring with frequency f1, f2,..., fn is given by
1


G.M.  x1f1 .x f22 ,....,fnn  N , where N = f1 + f2 + ... + fn.
118 STATISTICS

Where l = Lower limit of the median class


4. HARMONIC MEAN :
f = Frequency of the median calss
The harmonic mean of n items x1,x2,...., xn is defined as N = The sum of the all frequencies
n i = The width of the median class
H.M. 
1 1 1 C = The cumulative frequency of the class precending to
  ...... 
x1 x 2 xn median class.
(b) For series in descending order
If the frequency distribution is f1, f2, f3,......,fn respectively,

f1  f 2  f3  .....  f n N 
ten H.M.   C
 f1 f 2 f   2  i
 .......  n  Median = u  , where u = upper limit of the
  f
x
 1 x 2 x n 
median class.
Note : A.M. gives more weightage to larger values whereas
n
G.M. given more weightage to smalle. M   fi
i 1
5. MEDIAN :
As median divides a distribution into two equal parts,
Median is defined as the value of an item or observation similarly the quartiles quantiles, deciles and percentiles
above or below which lies on an equal number of observation divide the distribution respectively into 4, 5, 10 and 100
i.e., the median is the central value of the set of observations equal part. The j th quartile is given by
provided all the observations are arranged in the ascending
or descending orders.  N 
 j 4 C
(1) Calculation of median Qj  l    i; j  1, 2,3.Q1 is the lower quartile, Q2 is
f
 
(i) Individual series : If the data is raw, arrange in ascending  
or descending order. Let n be the number of observations.
the median and Q3 is called the upper quartile.
th (2) Lower quartile
 n 1 
If n is odd, Median = value of   item.
 2  th
(i) Discrete series : Q1  size of  n  1  item
If n is even, median  4 
th th
1 n n  
  value of   item + value of   1  item  N 
2  2 2    C
 4
(ii) Continous series : Q1      i
f
(ii) Discrete series : In this case, we first find the cumulative
frequencies of the variables arragned in ascending or (3) Upper quartile
descending order and the median is given by
th
 3  n  1 
 n 1 
th (i) Discrete series : Q3  size of   item
Median =   observation, where n is the cumulative  4 
 2 
frequency. 3N
C
(iii) For grouped or continous distributions : In this case, (ii) Continous series : Q3    4 i
following formula can be used f

(4) Decile divide total frequencies N into ten equal parts


N 
  C
2
(a) For series in ascending order, Median = l    i N j
C
f
D j    10  i [j =1, 2, 3, 4, 5, 6, 7, 8, 9]
f
STATISTICS 119

A distribution which is not symmetric is called is skewed


N
C distribution. In a moderately asymmetric the interval between
If j = 5 then D5    2  i . Hence D5 is also known as the mean and the median is approximately one-third of the
f
interval between the mean and the mode i.e. we have the
median. following empirical relation between them
(5) Percentile divide total frequencies N into hundred equal Mean - Mode = 3(Mean – Median )  Mode = 3 Median - 2
parts Mean. It is known as Empirical relation.

Nk
C
Pk    100 i
f

where k = 1, 2, 3, 4, 5, ..., 99. Some point about arithmetic mean


Of all types of averages the arithmetic mean is most
6. MODE :
commonly used average.
Mode : The mode or model value of a distribution is that It is based upon all observations.
value of the variable for which the frequency is maximum.
If the number of observations is very large, it is more accurate
For continous series, mode is calculated as, Mode
and more reliable basis for comparison.
 f1  f0 
 l1   i Some point about geometric mean
 2f1  f 0  f 2 
It is based on all items of the series
Where, l = The lower limit of the model class.
It is most suitable for constructing index number, average
f1 = The frequency of the model class ratios, precentages etc.
f0 = The frequency of the class precending the model class
G.M. cannot be calculated if the size of any of the item is
f2 = The frequency of the class succeeding the model class zero is negative.
i = The size of the model class.
Some point about H.M.
Symmetric distribution : A symmetric is a symmetric
distribution if the values of mean, mode and median coincide. It is based on all item of the series.
In a symmetric distribution frequencies are symmetrically This is useful in problems related with rates, ratios, time etc.
distributed on both sides of the centre point of the frequency
A.M.  G.M.  H.M. and also (G.M.)2 = (A.M.) (H.M.)
Some points about median
It is an appropriate average in dealing with qualitative date,
like intelligence, wealth etc.
The sum of the deviation of the item from median, ignoring
algebraic signs, is less than the sum from any other point.
Some points about mode
It is not based on all items of the series.
As compared to other averages mode is affected to a large
extent by fluctuations of sampling.
It is not suitable in a case where the relative importance of
items have to be considered.
120 STATISTICS

7. MEASURE OF DISPERSION :  xM


Mean deviation 
n
The degree to which numerical date tend to spread about an
average value is called the dispersion of the data. The four Where |x - M| means the modulus of the deviation of the
measure of the dispersion are variate from the mean (mean, median or mode). M and n is
the number of terms.
(1) Range (2) Mean deviation
(ii) Mean deviation from continous series : Here first of all
(3) Standard deviation (4) Square deviation we find the mean from which deviation is to be taken. Then
(1) Range : It is the difference between the values of extreme we find the deviation dM = |x - M| of each variate from the
items in a series. Range = Xmax - Xmin The coefficient of mean M so obtained.
x max  x min Next we multiply these deviation by the corrsponding
range (scatter)  x frequency and find the product f. dM and then the sum
max  x min
fdM of these products.
Range is not measure of central tendency. Range is widely Lastly we use the formula, Mean deviation
used in statistical series relating quality control in
production. f x  M fdM
  , where n = f
(i) Inter-quartile range : We know that quartiles are the n n
magnitudes of the items which divide the distribution into (3) Standard deviation : Standard deviation (or S.D.) is the
four equal parts. The inter-quartile range is found by taking square root of the arithmetic mean of the square of deviations
the differenc between third and first quartiles and is given of various values from their arithmetic mean and is generally
by the formul. denoted by read as sigma.
Inter-quartile range = Q3 - Q1 (i) Coefficient of standard deivation : To compare the
Where Q1 = First quartile or lower quartile and Q3 = Third dispersion of two frequency distributions the relative
quartile or upper quartile. measure of standard deviation is computed which is known
as coefficient of standard deviation and is given by
(ii) Percentile range : This is measured by the following
formula. 
Coefficient of S.D. = , where x is the A.M.
Percentile range = P90 - P10 x
Where P90 = 90th percentile and P10 = 10th percentile (ii) Standard deviation from individual series
Percentile ragne is considered better than range as well as
inter-quartile range. 2
x  x

(iii) Quartile deviation or semi inter-quartile range : It is one- N
half of the difference between the third quartile and first
where, x = The arithmetic mean of series
Q3  Q1
quartile i.e., Q.D.  and coefficient of quartile N = The total frequency.
2
(iii) Standard deviation from continuous series
Q3  Q1
deviation  Q  Q . 2
3 1 f1  x i  x 

Where, Q3 is the third or upper quartile and Q1 is the lowest N
or first quartile.
where, x = Arithmetic mean of series
(2) Mean deviation : The arithmatic average of the deviation
xi = Mid value of the class
(all taking positive) from the mean, median or mode is known
as mean deviation. fi = Frequency of the corresponding xi

(i) Mean deviation from ungerouped data (or indiviadual N = f = The total frequency
series)
STATISTICS 121

Short cut method Variance of the combined series : If n1; n2 are the sizes, x1 ; x 2

2
the means and 1 ; 2 the standard deviation of two series,
2
fd 2  fd  d 2  d 
(i)     (ii)    
N  N  N  N 1
2
then      
 n1 12  d12  n 2  22  d 22 
n1  n 2  
where, d = x – A = Deviation from the assumed mean A
f = Frequency of the item n1x1  n 2 x 2
Where, d1  x1  x, d 2  x 2  x  .
N = f = Sum of frequencies n1  n 2
(4) Square deviation
(i) Root mean square deviation

1 n 2
S  fi (x  A)
4 i 1 Range is widely used in satistical series relating to quality
control in production.
where A is any arbitrary number and S is called mean square
Standard deviation  Range i.e., variance (Range)2.
deviation.
Empirical relation between measures of dispersion.
(ii) Relation between S.D. and root mean square deviation :
If be the standard deivation and S be the root mean square 4
deviation. Mean deviation = (standard deviation)
5
Then S2 = 2 + d2.

Obviously, S2 will be least when d = 0 i.e. x  A 2


Semi interquartile range = (Standard deviation)
3
Hence, mean square deviation and consequently root mean
square deviation is least, if the deviations are taken from the
5
mean. Semi interquartile range = (mean deviation)
6
8. VARIANCE : For a symmetrical distribution, the following area relationship
hold good.
The square of standard deviation is called the variance.
Coefficient of standard deviation and variance : The X covers 68.72% items
coefficient of standard deviation is the ratio of the S.D. to
X covers 95.45% items

A.M. i.e., . Coefficient of variance = coefficient of
x X covers 99.74% items

 n2 1
S.D. 100  100 . S.D. of first n natural number is .
x 12

Range is not the measure of central tendency.


122 STATISTICS

9. SKEWNESS : (1) Distribution : There are three types of distributions.


(i) Normal distribution : When = 0, the distribution is said
“Skewness” Measures the lack of symmetry. It is measured to be normal. In this case Mean = Median = Mode
(ii) Positively skewed distribution : When  1 > 0, the
3
  xi    distribution is said to be positively skewed. In this case
by 1  3 and is denoted by .
  Mean > Median > Mode

 x i  
2
 2

  (iii) Negative skewed distribution : When  < 0, the
distribution is said to be negatively skewed. In this case
The distribution is skewed if, Mean < Median < Mode.
(i) Mean Median  Mode (2) Measures of skewness
(ii) Quartiles are not quidistant from the median and (i) Absolute measures of skewness : Various measures of
(iii The frequency curve is stretched more to one side than skewness are
to the other. (a) Sk = M - Md
(b) Sk = M - MO
(c) Sk = Q3 + Q1 - 2Md
Where, Md = median , MO = mode, M = mean
Absolute measures of skewness are not useful to compare
two series, therefore relative measure of dispersion are used,
as they are pure numbers.
STATISTICS 123

SOLVED EXAMPLES

Example 1 : Example 3 :

If the mean of the distribution is 2.6, then the value of y is If the mean of the set of number x1,x2, x3, ...,xn is x , then the
mean of the number xi + 2i, 1  i  n is
Variate x :1 2 3 4 5
Frequency f of x: 4 5 y 1 2 (a) x  2n (b) x  n  1
(a) 24 (b) 13 (c) 8 (d) 3 (c) x  2 (d) x  n

n
n  xi n
 fi x i Solution (b) : We know that x  i 1
i.e.,  x i  nx
i 1
Solution (c): We know that, Mean  n
n i 1
 fi
i 1
n n n
  x i  2i   xi  2 i nx  2 1  2  ...n 
 i 1
 i 1 i 1

n n n
1 4  2  5  3xy  4  1  5  2
i.e. 2.6  or 31.2 + 2.6y = 28
4  5  y 1 2
n  n  1
nx  2
+ 3y or 0.4y = 3.2  y = 8  2  x   n  1
n
Example 2 :
Example 4 :
In a class of 100 students there are 70 boys whose average
The harmonic mean of 4, 8, 16 is
marks in a subject are 75. If the average marks of the
(a) 6.4 (b) 6.7 (c) 6.85 (d)7.8
complete class are 72, then what are the average marks of
the girls.
3 48
(a) 73 (b) 65 (c) 68 (d) 74 Solution (c): H.M. of 4, 8, 16    6.85
1 1 1 7
 
4 8 16
Solution (b) : Let the average marks of the girls students be x,
then Example 5 :

The average of n number x1, x2, x3, ... xn is M. If xn is


70  75  30  x
72  (Number of girls = 100 - 70 = 30) replaced by x’, then new average is
100
nM  x n  x '
(a) M - xn + x’ (b)
n
7200  5250
i.e.,  x,  x  65.
30
 n  1 M  x ' M  x n  x'
(c) (d)
n n

x1  x 2  x 3 ...x n
Solution (b): M  i.e.,
n
124 STATISTICS

Example 8 :
nM  x1  x 2  x 3  ...x n 1  x n
nM  x n  x1  x 2  x 3  ...x n 1 The median of a set of 9 distinct observation is 20.5. If
nM  x n  x'  x1  x 2  x 3  x n 1  x ' each of the largest 4 observation of the set is increased by
2. then the median of the new set.
n n
(a) Is increased by 2
(b) is decreased by 2
nM  x n  x '
 New average  (c) Is two times the original median
n
(d) Remains the same as that of the original set
Example 6 :

Mean of 100 items is 49. It was discovered that three items  9 1 th
which should have been 60, 70, 80 were wrongly read as Solution (d): n = 9, then median term     5 term. Since
 2 
40, 20, 50 respectively. The correct mean is
last four observation are increased by 2.
1  The median is 5th observation which is remaining
(a) 48 (b) 82 (c) 50 (d) 80
2 unchanged.
 There will be no change in median.
Solution (c):
Example 9 :
Sum of 100 items = 49 × 100 = 4900
Sum of items added = 60 + 70 + 80 = 210 Compute the median from the following table
Sum of items replace = 40 + 20 + 50 = 110
New sum = 4900 + 210 - 110 = 5000 Marks obtained No. of students
0-10 2
5000 10-20 18
 Correct mean=  50
100 20-30 30
30-40 45
Example 7 : 40-50 35
50-60 20
The following date gives the distribution of height of 60-70 6
students 70-80 3

Height (in cm) 160 150 152 161 156 154 155
Number of students 12 8 4 4 3 3 7
(a) 36.55 (b) 35.55
The median of the distribution is (c) 40.05 (d) None of these
(a) 154 (b) 155 (c) 160 (d) 161
Solution (a):
Solution (b): Arranging the data in ascending order of magnitude,
Marks obtained No. of Cumulative
we obtain students Frequency
0-10 2 2
H eig h t (in c m ) 150 152 154 155 156 160
N u m b er o f stu d e nts 8 4 3 7 3 12
10-20 18 20
C u m u lativ e 8 12 15 22 25 37 20-30 30 50
freq u en c y 30-40 45 95
40-50 35 130
Here, total number of items is 41, i.e., an odd number. Hence, 50-60 20 150
41  1 60-70 6 156
the median is th i.e. 21st item 70-80 3 159
2
For cumulative frequency table, we find that median i.e., n =  f = 159
21st item is 155. Hence n = 159, which is odd.
(All items from 16 to 22nd are equal each = 155)
STATISTICS 125

1 1 Example 12 :
Median number   n  1  159  1  80, which is in In a series of 2n observations, half of them equal a and
2 2
the class 30-40 (see the row of cumulative frequency 95, remaining half equal -a. If the standard deviation of the
which contains 80). observations is 2, then |a| equals

Hence median class is 30-40. 2 1


(a) (b) 2 (c) 2 (d)
 We have l = Lower limit of median class = 30 n n
f = Frequency of median class = 45
C = Total of all frequency precending median class = 50 Solution (c): Let a, a, ...., n times -a, -a, -a, ---- n time i.e. mean = 0
i = Width of class interval of median class = 10 2 2
n  a  0   n  a  0 
 Required median and S.D.=
2n
N 159
C  50
2  i  30  2
295 na 2  na 2
 l  10  30   36.55 2  a 2  a. Hence |a| = 2.
f 45 45 2n

Example 10 : Example 13 :

A batsman scores runs in 10 inning 38, 70, 48, 34, 42, 55, If  is the mean of distribution (yi, fi), then fi(yi - ) =
63, 46, 54, 44, then the mean deviation is
(a) M.D. (b) S.D. (c) 0 (d) Relative frequency
(a) 8.6 (b) 6.4 (c) 10.6 (d) 9.6
Solution (c): We have fi(yi - ) = fiyi – fi = fi – fi = 0
Solution (a): Arranging the given data in ascending order, we
have  fi yi 
  
34, 38, 42, 44, 46, 48, 54, 55, 63, 70  fi 

46  48 Example 14 :
Here median M   47
2
What is the standard deviation of the following series
( n = 10, median is the mean of 5th and 6th items)
Measurements 0-10 10-20 20-30 30-40
 Mean deviation
Frequency 1 3 4 2
 xi  M  x i  47 13  9  5  3  1  1  7  8  16  23 (a) 81 (b) 7.6 (c) 9 (d) 2.26
   8.6
n 10 10
Solution (c):
Example 11 :

S.D. of data is 6 when each observation is increased by 1, Class Frequency y1 yi  A f1u1 f1u12
i  , A  25
then the S.D. of new data is 10
(a) 5 (b) 7 (c) 6 (d) 8 0–10 1 5 –2 –2 4
10–20 3 15 –1 –3 3
Solution (c): S.D. and variance of data is not changed, when each 20–30 4 25 0 0 0
observation is increased (OR decreased) by the same 30–40 2 35 1 2 2
constant. 10 –3 9

 f u 2  f u 2 2
2 2 
  c  i i  i i  
 f i  f 
  i  
126 STATISTICS

Example 16 :
2
 9  3  2 
 10       90  9  81    9
10  10   ax  b
The S.D. of a variate x is  . The S.D. of the variate
c
Example 15 :
where a, b, c are constant, is
In an experiment with 15 observation on x, the following
a a
resultswereavailablex2 = 2830, x = 170. On observation (a)    (b) 
that was 20 was found to be wrong and was replaced by c c
the correct value 30. Then the corrected variance is
 a2 
(a) 78.00 (b) 188.66 (c) 177.33 (d) 8.33 (c)  2   (d) None of these
c 
Solution (a): x = 170, x2 = 2830
Increase in x = 10 then x1 = 170 + 10 = 180 Solution (b):
Increase in x2 = 900 - 400 = 500, then x’ = 2830 + 500 = 3330 ax  b a b
Let y  i.e., y  x  i.e. y = Ax + B, where
2 2 c c c
1  x '  3330  180 
Variance  x '2        222  144  78
n  n  15  15  a b
A ,B 
c c

 y  Ax  B

2 2
yy  A xx  yy     
 A2 x  x 
2 2
   y  y   A 2   x  x   n.2y  A.n2y  2y  A 2 2x

a
  y  A x   y  x
c

a
Thus, new S.D.  .
c
127 STATISTICS

EXERCISE - 1 : BASIC OBJECTIVE QUESTIONS


7. The number which indicates variability of data or
1. The mean of a set of observation is x . If each observation
observations, is called
is divided by  0 and then is increased by 10, then
the mean of the new (a) measure of central tendency
(b) mean
x x  10 (c) median
(a) (b)
 
(d) measure of dispersion

x  10 8. Mean deviation about the median for the data 3, 9, 5, 3,


(c) (d) x  10 12, 10, 18, 4, 7, 19, 21 is

(a) 4.27 (b) 5.24
2. If the arithmetic mean of the numbers x1, x2, x3,...........,xn is
(c) 5.27 (d) 4.24
x , then the arithmetic mean of numbers ax1 + b, ax2 + b,
9. The mean deviation of the data 3, 10, 10, 4, 7, 10, 5 from
ax3 + b, ......, axn + b, where a, b are two constants would be
the mean is
(a) x (b) nax  nb (a) 2 (b) 2.57

(c) a x (d) ax  b (c) 3 (d) 3.75


10. When tested, the lives (in hours) of 5 bulbs were noted
3. The number of observation in a group is 40. If the average
as follows
of first 10 is 4.5 and average of remaining 30 is 3.5, then
the average of the whole group is 1357, 1090, 1666, 1494, 1623
The mean deviation (in hours) from their mean is
1 15
(a) (b) (a) 178 (b) 179
5 4
(c) 220 (d) 356
(c) 4 (d) 8
11. Following are the marks obtained by 9 students in a
4. Mean of 100 observations was found to be 45. It was later Mathematics test.
found that two observations 19 and 31 were incorrectly
50, 69, 20, 33, 53, 39, 40, 65, 59
recorded as 91 and 13. The correct mean is
The mean deviation from the median is
(a) 44.0 (b) 44.46
(a) 9 (b) 10.5
(c) 45.00 (d) 45.54
(c) 12.67 (d) 14.76
5. The average weight of students in a class of 35 students
is 40 kg. If the weight of the teacher be included, the 12. The mean deviation of the data 2, 9, 9, 3, 6, 9, 4 from the
mean is
1
average rises by kg; the weight of the teacher is (a) 2.23 (b) 2.57
2
(c) 3.23 (d) 3.57
(a) 40.5 kg (b) 50 kg
13. The variance of the data 2, 4, 6, 8, 10 is
(c) 41 kg (d) 58 kg
(a) 6 (b) 7
6. The A.M. of n observations is M. If the sum of n–4
(c) 8 (d) None of these
observations is a, then the mean of remaining 4
observations is 14. If the standard deviation of 0, 1, 2, 3,......9 is K, then the
standard deviation of 10, 11, 12, 13, ..... 19 is
nM  a nM  a
(a) (b) (a) K (b) K + 10
4 2
(c) K  10 (d) 10 K
nM  A
(c) (d) nM + a
2
128 STATISTICS

15. Statement I The average marks of boys in a class is 52 21. The standard deviation of 25 numbers is 40. If each of the
and that of girls is 42. The average marks of boys and numbers is increased by 5, then the new standard
girls combined is 50. The percentage of boys in the class deviation will be
is 80%. (a) 40 (b) 45
Statement II Mean marks scored by the students of a
class is 53. The mean marks of the girls is 55 and the mean 21
(c) 40  (d) None of these
marks of the boys is 50. The percentage of girls in the 25
class is 64%. 22. The median of a set of 9 distinct observations is 20.5. If
(a) Only statement I is true (b) Only statement II is true each of the largest 4 observations of the set is increased
(c) Both statements are true(d) Both statements are false by 2, then the median of the new set is
16. The variance of the first n natural numbers is (a) increased by 2 (b) decreased by 2
(c) two times the original median(d) remains the same as
n2 1 n2 1 that of original set
(a) (b)
12 6 23. The mean age of a combined group of men and women is
25 yrs. If the mean age of the group of men is 26 and that
n2 1 n2 1 of the group of women is 21, then the ratio of men and
(c) (d)
6 12 women in the group is
17. If v is the variance and  is the standard deviation, then (a) 1 : 4 (b) 4 : 1
(c) 3 : 1 (d) 1 : 3
(a) v2   (b) v   2
24. The mean of five observations is 4 and their variance is
1 1 5.2. If three observations are 1, 2, and 6, the other two are
(c) v  (d) v 
 2 (a) 2 and 9 (b) 3 and 8
(c) 4 and 7 (d) 5 and 6
18. If each observation of a raw data whose variance is 2 ,
25. Consider any set of observations x1, x2, x3,....,x101; it being
is increased by  , then the variance of the new set is
given that x1 < x2 < x3 < ... <x101; then the mean deviation of
(a)  2 (b)  2  2 this set of observations about a point k is minimum when
k equals
(c)   2 (d)  2  2 (a) x1 (b) x51

19. If each observation of a raw data whose variance is 2 , x1  x 2  ....  x101


(c) (d) x50
is multiplied by  , then the variance of the new set is 101

(a)  2 (b)  2  2 26. Statement I The mean and variance for first n natural

(c)   2 (d)  2  2 n 1 n2 1
numbers are and , respectively..
2 12
20. If the variance of observations x1, x2, ....... xn is  2 , then
Statement I The mean and variance for first 10 positive
the variance of observations ax1, ax2, ......, axn, a  0 is multiples of 3 are 16.5 and 74.25, respectively.
(a)  2 (b) a2 (a) Only statement I is true (b) Only statements II is true
(c) Both statements are true(d) Both statements are false
2 27. The standard deviation of the data 6, 5, 9, 13, 12, 8, 10 is
(c) a 2  2 (d)
a2
52 52
(a) (b)
7 7

(c) 6 (d) 6
STATISTICS 129

28. Variance of the data 2, 4, 5, 6, 8, 17 is 23.33. Then, variance Further, another set of 15 observations x1, x2......,xn (also
of the data 4, 8, 10, 12, 16, 34 will be n 15
(a) 23.33 (b) 25.33 in seconds) is now available and we have  x i  279
i 1
(c) 93.32 (d) 98.32
15
29. The mean of 100 observations is 50 and their standard 2
and  x i  5524. The standard deviation of all 40
deviation is 5. The sum of squares of all observations is i 1

(a) 50000 (b) 250000 observations is


(c) 252500 (d) 255000 (a) 2.87 (b) 4.87
30. Consider the first 10 positive integers. If we multiply each (c) 3.87 (d) 5.87
number by (-1) and then add 1 to each number, the 36. The variance of 20 observations is 5. If each observation
variance of the numbers so obtained is is multiplied by 2, then the new variance of the resulting
(a) 8.25 (b) 6.5 observations is
(c) 3.87 (d) 2.87 (a) 23  5 (b) 22  5
31. If the coefficient of variation of two distributions are 60 (c) 2  5 (d) 24  5
and 70 and their standard deviations are 21 and 16 37. The mean and standard deviation of a group of 100
respectively, then their arithmetic means are observations were found to be 20 and 3 respectively. Later
(a) 35, 22. 80 (b) 32, 22. 80 on, it was found that three observations were incorrect,
which were recorded as 21, 21 and 18, then the mean and
(c) 35, 22. 85 (d) 32, 22. 85
standard deviation, if the incorrect observations are
32. Coefficient of variation of two distributions are 50 and 60 omitted, are
and their arithmetic means are 30 and 25 respectively.
(a) 21, 3.02 (b) 20, 3.02
Then, difference of their standard deviations is
(c) 21, 3.03 (d) 20, 3.03
(a) 0 (b) 1
38. Let a, b, c, d and e be the observations with mean m and
(c) 1.5 (d) 2.5
standard deviation s. The standard deviation of the
33. If the coefficient of variation is 45 and the mean is 12, observations a + k, b + k, c + k, d + k and e + k is
then its standard deviation is
(a) s (b) ks
(a) 5.2 (b) 5.3
(c) s + k (d) s/k
(c) 5.4 (d) None of these
39. Let x1, x2, x3, x4 and x5 be the observations with mean m and
34. Coefficient of variation of two distributions are 50 and 60 standard deviation s. Then, standard deviation of the
and their standard deviations are 10 and 15, respectively. observations kx1, kx2, kx3, kx4 and kx5 is
Then, difference of their arithmetic means is
(a) k + s (b) s / k
(a) 3 (b) 4
(c) ks (d) s
(c) 5 (d) 6
40. Let x1, x2 ......, xn be n observations. Let i  x i  k for
35. The mean and standard deviation of some data (in
seconds) for the time taken to complete a tests, calculated i = 1, 2,.........,n, where l and k are constants. If the mean of
with the following results : xi’s is 48 and their standard deviation is 12, the mean of

Number of observations = 25, i ’s is 55 and standard deviation of i ’s is 15. The values

Mean = 18.2, of l and k should be

Standard deviation = 3.25 (a) l = 1.25, k = -5 (b) l = –1.25, k = 5


(c) l = 2.5, k = -5 (d) l = 2.5, k = 5
STATISTICS 130

EXERCISE - 2 : PREVIOUS YEAR JEE MAINS QUESTIONS


1. In a class of 100 students there are 70 boys whose average 6. In a frequency distribution, the mean and median are 21
marks in a subject are 75. If the average marks of the complete and 22 respectively, then its mode is approximately
class is 72, then what is the average marks of the girls ? (2005)
(2002) (a) 20.5 (b) 22.0
(a) 73 (b) 65 (c) 24.0 (d) 25.5
(c) 68 (d) 74 7. Let x1 ,x 2 , ........ xn be n observations such that
2. In an experiment with 15 observations on x, the following
results were available.  xi2  400 and  x i  80. Then a possible value of n
among the following is (2005)
 x 2  2830,  x  170
(a) 18 (b) 15
One observation that was 20 was found to be wrong and (c) 12 (d) 9
was replaced by the correct value 30. Then the corrected
8. Suppose a population A has 100 observations 101, 102,
variance is (2003)
......., 200, and another population B has 100 observations
(a) 188.66 (b) 177.33
151, 152, ....., 250. If VA and VB represent the variances of
(c) 8.33 (d) 78.00
the two populations, respectively, then VA / VB is
3. The median of a set of 9 distinct observation is 20.5. If
each of the largest 4 observations of the set is increased (2006)
by 2, then the median of the new set (2003) (a) 1 (b) 9/4
(a) is decreased by 2 (c) 4/9 (d) 2/3
(b) is two times the original median 9. The average marks of boys in class is 52 and that of girls
(c) remains the same as that of the original set is 42. The average marks of boys and girls combined is 50.
(d) is increased by 2 The percentage of boys in the class is (2007)
4. Consider the following statements : (a) 80 (b) 60
(1) Mode can be computed from histogram (c) 40 (d) 20
(2) Median is not independent of change of scale 10. The mean of the numbers a, b, 8, 5, 10 is 6 and the variance
is 6.80. Then which one of the following gives possible
(3) Variance is independent of change of origin and scale.
values of a and b ? (2008)
Which of these is/are correct ? (2004)
(a) a = 3, b = 4 (b) a = 0, b = 7
(a) only (1) and (2) (b) only (2)
(c) a = 5, b = 2 (d) a = 1, b = 6
(c) only (1) (d) (1), (2) and (3)
11. If the mean deviation of numbers 1, 1 + d, 1 + 2d, ....... 1 +
5. In a series of 2n observations, half of them equal a and 100d from their mean is 255, then the d is equal to
remaining half equal – a. If the standard deviation of the
(2009)
observations is 2, then |a| equals (2004)
(a) 20.0 (b) 10.1
(a) 2 (b) 2
(c) 20.2 (d) 10.0

1 2
(c) (d)
n n
STATISTICS 131

12. Statement-1 : The variance of first n even natural numbers 16. All the students of a class performed poorly in
Mathematics. The teacher decided to give grace marks of
n2 1 10 to each of the students. Which of the following statistical
is
4 measures will not change even after the grace marks were
Statement-2 : The sum of first n natural numbers is given ? (2013)
(a) median (b) mode
n (n  1)
and the sum of squares of first n natural numbers (c) variance (d) mean
2
17. The mean of a data set consisting of 20 observations is 40.
n (n  1) (2 n  1)
is (2009) If one observation 53 was wrongly recorded as 33, then
6 the correct mean will be (2013/Online Set 1)
(a) Statement 1 is true, Statement 2 is true; Statement 2 is (a) 41 (b) 49
not a correct explanation for Statement 1.
(c) 40.5 (d) 42.5
(b) Statement 1 is true, Statement 2 is false
18. Mean of 5 observations is 7. If four of these observations
(c) Statement 1 is false, Statement 2 is true. are 6, 7, 8, 10 and one is missing, then the variance of all
(d) Statement 1 is true, Statement 2 is true; Statement 2 is the five observations is (2013/Online Set 2)
a correct explanation for Statement 1 (a) 4 (b) 6
13. For two data sets, each of size 5, the variances are given to (c) 8 (d) 2
be 4 and 5 and the corresponding means are given to be 2
19. If the median and the range of four numbers
and 4, respectively. The variance of the combined data set
[x, y, 2x + y, x – y], where 0 < y < x < 2y, are 10 and 28
is (2010)
respectively, then the mean of the numbers is
5 11 (2013/Online Set 3)
(a) (b)
2 2 (a) 18 (b) 10
(c) 5 (d) 14
13
(c) 6 (d) 20. In a set of 2n observation, half of them are equal to a and
2
the remaining half are equal to –a. If the standard deviation
14. If the mean deviation about the median of the numbers a, of all the observations is 2, then the value of |a| is
2a, ..........., 50a is 50, then |a| equals (2011) (2013/Online Set 4)
(a) 4 (b) 5
(a) 2 (b) 2
(c) 2 (d) 3
(c) 4 (d) 2 2
15. Let x1 , x 2 , ......., x n be n observations, and let x be their
21. The variance of first 50 even natural numbers is (2014)
arithmetic mean and 2 be their variance. (2012)
(a) 833 (b) 437
2
Statement 1 : Variance of 2x1 , 2x 2 , ........., 2 x n is 4 . 437 833
(c) (d)
Statement 2 : Arithmetic mean of 2x1 , 2x 2 , ......, 2x n is 4 4
22. In a set of 2n distinct observations, each of the observation
4x.
below the median of all the observations is increased by 5
(a) Statement 1 is true, Statement 2 is true; Statement 2 is and each of the remaining observations is decreased by 3.
not a correct explanation for Statement 1. Then the mean of the new set of observations:
(b) Statement 1 is true, Statement 2 is false (2014/Online Set 1)
(c) Statement 1 is false, Statement 2 is true. (a) increases by 1 (b) decreases by 1
(d) Statement 1 is true, Statement 2 is true; Statement 2 is (c) decreases by 2 (d) increases by 2
a correct explanation for Statement 1
132 STATISTICS

23. Let X and M.D. be the mean and the mean deviation 29. The mean age of 25 teachers in a school is 40 years. A
teacher retires at the age of 60 years and a new teacher is
about X of n observations x i , i  1, 2,......., n. If each of appointed in his place. If now the mean age of the teachers
the observations is increased by 5, then the new mean in this school is 39 years, then the age (in years) of the
and the mean deviation about the new mean, respectively, newly appointed teacher is (2017/Online Set 1)
are : (2014/Online Set 3) (a) 25 (b) 35
(a) X, M.D. (b) X  5, M.D. (c) 30 (d) 40
30. The sum of 100 observations and the sum of their squares
(c) X, M.D.  5 (d) X  5, M.D.  5 are 400 and 2474, respectively. Later on, three observations
3, 4 and 5, were found to be incorrect. If the incorrect
24. Let x, M and  2 be respectively the mean, mode and observations are omitted, then the variance of the
variance of n observations x1 , x 2 ..........., x n and remaining observations is (2017/Online Set 2)
(a) 8.00 (b) 8.25
di   x i  a, i  1, 2, .........., n, where a is any number..
(c) 9.00 (d) 8.50
(2014/Online Set 4)
9 9
2
Statement-1 : Variance of d1 , d 2 , .............., d n is  2 31. If  (x i  5)  9 and  (x i  5)  45, then the
i 1 i 1
Statement-2 : Mean and mode of d1 , d 2 , .........., d n are
standard deviation of the 9 items x1 , x 2 , ........, x 9 is :
 x  a and – M – a, respectively.. (2018)
(a) Statement-1 and Statement-2 are both false (a) 9 (b) 4
(b) Statement-1 and Statement-2 are both true (c) 2 (d) 3
(c) Statement-1 is true and Statement-2 is false 32. The mean of a set of 30 observations is 75. If each
(d) Statement-1 is false and Statement-2 is true observation is multiplied by a non-zero number and then
each of them is decreased by 25, their mean remains the
25. The mean of the data set comprising of 16 observations is same. Then is equal to : (2018/Online Set 1)
16. If one of the observation valued 16 is deleted and three
new observations valued 3, 4 and 5 are added to the data, 1 2
then the mean of the resultant data is : (a) (b)
3 3
(2015)
(a) 16.0 (b) 15.8 4 10
(c) (d)
3 3
(c) 14.0 (d) 16.8
26. If the standard deviation of the numbers 2, 3, a and 11 is 33. If the mean of the data : 7, 8, 9, 7, 8, 7, , 8 is 8, then the
3.5, then which of the following is true ? (2016) variance of this data is : (2018/Online Set 2)

(a) 3a 2  23a  44  0 (b) 3a 2  26a  55  0 7


(a) (b) 1
8
2 2
(c) 3a  32a  84  0 (d) 3a  34a  91  0
9
27. If the mean deviation of the numbers 1, 1 + d, ......., 1 + 100d (c) (d) 2
from their mean is 255, then a value of d is 8
(2016/Online Set 1) 34. The mean and the standard deviation (s.d.) of five
observations are 9 and 0, respectively. If one of the
(a) 10.1 (b) 5.05
observations is changed such that the mean of the new
(c) 20.2 (d) 10 set of five observations becomes 10, then their s.d. is:
28. The mean of 5 observations is 5 and their variance is 124. (2018/Online Set 3)
If three of the observations are 1, 2 and 6; then the mean
(a) 0 (b) 1
deviation from the mean of the data is
(c) 2 (d) 4
(2016/Online Set 2)
(a) 2.8 (b) 2.6
(c) 8.4 (d) 2.4
133 STATISTICS

ANSWER KEY
EXERCISE - 1 : BASIC OBJECTIVE QUESTIONS
1. (c) 2. (d) 3. (b) 4. (b) 5. (d) 6. (a) 7. (d) 8. (c) 9. (b) 10. (a)
11. (c) 12. (b) 13. (c) 14. (a) 15. (a) 16. (a) 17. (b) 18. (a) 19. (b) 20. (c)
21. (a) 22. (d) 23. (b) 24. (c) 25. (b) 26. (b) 27. (a) 28. (c) 29. (c) 30. (a)
31. (c) 32. (a) 33. (c) 34. (c) 35. (c) 36. (b) 37. (d) 38. (a) 39. (c) 40. (a)

EXERCISE - 2 : PREVIOUS YEAR JEE MAINS QUESTIONS


1. (b) 2. (d) 3. (c) 4. (a) 5. (a) 6. (c) 7. (a) 8. (a) 9. (a) 10. (a)
11. (b) 12. (c) 13. (b) 14. (a) 15. (b) 16. (c) 17. (a) 18. (a) 19. (d) 20. (a)
21. (a) 22. (a) 23. (b) 24. (b) 25. (c) 26. (c) 27. (a) 28. (c) 29. (b) 30. (c)
31. (c) 32. (c) 33. (b) 34. (c)

Dream on !!

05
MATHEMATICAL REASONING
135 MATHEMATICAL REASONING

MATHEMATICAL REASONING

1. STATEMENT (Proposition) : 3. NEGATION OF A STATEMENT :


In reasoning we communicate our ideas or thoughts with The denial of a statement p is called its negation and is
the help of sentences in particular language. “A sentence written as ~p and read as ‘not p’. Negation of any statement
is called a mathematically acceptable statement or p is formed by writing “It is not the case that .....”
proposition if it is either true or false but not both.” A
statement is assumed to be either true or false. A true p ~p
statement is known as a valid statement and a false T F
statement is known as an invalid statement. F T

2. TRUTH TABLE : Truth Table


Truth table is that which gives truth values of statements. or “It is false that.....
It has a number of rows and columns. Note that for n
or inserting the word “not” in p.
statements, there are 2n rows,
(i) Truth table for single statement p: 4. COMPOUND STATEMENTS :
Number of rows = 21 = 2 If a statement is combination of two or more statements,
then it is said to be a compound statement. Each statement
which form a compound statement is known as its sub-
statement or component statement.
5. BASIC CONNECTIVES :
(ii) Truth table for two statements p and q : In the compound statement, two or more statements are
connected by words like ‘and’, ‘or’, if.... then’, ‘only if’, ‘if
and only if’, ‘there exists’, ‘for all’ etc. These are caled
p q
T T connectives. When we use these compound statements,
T F it is necessary to understand the role of these words.
F T 6. THE WORD “ AND” (CONJUCTION) :
F F
Any two statements can be connected by the word “and”
to form a compound statement. The compound statement
Number of rows = 22 = 4 with word “and” is true if all its component statements are
(iii) Truth table for three statements p, q and r. true. The compound statement with word “and” is false if
any or all of its component statement with word “and” is
Number of rows = 23 = 8
false if any or all of its component statements are false.
The compound statement “p and q” is denoted by “p ^
p q r q”.
T T T
T T F
p q p^q
T F T
T T T
T F F
T F F
F T T F T F
F T F F F F
F F T
F F F
Truth Table
136 MATHEMATICAL REASONING

7. THE WORD “OR” (DISJUNCTION) :


p q pq qp
Any two statements can be connected by the word “OR” T T T T
to form a compound statement. The compound statement T F F T
with word “or” is true if any or allof its component F T T F
statements are true. The compound statement with word F F T T
“or” is false if all its component statement are false. The Truth Table
compound statement “p or q” is denoted by “p v q:
Examples:
p q pvq (i) If x = 4, then x2 = 16
T T T
T F T (ii) If ABCD is a parallelogram, then AB = CD
F T T (iii) If Mumbai is in England then 2 + 2 = 5
F F F (iv) If Shikha works hard, then it will rain today.

Truth Table
10. CONTRAPOSITIVE, CONTRADICTION AND
CONVERSE OF A CONDITIONAL STATEMENT:
8. TYPES OF “OR”:
If p and q are two statements then
(i) Exclusive OR : If in statement p v q i.e., p or q, happening
of any one of p, q excludes the happening of the other Let p  q then
then it is exclusiver or. Here both p and q cannot occur (i) (Contrapositive of p  q) is (~q  ~p)
together. For example in statement “I will go to delhi either (ii) Contradiction of p  q) is (q p)
by bus or by train”, the use of ‘or’ is exclusive.
(iii) (Converse of p  q) is (q  p)
(ii) Inclusive OR : If in statement p or q, both p and q can
also occur together then it is inclusive or. The statement
‘In senior secondary exam you can take optional subject
as physical education or computers’ is an example of use
A statement and its contrapositive convey the same
of inclusive OR.
meaning.
IMPLICATION :
11. BICONDITIONAL CONNECTIVE “IF AND ONLY IF”:
There are three types of implications which are “if... then”,
“Only if” and “if and only if” If p and q are any two statements then the compound
statement in the form of “p if and only if q” is called a
9. CONDITIONAL CONNECTIVE ‘IF... THEN’: biconditional statement and is written in symbolic form as
p  q or p  q.
If p and q are any two statements then the compound
statement in the form “if p then q” is called a conditional Statement p q reveals the following facts
statement. The statement “If p then q” is denoted by p  (i) p if and only if q
q or p  q (to be read as p implies q).In the implication p
(ii) q if and only if p
 q, p is called the antecedent (or the hypothesis) and q
the consequent (or the conclusion) (iii) p is necessary and sufficient condition for q

If p then q reveals the following facts : (iv) q is necessary and sufficient condition for p

(i) p is a sufficient condition for q


p q pq qp
(ii) q is necessary condition for p T T T T
(iii) ‘If p then q’ has same meaning as that of ‘p only if q’ T F F T
(iv) p  q has same meaning as that of ~q ~p F T F F
F F T T
Truth Table
MATHEMATICAL REASONING 137

For Example : (iii) p ^ (q  r)  ( p ^ q) ^ (p ^ r)


The following statements are biconditional statements (iv) p  (q  r)  (p  q)  (p  r)
(i) A number is divisible by 3 if and only if the sum of the (4) Identity Law :
digits forming the number is divisible by 3. (i) p  t = t (ii) p ^ t = p
(ii) One is less then seven if and only if two is less then (iii) p  c = p (iv) p ^ c = c
eight.
(5) Complement Law :
(iii) A triangle is equilateral if and only if it is equiangular.
(i) p  (~p) = t (ii) p^ (~p) = c
12. TAUTOLOGY AND FALLACY/CONTRADICTION: (iii) ~t = c (iv) ~c = t (v) ~(~p) = p
(6) Idempotent Law :
(a) Tautology : This is a statement which is true for all
truth values of its components. It is denoted by t. consider (i) p  p = p (ii) p ^ p = p
truth table of pv ~p (7) De Morgan’s Law
(i) ~ (p  q) = (~p) ^ (~q)
p ~p p v ~p
T F T (ii) ~ (p ^ q) = (~p)  (~q)
F T T (8) Involution laws (or Doube negation laws):

We observe that last column is always true. Hence ~  ~ p  p


p ~ p is a tautology.. (9) Contrapositive Laws : p  q  ~q ~p
(b) Fallacy (contradiction) : This is staement which is
15. NEGATION OF COMPOUND STATEMENTS :
false for all truth values of its components. It is denoted
by f or c. Consider truth table of p ^ ~p If p and q are two statements then
p ~p p ^ ~p (i) Negation of conjuction : ~ (p ^ q)  ~ p  ~q
T F F (ii) Negation of disjunction : ~ (p  q)  ~p ^ ~q
F T F (iii) Negation of conditional : ~(p  q)  p ^ ~q

we observe that last column is always false. Hence p ^ ~p (iv) Negation of biconditional : ~(p  q)  (p ^ ~q) v (q
is a fallacy (contradiction). ^ ~q) or p  ~q
we known that p  q  ( p  q) ^ (q p)
13. LOGICALLY EQUIVALENT STATEMENTS :
~(p  q) ~[(p  q) ^ (q  p)]
If truth values of statements p and q are same then they ~(p  q)  ~ (q  p) (p ^ ~q)  (q ^ ~q)
are logically equivalent and written as p = q.
Summary
14. ALGEBRA OF STATEMENTS : (i) ~ (p^ q)  (~p)  (~q)
(ii) ~ (p  q)  (~p) ^ (~q)
If p, q,r are any three statements and t is a tautology, c is
a contradiction then (iii) ~(p q)~(~p  q) = p ^ (~q)

(1) Commutative Law : (iv) ~(p q)(p ^ ~q)  (q^ ~p) or p  ~q

(i) p  q = q  p (ii) p ^ q = q ^ p 16. VALIDITY OF A STATEMENT:


(2) Associative Law :
There are four methods to prove validity of a statement
(i) p  ( q  r) = (p  q) v r
(a) Direct method :
(ii) p ^ (q ^ r) = (p ^ q) ^ r
(i) To prove that “p and q” is true, show that both p and q
(3) Distributive Law :
are true
(i) p ^ (q  r) = ( p ^ q)  (p ^ r)
(ii) To prove “p or q”, show that any one of p or q is true
(ii) p  (q ^ r) = (p  q) ^ (p  r)
(iii) To prove p  q, assume that p is true and show that
q must be true
138 MATHEMATICAL REASONING

(iv) To prove p  q, show that if p is true then q is true. s1,s2,...sn ; s or


Also show that if q is true, then p is true s1,s2,...sn /-s or
(b) Contrapositive Method: s1^ s2 ^...^ sn)  s or
To prove p  q, assume that q is false and prove that p s1
must be false.
s2
(c) Contradiction Method:
s3
To prove that a statement p is true, we assume that p is
...
not true, then we arrive at some result which contradicts
our assumption. Therefore, we conclude that p is true. ...
(d) Counter example Method : sn
To show that a statement is false, we give an example So s
where the statement is not valid. Note that this method is The symbol “/-” is real as turnstile
used to disprove the statement. Giving examples in favour
An argument is said to be a valid argument if the
of a statement cannot prove that the given statement is
conclusion ‘s’ is true whenever all the hypotheses s1,s2,...sn
valid. are true or equivalentry argument is valid when it is a
17. VALIDITY OF AN ARGUMENT : tautology, otherwise the argument is called an invalid
An argument is an assertion that a given set of statements argument.
s1, s2, .... sn implies other statement ‘s’. In other words, an Method of testing the validity of argument :
argument is an assertion that the statement ‘s’ follows Step I - Construct the truth table for conditional statement
from statements s1,s2,...sn which are called hypothesis. The s1^s2^s3^...^sn  s.
statement ‘s’ is called the conclusion.
Step II - Check the last column of truth table. If the last
We denote the argument containing hypotheses s1,s2,...sn column contains T only, then the given argument is valid
and conclusion ‘s’ by otherwise it is an invalid argument.
MATHEMATICAL REASONING 139

SOLVED EXAMPLES

Example 1 : Example 6 :
Find the truth values of the compound statement
Write negation of following statement:
(p v ~r) ^ (q v ~r)
“All cats scratch”
Solution :
Solution : Some cats do not scratch
OR
p q r ~r pv~r qv~r (pv~r)^(qv~r)
There exists a cat which does not scratch T T T F T T T
OR T T F T T T T
T F T F T F F
At least one cat does not scratch.
F T T F F T F
Example 2: T F F T T T T
F T F T T T T
Write negation of statement ‘2 + 2 = 7’ F F T F F F F
F F F T T T T
Solution : 2 + 2  7

Example 3 : Example 7 :
Find the truth value of the statement “2 divides 4 and Write the contrapositive of the following statement: “If
3 + 7 = 8” Mohan is poet, the he is poor”

Solution: 2 divides 4 is true and 3 + 7 = 8 is false. so given Solution : Consider the following statements :
statement is false.
p : Mohan is a poet
Example 4 : q : Mohan is poor
Write component statements of the statement “All living Clearly, the given statement is symbolic form is p  q.
things have two legs and two eyes”. Therefore, its contrapositive is given by ~q  ~p.
Now, ~p : Mohan is not a poet.
Solution : Component statements are : ~q : Mohan is not poor
All living things have two legs ~q ~p: Mohan is not poor, then he is not a poet.
All living things have two eyes Hence the contrapositive of the given statement is “If
Example 5 : Mohan is not poor, then he is not a poet”.

Example 8 :
Find the truth values of ~ p v q
Write the converse and the contrapositive of the
Solution : statement “If x is a prime number, then x is odd”.
p q ~p ~p v q
T T F T Solution : Given statement is : “If x is a prime number then x is
T F F F odd”.
F T T T
F F T T Let p : x is prime number and q : x is odd
Given statement is p  q
The converse of p  q is q  p i.e., “If x is odd then x is
a prime number”
140 MATHEMATICAL REASONING

The contrapositive of p  q is ~q ~p i.e., “If x is not The given statement is p and q, so its negation is
odd then x is not a prime number” ~p or ~q = Some of the students did not complete their
home work or the teacher is not present.
Example 9 :
(ii) The component statement of the given statements
Write the contradiction of “If it rains, the I stay” at are:
home. p : Square of an integer is positive.
q : Square of an integer is ngeative.
Solution : If I stay at home then. It does not rain.
The given statement is p or q. so its negation is ~p and ~q
Example 10 : = Their exists an integer whose square is neither positive
nor negative.
Let p and q stand for the statement ‘Bhopal is in M.P.’
(iii) Consider the following statements :
and ‘3 + 4 = 7’ respectively. Describe the conditional
statement ~p ~q p : My car is not in workshop
q : I can go to college
Solution : ~p ~q: If Bhopal is not in M.P. then 3 + 4  7 The given statements in symbolic form is p  q
Example 11 : Now, ~ (p q) p ^ (~q)
~(p  q) : My car is not in workshop and I cannot go
Find the truth value of (p  ~q)  (p  q) to college.
Hence the negation of the given statements is “My car is
Solution :
not in workshop and i can not go to college”.
(iv) Consider the following statements:
p : ABC is an equilateral triangle.
q : It is equiangular
Clearly, the given statement in symbolic form is p  q.
Now, ~ (p  q) (p ^ ~q)  (~p ^ q)

Example 12 : ~(p  q) : Either ABC is an equilateral triangle and it is


not equiangular or ABC is not an equilateral triangle and it
Write the negation of the following compound is equiangular.
statements:
Example13 :
(i) All the students completed their homework and the
teacher is present. Show that p (p  q) is a tautology
(ii) Square of an integer is positive or negative
Solution :
(iii) If my car is not in workshop, then I can go college.
(iv) ABC is an equilateral triangle if and only if it is pq
p q p  p  q
equiangular.
T T T T
T F T T
Solution : (i) The component statements of the given statement
F T T T
are :
F F F T
p : all the student completed their homework
q : The teacher is present.
MATHEMATICAL REASONING 141

Example 14 : Example 16 :

By using laws of algebra of statements show that Show that the following argument is not valid : “If it
(p  q) ^ ~p~p ^ q) rains, crops will be good. It did not rain. Therefore the
crops were not good”.
Solution : (p  q) ^ ~p (~q) ^ (p ^ q)
 (~p ^ q)  (~p ^ q) Solution : p : it rains

 f  (~p ^ q) q : crops will be good

 ~p ^ q S1: p  q, S2 : ~p S : ~q

Example 15 : p q S1 S2 S
T T T F F
Find the negation of statement p ^ ~q
T F F F T
F T T T F
Solution : Negation of (p ^ ~q)  ~(p ^ ~q) F F T T T
~ p  ~~q  ~p  q
Not valid
142 MATHEMATICAL REASONING

EXERCISE - 1 : BASIC OBJECTIVE QUESTIONS


1. Which of the following is a logical statement ? 9. The negation of the statement ‘ 2 is not a complex
(a) Open the door number’ is
(b) What an intelligent student
(a) 2 is a rational number
(c) Are you going to Delhi
(d) All prime numbers are odd numbers (b) 2 is an irrational number

2. Which of the following is a statement? (c) 2 is a real number


(a) x is a real number (b) Switch off the fan
(d) 2 is complex number
(c) 6 is a natural number (d) Let me go
3. Which of the following is not a statement? 10. Let S be a non-empty subset of R. Consider the following
statement :
(a) Smoking is injurious to health
(b) 2 + 2 = 4 p : There is a rational number x  S such that x > 0. Which
of the following statement(s) is the negation of the
(c) 2 is the only even prime number
statement p?
(d) Come here
(a) There is a rational number x S such that x  0
4. The sentence “New Delhi is in India”, is
(b) There is no rational number x S such that x  0
(a) a statement
(b) Not a statement (c) Every rational number x S satisfied x  0

(c) may be statement or not (d) x S and x  0  x is not rational.

(d) None of the above 11. The negation of the statement “101 is not a multiple of 3”
is
5. Which of the following is/are not a statement?
(a) 101 is a multiple of 3 (b) 101 is a multiple of 2
(I) Earth is a planet.
(c) 101 is an odd number (d) 101 is an even number
(II) Plants are living objects.
12. Which of the following is not a negation of the statement
(III) 3 is a rational number.. “A natural number is greater than zero”
(IV) x2 – 5x + 6 < 0, when x R. (a) A natural number is not greater than zero
(a) I and II (b) II and IV (b) It is false that a natural number is greater than zero.
(c) III and I (d) None of these (c) It is false that a natural number is not greater than zero.
6. The negation of the statement “A circle is an ellipse” is (d) None of the above
(a) an ellipse is a circle (b) an ellipse is not a circle 13. Which of the following is/are connectives?
(c) a circle is not an ellipse (d) a circle is an ellipse (a) Today (b) Yesteday
7. The negation of the statement “7 is greater than 8” is (c) Tommorow (d) “And”, “or”
(a) 7 is equal to 8 (b) 7 is not greater than 8 14. The component statements of the statement “The sky is
(c) 8 is less than 7 (d) None of these blue or the grass is green” are

8. Consider the statement p : “New Delhi is a city”. Which of (a) p: The sky is blue, q : The sky is not blue
the following is not negation of p? (b) p : The sky is blue, q : The grass is green
(a) New Delhi is not a city (c) Both (a) and (b)
(b) It is false that New Delhi is a city (d) None of the above
(c) It is not the case that New Delhi is not a city
(d) None of these
MATHEMATICAL REASONING 143

15. The component statement of the compound statement “If 21. The negation of the statement “If a quadrilateral is a square
you are born in some country, then you are a citizen of the then it is a rhombus”.
country” are (a) If a quadrilateral is not a square then it is a rhombus
p : You are born in some country (b) If a quadrilateral is a square then it is not a rhombus
q : You are a citizen of that country (c) a quadrilateral is a square and it is not a rhombus
Then, the correct option is (d) a quadrilateral is not a square and it is a rhombus
(a) If p is true, then q must be true 22. Negation of the statement p  (q ^ r) is
(b) If p is true, then q is not true
(a) ~ p ~ (q ^r) (b) ~p  (q ^r)
(c) Both p and q are false (c) (q ^ r) p (d) p ^ (~q  ~r)
(d) None of the above 23. The contrapositive of the statement
16. For the compound statement “If 7 is greater than 5, then 8 is greater than 6” is
All prime numbers are either even or odd’. Which of the (a) If 8 is greater than 6, then 7 is greater than 5
following is true?
(b) If 8 is not greater than 6, then 7 is greater than 5
(a) Both component statement are false
(c) If 8 is not greater than 6, then 7 is not greater than 5
(b) Exactly one of the component statements is true
(d) If 8 is greater than 6, then 7 is not greater than 5
(c) Atleast one of the component statements is true
24. The converse of the statement
(d) Both the component statements are true.
“If x > y, then x + a > y + a” is
17. Which of the following is the conditional p  q?
(a) if x < y, then x + a < y + a
(a) q is sufficient for p (b) If x + a > y + a, then x > y
(b) p is necessary for q
(c) If x < y, then x + a > y + a
(c) p only if q (d) If x > y, then x + a < y + a
(d) if q, then p
25. The contrapositive of (p ^ q)  r is
18. The statement ‘x is an even number implies that x is
(a) ~r (p  q) (b) r (p  q)
divisible by 4’ means the same as
(c) ~r (~p  ~q) (d) p (q  r)
(a) x is divisible by 4 is necessary condition for x to be an
26. The converse of the statement ‘p implies q’ is
even number
(a) ~q implies ~p (b) q implies p
(b) x is an even number is a necessary condition for x to
(c) p only if q (d) ~p implies q
divisible by 4
27. Which of the following is logically equivalent
(c) x is divisible by 4 is a sufficient condition for x to be an
to ~ (p  q)
even number
(a) (~p)  q (b) (~p) (~q)
(d) x is divisible by 4 implies that x is not always an even
(c) p (~q) (d) p  q
number
28. The converse of the statement “If sun is not shining, then
19. If p is true and q is false, then which of the following
sky is filled with clouds” is
statement is not true?
(a) If sky is filled with clouds, then the sun is not shining
(a) p  q (b) p q
(c) p ^ (~q) (d) q p (b) If sun is shining, then sky is filled with clouds
20. The negation of the statement “Ramesh is cruel or he is (c) If sky is clear, then sun is shining
strict” is (d) If sun is not shining, then sky is not filled with clouds
(a) Ramesh is neither cruel nor strict
(b) Ramesh is cruel or he is not strict
(c) Ramesh is not cruel or he is strict
(d) Ramesh is not cruel and he is strict
144 MATHEMATICAL REASONING

29. The contrapositive of “If two triangle are identical, then (a) Statement I is true. Statement II is true; Statement II is
these are similar” is ...X...Here, X refers to a correct explanation for Statement I.
(a) If two triangle are not similar, then these are not identical (b) Statement I is true; Statement II is true; Statement II is
(b) If two triangle are not identical, then these are not not a correct explanantion for Statement I
similar (c) Statement I is true; Statement II is false
(c) Both (a) and (b) (d) Statement I is false; Statement II is true.
(d) None of the above 36. Let p : 25 is multiple of 5.
30. The contrapositive of the statement “If p, then q” is q : 25 is multiple of 8
(a) If q, then p (b) If p, then ~q Statement I : The compound statement “p and q” is false.
(c) If ~q, then ~p (d) If ~p, then ~q Statement II: The compound statement “p or q” is false
31. The statement “If x2 is not even, then x is not even” is Chose the correct option
converse of the statement
(a) Only statement I is true (b) Only statement II is true
(a) If x2 is odd, then x is even
(c) Both statements are true
(b) If x is not even then x2 is not even (d) Both statements are false
(c) If x is even, then x2 is even 37. The negation of the statement
(d) If x is odd, then x2 is even p : “for every positive real number x, the number x - 1 is
32. For the statement “If you drive over 80 km/h, then you will also positive” is
get a fine”, the sufficient condition is ...X...Here, X refer to (a) “there exists atleast one positive real number x for which
(a) getting a fine (x - 1) is not positive”
(b) driving over 80 km/h (b) “for every positive real number x, the number (x + 1) is
(c) Both (a) and (b) also positive”
(d) None of these (c) Both (a) and (b)
33. The statement p : For any real numbers x, y if x = y, then (d) Neither (a) nor (b)
2x + a = 2y + a when aZ
38. Negation of the statement S : “There exist a number x such
(a) is true that 0 < x < 1” is ...P... Here, P refers to
(b) is false
(a) there does not exist a number such that 0 < x < 2
(c) its contrapositive is not true
(d) None of the above (b)there does not exist a number such that 0 < x < 1

34. If p : “100 is a multiple of 4 and 5” (c) Both (a) and (b)

Statement I : p is true (d) None of the above

Statement II: Both the component statements of p are 39. Consider the statement
true. p : “It is necessary to have a password to log on to the
Choose the correct option server”.

(a) Statement I is true. Statement II is true; Statement II is Then, “If-then” form of this statement is
a correct explanation for Statement I. (a) If you log on to sever, then you have a password
(b) Statement I is true; Statement II is true; Statement II is (b) If you log on to server, then you do not have a password
not a correct explanantion for Statement I (c) Both (a) and (b)
(c) Statement I is true; Statement II is false (d) None of the above
(d) Statement I is false; Statement II is true. 40. If x = 5 and y = – 2 then x – 2y = 9. The contrapositive of
35. Let q: “60 is a multiple of 3 or 5”. this statement is
Statement I : q is true (a) If x - 2y = 9 then x = 5 and y = – 2
Statement II : Both the component statements of q are (b) If x - 2y  9 then x  5 and y – 2
true. (c) If x - 2y  9 then x  5 and y – 2
Choose the correct option (d) If x - 2y  9 then either x  5 or y  – 2
MATHEMATICAL REASONING 145

EXERCISE - 2 : PREVIOUS YEAR JEE MAINS QUESTIONS


1. The statement p  (q  p) is equivalent to (2008) 5. Consider the following statements (2011)
(a) p  (p q) (b) p  (p  q) P : Suman is brilliant
(c) p  (p v q) (d) p  (p ^ q) Q : Suman is rich
2. Let p be the statement “x is an irrational number”, q be the R : Suman is honest
statement “y is a transcendental number”, and r be the The negation of the statement “Suman is brilliant and
statement “x is a rational number iff y is a transcendental dishonest if and only if Suman is rich” can be expressed
number”. (2008) as
Statement–1 : r is equivalent to either q or p. (a) ~ Q ~ P ^ R
Statement–2 : r is equivalent to ~ (p ~ q). (b) ~ (P ^ ~ R) Q
(a) Statement–1 is true, Statement–2 is false (c) ~ P ^ (Q ~ R)
(b) Statement–1 is false, Statement–2 is true (d) ~ (Q (P ^ ~ R))
(c) Statement–1 is true, Statement–2 is true; 6. The negation of the statement “If I become a teacher, then
Statement–2 is a correct explanation for Statement–1 I will open a school”, is (2012)
(d) Statement–1 is true, Statement–2 is true; (a) Neither I will become a teacher nor I will open a school.
Statement–2 is not a correct explanation for Statement–1 (b) I will not become a teacher or I will open a school.
3. Statement–1 : ~ (p ~q) is equivalent to p q. (c) I will become a teacher and I will not open a school.
Statement–2 : ~ (p ~q) is a tautology. (2009) (d) Either I will not become a teacher or I will not open a
(a) Statement–1 is true, Statement–2 is true; school.

Statement–2 is not a corrrect explanation for Statement–1 7. Consider : (2013)

(b) Statement–1 is true, Statement–2 is false Statement–1 : (p ^ ~q) ^ (~ p ^ q) is a fallacy.

(c) Statement–1 is false, Statement–2 is true Statement–2 : (p q) (~ q ~ p) is a tautology.

(d) Statement–1 is true, Statement–2 is true; (a) Statement–1 is true, Statement–2 is true,

Statement–2 is correct explanation for Statement–1 Statement–2 is not a correct explanation for Statement–1.

4. Let S be a non-empty subset of R. Consider the following (b) Statement–1 is true, Statement–2 is false.
statement: (2010) (c) Statement–1 is false, Statement–2 is true.
p : There is a rational number x S such that x > 0. (d) Statement–1 is true, Statement–2 is true,
Which of the following statements is the negation of the Statement–2 is a correct explanation for Statement–1.
statement P ? 8. The statement ~ (p  ~ q) is (2014)
(a) There is a rational number x S such that x 0. (a) equivalent to ~ p  q
(b) There is no rational number x S such that x 0. (b) a tautology
(c) Every rational number x S satisfies x 0. (c) a fallacy
(d) x S and x 0 x is not rational. (d) equivalent to p  q
9. The contrapositive of the statement “I go to school if it
does not rain” is : (Online Set–1 2014)
(a) If it rains, I do not go to school.
(b) If I do not go to school, it rains.
(c) If it rains, I go to school.
(d) If I go to school, it rains.
146 MATHEMATICAL REASONING

10. The proposition ~ (p  ~q)  ~ (p  q) is logically


equivalent to : (Online Set–2 2014)
15. The Boolean expression  p ~ q   q   ~ p  q  is
(a) p (b) q equivalent to (2016)

(c) ~p (d) ~ q (a) p ~ q (b) ~ p  q


11. Let p, q, r denote arbitrary statements. Then the logically (c) p  q (d) p  q
equivalent of the statement p (q  r) is :
16. Consider the following two statements :
(Online Set–3 2014) (Online Set–1 2016)
(a)  p  q   r (b)  p  q   (p  r) P : If 7 is an odd number, then 7 is divisible by 2.
Q : If 7 is a prime number, then 7 is an odd number.
(c)  p  ~ q   (p  r) (d)  p  q   (p  ~ r)
If V1 is the truth value of the contrapositive of P and V2 is
12. The contrapositive of the statement “if I am not feeling the truth value of contrapositive of Q, then the ordered
well, then I will go to the doctor” is : pair (V1, V2) equals
(Online Set–4 2014) (a) (F, F) (b) (F, T)
(a) If I am feeling well, then I will not go to the doctor
(c) (T, F) (d) (T, T)
(b) If I will go to the doctor, then I am feeling well
17. The contrapositive of the following statement, “If the side
(c) If I will not go to the doctor, then I am feeling well of a square doubles, then its area increases four times”, is
(d) If I will go to the doctor, then I am not feeling well
(Online Set–2 2016)
13. The contrapositive of the statement “If it is raining, then I
(a) If the area of a square increases four times, then its side
will not come”, is : (Online Set–1 2015)
is not doubled.
(a) If I will come, then it is not raining.
(b) If the area of a square increases four times, then its
(b) If I will not come, then it is raining.
side is doubled.
(c) If I will not come, then it is not raining.
(c) If the area of a square does not increase four times,
(d) If I will come, then it is raining. then its side is not doubled.
14. Consider the following statements :
(d) If the side of a square is not doubled, then its area does
(Online Set–2 2015) not increase four times.
P : Suman is brilliant. 18. The following statement (pq)  [~ p  q)  q] is
Q : Suman is rich. (2017)
R : Suman is honest. (a) equivalent to ~p  q (b) equivalent to p  ~q
The negation of the statement, (c) a fallacy (d) a tautology
“Suman is brilliant and dishonest if and only if Suman is
rich” can be equivalently expressed as : 19. The proposition  ~ p    p ~ q  is equivalent to

(a) ~ Q  ~ P  R (Online Set–1 2017)


(a) p ~ q (b) p ~ q
(b) ~ Q  ~ P  R
(c) p  ~ q (d) q  p
(c) ~ Q  ~ P ~ R

(d) ~ Q  P  ~ R
MATHEMATICAL REASONING 147

20. Contrapositive of the statement ‘If two numbers are not 23. Consider the following two statements :
equal, then their squares are not equal’ is Statement P :
(Online Set–2 2017) The value of sin 1200 can be derived by taking  = 2400 in
(a) If the squares of two numbers are not equal, then the the equation
numbers are equal. 
2sin = 1 + sin  - 1 - sin  .
(b) If the squares of two numbers are equal, then the 2
numbers are not equal. Statement q :
The angles A, B, C and D of any quadrilateral ABCD satisfy
(c) If the squares of two numbers are equal, then the the equation
numbers are equal.
1  1 
(d) If the squares of two numbers are not equal, then the cos   A + C   + cos   B + D   = 0
2  2 
numbers are not equal.
Then the truth values of p and q are respectively :
21. The Boolean expression   p  q     p ^ q  is (2018/Online Set–2)
equivalent to : (2018) (a) F, T (b) T, F
(c) T, T (d) F, F
(a)  q (b)  p
24. If p  ( p   q) is false, then the truth values of p and
(c) p (d) q q are respectively :
22. If (p  q)  (p  r)  p  q is false, then the truth (2018/Online Set–3)
values of p, q and r are, respectively: (a) F, F (b) T, F
(2018/Online Set–1) (c) F, T (d) T, T
(a) F, T, F (b) T, F, T
(c) T, T, T (d) F, F, F
148 MATHEMATICAL REASONING

ANSWER KEY
EXERCISE - 1 : BASIC OBJECTIVE QUESTIONS

1. (d) 2. (c) 3. (d) 4. (a) 5. (d) 6. (c) 7. (b) 8. (c) 9. (d) 10. (c)
11. (a) 12. (c) 13. (d) 14. (b) 15. (a) 16. (a) 17. (c) 18. (a) 19. (b) 20. (a)
21. (c) 22. (d) 23. (c) 24. (b) 25. (c) 26. (b) 27. (a) 28. (a) 29. (a) 30. (c)
31. (b) 32. (b) 33. (a) 34. (a) 35. (a) 36. (a) 37. (a) 38. (b) 39. (a) 40. (d)

EXERCISE - 2 : PREVIOUS YEAR JEE MAINS QUESTIONS

1. (c) 2. (a) 3. (b) 4. (c) 5. (d) 6. (c) 7. (a) 8. (d) 9. (b) 10. (c)
11. (b) 12. (c) 13. (a) 14. (d) 15. (d) 16. (b) 17. (c) 18. (d) 19. (c) 20. (c)
21. (b) 22. (b) 23. (a) 24. (d)

Dream on !!


You might also like